Download as pdf or txt
Download as pdf or txt
You are on page 1of 285

Levofloxacin (Levaquin) 750 mg IVPB is prescribed for a client with pneumonia.

The dose is
available in 150 mL of 5% dextrose and is to infuse over 90 minutes. The administration set has
a drop factor of 15 drops per mL. At how many drops per minute should the nurse regulate the
IVPB to infuse? Record your answer using a whole number.
25

A client is diagnosed with emphysema (COPD). For what long-term problem should the
nurse monitor this client?

localized tissue necrosis

carbon dioxide retention

increased respiratory rate

saturated hemoglobin molecules

A nurse assesses that several clients have low oxygen saturation levels. Which client
would benefit the most from receiving oxygen via a nasal cannula?

has an upper respiratory infection

receives many visitors while sitting in a chair

has a nasogastric tube for gastric decompression

exhibits dry oral mucous membranes from mouth breathing

Which of the following are manifestations of a client with COPD?

increased anteroposterior chest diameter

underdeveloped neck muscles.

collapsed neck veins.

Increased chest excursions with respiration.

Which of the following diets would be most appropriate for a client with chronic
obstructive pulmonary disease (COPD)?

low fat, low cholesterol diet

bland, soft diet


low sodium diet

high calorie, high protein diet

The nurse cares for a client with chronic obstructive pulmonary disease (COPD). The
physician orders oxygen via nasal cannula for this client. Which action should the nurse
take?

Teach the client to adjust the oxygen rate.

Change the oxygen tubing each shift.

Increase oxygen to 6 litre per minute as needed.

Maintain oxygen at 3 litre per minute or less.

The nurse assesses the respiratory status of a client who is experiencing an


exacerbation of chronic obstructive pulmonary disease (COPD) secondary to an upper
respiratory tract infection. Which of the following findings would be expected?

normal breath sounds

prolonged inspiration

normal chest movement

coarse crackles and rhonchi

Which of the following indicates that the client with chronic obstructive pulmonary
disease (COPD) who has been discharged to home understands the care plan?

The client promises to do pursed-lip breathing at home

The client states actions to reduce pain.

The client will use oxygen via a nasal cannula at 5 L/min.

The client agrees to call the physician if dyspnea on exertion increases.

The nurse instructs a client diagnosed with chronic obstructive pulmonary disease
(COPD) about positions to use during times of dyspnea. The nurse recognizes further
teaching is necessary when the client states:

I will lie flat on my back.


I will sit up and rest my elbows on my knees.

I will lean up against a wall

I will sit up and lean over a table.

When developing a discharge plan to manage the care of a client with chronic
obstructive pulmonary disease (COPD), the nurse should advise the client to expect to

develop respiratory infections easily

maintain current status

require supplemental oxygen

show permanent improvement

The nurse is preparing to administer vancomycin 500 mg 1V to the client with


pneumonia. The vancomycin is supplied in a 100-mL piggyback that is to be given over
1 hour. At what rate in milliliters per hour should the nurse set the infusion pump?

____50___ ml/hour (record your answer as a whole number)

The nurse is providing teaching to the client with COPD about the purpose of pursed-lip
breathing. Which explanation is most appropriate?

It reduces upper airway inflammation.

It strengthens the respiratory muscles

It improves inhaled drug effectiveness.

It reduces anxiety by slowing the heart rate.

What nursing action will most help a client obtain maximum benefits after postural
drainage?

administer prn oxygen

encourage coughing deeply

place the client in a sitting position

encourage the client to rest for a half hour


A client's arterial blood gas values are as follows: pH, 7.31; PaO2 , 80 mm Hg; PaCO2 ,
65 mm Hg; HCO3 – , 36 mEq/L. The nurse should assess the client for

cyanosis

flushed skin

irritability

anxiety

The nurse reviews arterial blood gas results for his client with chronic obstructive
pulmonary disease (COPD). The pH is 7.3, PaCO2 is 56 mm Hg, and HCO3 is 24
mEq/L. Which acid–base imbalance is this client experiencing?

respiratory acidosis

metabolic acidosis

respiratory alkalosis

metabolic alkalosis

The nurse reviews an arterial blood gas report for a client with chronic obstructive
pulmonary disease (COPD). The results are: pH 7.35; PCO2 62 (8.25 kPa); PO2 70
(9.31 kPa) (34 mmol/L); HCO3 34. The nurse should first:

apply a 100% non rebreather mask

assess the vital signs.

reposition the client.

prepare for intubation


The nurse enters the client’s room after hearing the pulse oximeter alarming and sees
the tracing illustrated on the screen. Which action should be immediately taken by the
nurse?
Call for the acute respiratory response team.

Remove the machine and call maintenance.

Give oxygen through a nasal cannula or mask.

Assess the level of consciousness and skin color.


.
MAJOR QUIZ

1.
0.8 / 1 point
These are all considered as immunologic criteria of systemic lupus
erythematosus. Select all that applies:

Your Answer:

Antinuclear antibodies (ANA) level above laboratory reference range

Anti-double stranded deoxyribonucleic acid (dsDNA) antibody level above laboratory


Reference range [or >2-fold the reference range if tested by enzyme-linked
immunosorbent assay (ELISA)

Anti-Smith (Anti-Sm): presence of antibody to Smith (Sm) nuclear antigen

Low complement (C3, C4, or CH50)

It is a spectrum of chronic idiopathic inflammatory condition


2.
1 / 1 point
A type of inflammatory response where a reaction to sun or light that causes a
skin rash to appear or get worse

Your Answer: Correct

Malar Rash

Discoid Rash

Serositis

Photosensitivity
3.
1 / 1 point
A 26-year-old woman has been diagnosed with early systemic lupus
erythematosus (SLE) involving her joints. In teaching the patient about the
disease, the nurse includes the information that SLE is a(n):

Your Answer: Correct

Hereditary disorder of women but usually does not show clinical symptoms unless a
woman becomes pregnant.

Autoimmune disease of women in which antibodies are formed that destroy all
nucleated cells in the body.

Disorder of immune function, but it is extremely variable in its course, and there is no
way to predict its progression.

Disease that causes production of antibodies that bind with cellular estrogen receptors,
causing an inflammatory response.
4.
1 / 1 point
Lupus is an infection

Your Answer: Correct

True

False
5.
1 / 1 point
A patient with an acute exacerbation of systemic lupus erythematosus (SLE) was
hospitalized with incapacitating fatigue, acute hand and wrist pain, and
proteinuria. The health care provider prescribes prednisone (Deltasone) 40 mg
twice daily. Which nursing action should include in the plan of care?

Your Answer: Correct

Institute seizure precautions.

Reorient to time and place PRN.


Monitor intake and output

Place on cardiac monitor.


6.
1 / 1 point
People do not die from lupus.

Your Answer: Correct

True

False
7.
1 / 1 point
Nurse Dakota is making morning rounds, and as she assists the patient to dress,
she recognizes that the patient's fingers are white in color and cold to touch. The
nurse is correct when she thinks that the condition is

Your Answer: Correct

Arthritis

Arthralgia

Raynaud's Phenomenon

Seizures
8.
1 / 1 point
Nurse Paco is asking for the health history and family health history of the
patient. Which among the options does not apply to ask?

Your Answer: Correct

Have your parents or siblings had lupus or other autoimmune disorders?

Have your neighbors been diagnosed with any other medical conditions?

What medications and supplements do you take regularly?


When did your symptoms begin? Do they come and go?
9.
1 / 1 point
A patient reports painful swelling joints with a history of systemic lupus
erythematosus (SLE). What kind of prescription medication should the nurse
expect?

Your Answer: Correct

Nonsteroidal anti-inflammatory drug (NSAID)

Immunosuppressive agent

Antimalarial drug

Topical corticosteroid
10.
1 / 1 point
When a symptom becomes significantly worse than it was previously is called
_______.

Use UPPER-CASE letters, plural or singular form is accepted

Your Answer: Correct


FLARE
11.
1 / 1 point
A client with systemic lupus erythematosus undergoes an entrance assessment
by a clinic nurse (SLE). Which cutaneous sign of SLE is most likely to be
assessed by the nurse?

Your Answer: Correct

Urticaria

Butterfly-shaped rash

Purpura
Photosensitivity
12.
1 / 1 point
When symptoms are inactive and under control is called _______.

Use UPPER-CASE letters, plural or singular form is accepted

Your Answer: Correct


REMISSION
13.
1 / 1 point
Student nurse John is correct when he stated that the goal of treatment in SLE
are

Your Answer: Correct

Remission of symptoms

Prevention of disease flares and organ damage

Minimizing drug side effects

Flare of symptoms
14.
1 / 1 point
Although the exact cause of lupus isn't yet known, which of these are believed to
be factors?

Your Answer: Correct

Inherited gene

Exposure to ultraviolet light

Estrogen

Testosterone
A, B, C

All of the above


15.
0.75 / 1 point
The head nurse assessed the knowledge of the student nurse regarding SLE. The
student nurse is correct when stating that the complication of SLE include

Your Answer: Partially correct

Glaucoma

Lung Cancer

Inflammation of the kidneys

Dislocated shoulders
16.
1 / 1 point
Who are the people who are most likely to develop lupus?

Your Answer: Correct

Men 15 to 45 years of age

Women 15 to 45 years of age

Men 45 to 70 years of age

Women 45 to 70 years of age


17.
1 / 1 point
Raynaud syndrome is characterized by all of the following, EXCEPT:

Your Answer: Correct


Results from vasospasms

Caused by exposure to heat

Occurs in the hands and feet

Causes the hands to turn red, white, or blue


18.
1 / 1 point
Patients with lupus taking an antimalarial drug should have which examination
before treatment and annually during treatment?

Your Answer: Correct

Pap test

Ear examination

Eye examination

Colorectal examination
19.
0.67 / 2 points
A 20-year-old African-American woman visits the clinic because she had generalized
joint discomfort in her hips and knees for the previous five months. During this period,
she has also noticed a rash on her nose and cheeks. Her medical history is
unremarkable, and she does not suffer from any allergies. Her current temperature is
100.5°F. A physical exam is notable for warmth and swelling in both of the patient’s
knees. Lab studies are performed and shown below:

 Hemoglobin: 10.7 g/dL


 ESR: 43 mm/h
 Serum urea nitrogen: 31 mg/dL
 Creatinine: 1.9 mg/dL

What is the symptom disclosed by the patient that diagnoses the relevance of
SLE?

Your Answer:

Joint pain
Malar rash

Hemoglobin: 10.7 g/dL

ESR: 43 mm/h

Serum urea nitrogen: 31 mg/dL

Creatinine: 1.9 mg/dL


20.
0 / 1 point
Which medication below is used to treat Systemic Lupus Erythematosus
decreases inflammation quickly, is not for long-term usage, and can lead to
weight gain, susceptibility to infection, diabetes, and osteoporosis?

Your Answer:

Hydroxychloroquine

Prednisone.

Azathioprine

Belimumab
21.
1 / 1 point
You're giving a Systemic Lupus Erythematosus support group information on
how to avoid flares. Which statement by a participant requires re-education about
this topic?

Your Answer: Correct

"Emotional stress and illness are triggers for a flare-up."

"I always wear large-brimmed hats and long sleeves when I'm outside."
"Exercise should be avoided due to the physical stress it causes on the body."

"I will make it a priority to receive my yearly influenza vaccine.


22.
1 / 1 point
Nurse Schmidt needed further teaching when he stated that this medication could
induce lupus.

Your Answer: Correct

Isoniazid

Procainamide

Plaquenil

Hydralazine.
23.
1 / 1 point
On a client with systemic lupus erythematosus, a complete blood cell count is
performed. The nurse would suspect that which of the following findings will be
reported from this blood test?

Your Answer: Correct

Increased red blood cell count

Decrease of all cell types

Increased white blood cell count

Increased neutrophils
24.
1 / 1 point
How many unique criteria (least) must a patient meet simultaneously or in
sequential evaluation to get diagnosed with systemic lupus erythematosus
(SLE)?
Your Answer: Correct

Six

Five

Three

Four
25.
1 / 1 point
A patient voices concerns about taking procainamide. States that a friend was
recently hospitalized for drug-induced lupus erythematosus. What is your best
response?

Your Answer: Correct

"Procainamide is not typically associated with drug-induced SLE."

"If this hasn't occurred yet, there is no continued risk of drug-induced SLE."

"Drug-induced SLE generally occurs months to a year after continuous use of this drug."

"Drug-induced SLE is a common side effect of medication therapy, particularly this


medication."
26.
1 / 1 point
It is the chemical markers released by disease-fighting cells to target a foreign
body.

Your Answer: Correct

Autoantibody

Antigens

Antibody

Neutrophils
27.
1 / 1 point
A patient is diagnosed with systemic lupus erythematosus (SLE). What
neurologic manifestations should the nurse closely monitor? Select all that apply.

Your Answer: Correct

Stroke

Seizures

Alopecia

Synovitis

Psychosis
28.
1 / 1 point
A client diagnosed with lupus is experiencing pulmonary interstitial fibrosis.
Which classification of lupus should the nurse suspect?

Your Answer: Correct

Systemic

Drug-induced

Discoid

Cutaneous
29.
0 / 1 point
Your immune system is made up of white blood cells that recognize the antigens
from the invaders. And your white blood cells create antibodies to mark those
invaders as targets for your immune system to destroy them.

In autoimmune diseases, your white blood cells mistakenly create antibodies to


mark some of your body cells for destruction. As the antibodies attached to your
normal cells, white blood cells begin to attack them as if they were foreign.
Your Answer: Incorrect

1st statement is correct & 2nd statement is incorrect

1st statement is incorrect & 2nd statement is correct

Both statements are correct

Both statements are incorrect


30.
? / 0 points
State a situation that can arise during your shift to your patient Faustina. Give
one appropriate independent nursing action you can do concerning the current
condition of your patient.

Your Answer: Awaiting score


¶tell patient faustina to drink lots of water.

Logout

10/20
A condition of multisystem, inflammatory, autoimmune disorder characterized by
formation of autoantibodies directed against self-antigens and immune-complex
formation resulting in damage to essentially any organ.

(Title Case)
Answer: Systemic Lupus Erythematosus
Give an example of medication that can cause a lupus-like disease when prescribed.

(Sentence case)
Corticosteroids

Exacerbation of SLE might increase due to these factors. (SATA)

Surgery

Gastrointestinal upset
Fever

Pregnancy

Divorce

Hypotension
A type of inflammatory response where a reaction to sun or light causes a skin rash to
appear or get worse.

Serositis

Malar rash

Discoid Rash

Photosensitivity

Prednisone was prescribed for a patient with SLE. What nursing action should be
included?

Side rails up to prevent from falling.

Set a cardiac monitoring.

Establishing a seizure precaution.

I&O monitoring.

70-90% of which group of people does lupus make up?

Women in their 50's

Older men

Women of child-bearing age

Asians

If patient is asking on how does SLE affects her life, nurse's reply should be:

"You should enjoy your life now with your families, you will be expired soon."
"It is hard to tell because the severity and progression of the disease varies.

"Using a long-term corticosteroid can manage SLE, but your life span is lessened."

"SLE causes death once in a while, you can still plan to have a normal life."

General manifestations of SLE are:

Weight gain

Abdominal pain

Fatigue

Rash

Fever

Alopecia

Nausea & vomiting

Joint pain
They are all to be considered as immunologic criteria of systemic lupus erythematous,
select all that applies:

Anti-double stranded deoxyribonucleic acid (dsDNA) antibody level above laboratory


Reference range [or >2-fold the reference range if tested by enzyme-linked
immunosorbent assay (ELISA)

Low complement (C3, C4, or CH50).

Anti-Smith (Anti-Sm): presence of antibody to Smith (Sm) nuclear antigen

Antinuclear antibodies (ANA) level above laboratory reference range.


In the clinical management of Systemic Lupus Erythematosus, monitoring should all be
considered, but one:

Complete Blood Count

Physical exam

History taking
Skin biopsy
Patient Lyn is INCORRECT about preventing flares in SLE if she states that:

"I should avoid working out as it can cause physical stress."

"An influenza vaccine a year is a priority"

"When I plan to go outside, I should wear sun hats and long clothes for my protection."

"Flare-up can be caused by emotional stress and other illnesses."


A malar rash, sometimes called a "butterfly rash" that is formed through sun exposure is
a plaque-like rash and it forms a patchy redness and can scar.

True

False
A nurse is educating patient Jen about what she can do to manage fatigue during
exacerbation, the patient needs a further teaching if she states that:

"I can sit whenever it is possible."

"A short period of rest for my joints is good."

"Whenever I don't feel extreme tiredness, I can do little exercise."

"Hot bath before going to sleep will help to reduce fatigue."


The nurse is CORRECT if she question a prescription of this medication to a patient
with SLE.

Corticosteroid

Oral contraceptive

Immunosuppressants

Antineoplastic

The best test to evaluate a client for SLE is:

Fluorescent test for ANA


Serum complement levels

Anti-Ro antibody test

Antiphospholipid antibody test


What is/are the main cause(s) of SLE?
Unknown

What food items to avoid for a patient with SLE?

Broccoli

Garlic

Turkey

Bacon

Potatoes
\Give one goal of treatment for SLE.
Prevent progressive loss \of organ function.

A medication for SLE aimed to decrease the inflammation rapidly and it is for short-term
usage only. This is also susceptible to osteoporosis, DM, and infection.

Azathioprine

Plaquenil

Prednisone

Paracetamol
To diagnose SLE, 4 or more should be met from 14 classification criteria given by ACR.

True

False
LP: CARE of CLIENTS with AUTO
IMMUNE DISORDER
Total points18/20

1. Which of the following is recommended for patients with mild or remittent SLE with
prominent arthralgias?
1/1
A. Nonsteroidal anti-inflammatory drugs

B. Iron supplementation
C. Corticosteroids
D. Heparin

2. Which of the following is the best screening test to evaluate a patient for possible
SLE?
1/1
A. Anti-Ro antibody test
B. Antiphospholipid antibody test
C. Serum complement levels
D. Fluorescent test for ANA

3. What is a lupus flare?


1/1
A Symptoms of a disease cause pain
B Symptoms of a chronic disease or condition are suddenly worse

C Any form of itching, pain, or fever associated with a disease


D None of the above

4. The last word in Systemic Lupus Erythematosus refers to _____.


1/1
A. redness

B. bluishness
C. paleness
D. purpleness

5. A _____ is a butterfly rash characteristic of some people who have SLE.


1/1
A. malar rash

B. alar rash
C. manard rash
D. apoptotic rash

6. Systemic lupus erythematosus (SLE) is a chronic inflammatory connective tissue


disorder that can involve joints, kidneys, skin, mucous membranes, and blood vessel
walls. Which group of people makes up 70 to 90% of patients with lupus?
1/1
A. People of Asian descent
B. Older men
C. Women of child-bearing age

D. Women over age 50

7. Very active lupus (sometimes called severe lupus) is treated immediately with which
of the following?
0/1
A. Heparin or warfarin
B. Antimalarial drugs such as hydroxychloroquine

C. Nonsteroidal anti-inflammatory drugs (NSAIDs)


D. A corticosteroid such as prednisone

Correct answer
D. A corticosteroid such as prednisone
Feedback
A corticosteroid such as prednisone (taken by mouth) or methylprednisolone (given by vein) is the
immediate treatment for very severe lupus.

8. A patient with an acute exacerbation of systemic lupus erythematosus (SLE) is


hospitalized with incapacitating fatigue, acute hand and wrist pain, and proteinuria.
The health care provider prescribes prednisone (Deltasone) 40 mg twice daily. Which
nursing action should be included in the plan of care?
1/1
A. Institute seizure precautions.
B. Reorient to time and place PRN.
C. Monitor intake and output.

D. Place on cardiac monitor.

9. A patient with systemic lupus erythematosus (SLE) who has a facial rash and
alopecia tells the nurse, "I hate the way I look! I never go anyplace except here to the
health clinic." An appropriate nursing diagnosis for the patient is:
1/1
A. Activity intolerance related to fatigue and inactivity.
B. Impaired skin integrity related to itching and skin sloughing.
C. Social isolation related to embarrassment about the effects of SLE.

D. Impaired social interaction related to lack of social skills.

10. A client is suspected of having systemic lupus erythematous. The nurse monitors
the client, knowing that which of the following is one of the initial characteristic sign of
systemic lupus erythematous?
1/1
A. Weight gain
B. Subnormal temperature
C. Elevated red blood cell count
D. Rash on the face across the bridge of the nose

11. The nurse is assigned to care for a client with systemic lupus erythematosus
(SLE). The nurse plans care knowing that this disorder is:
1/1
A. A local rash that occurs as a result of allergy
B. A disease caused by overexposure to sunlight
C. An inflammatory disease of collagen contained in connective tissue

D. A disease caused by the continuous release of histamine in the body


12. he nurse is assigned to care for a client admitted to the hospital with a diagnosis of
systemic lupus erythematosus (SLE). The nurse reviews the health care provider's
prescriptions. Which of the following medications would the nurse expect to be
prescribed?
1/1
A. Antibiotic
B. Antidiarrheal
C. Corticosteroid

D. Opioid analgesic

13. The nurse monitors a patient to have Systemic Lupus Erythematosus. Which of
the following symptoms is characteristic of this diagnosis?
1/1
A. Increased T-cell count
B. Scaley, inflamed rash on shoulders, neck, and face

C. Swelling of the extremities


D. Decreased erythrocyte sedimentation rate (ESR)

14. In teaching a patient with SLE about the disorder, the nurse knows that the
pathophysiology of SLE includes:
1/1
A. Circulating immune complexes formed from IgG autoantibodies reacting with IgG
B. An autoimmune T-cell reaction that results in destruction of the deep dermal skin layer
C. Immunologic dysfunction leading to chronic inflammation in the cartilage and muscles
D. The production of a variety of autoantibodies directed against components of the cell nucleus

15. A female patient's complex symptomatology over the past year has culminated in
a diagnosis of systemic lupus erythematosus (SLE). Which of the patient's following
statements demonstrates the need for further teaching about the disease?
1/1
A. "I'll try my best to stay out of the sun this summer."
B. "I know that I probably have a high chance of getting arthritis."
C. "I'm hoping that surgery will be an option for me in the future."

D. "I understand that I'm going to be vulnerable to getting infections."


16. The pathophysiology of systemic lupus erthematosus (SLE) is characterized by:
1/1
A. Destruction of nucleic acids and other self-proteins by autoantibodies

B. Overproduction of collagen that disrupts the functioning of internal organs


C. Formation of abnormal IgG that attaches to cellular antigens, activating complement
D. Increased activity of T-suppressor cells with B-cell hypoactivity, resulting in an
immunodeficiency

17. A patient with newly diagnosed SLE asks the nurse how the disease will affect her
life. The best response by the nurse is:
1/1
A. You can plan to have a near-normal life since SLE rarely causes death
B. It is difficult to tell because to disease is so variable in its severity and progression

C. Life span is shortened somewhat in people with SLE, but the disease can be controlled with
long-term use of corticosteroids
D. Most people with SLE have alternating periods of remissions and exacerbations with rapid
progression to permanent organ damage

18. During an acute exacerbation, a patient with SLE is treated with corticosteroids.
The nurse would expect the steroids to begin to be tapered when serum laboratory
results indicate:
1/1
A. Increased RBCs
B. Decreased ESR
C. Decreased anti-DNA

D. Increased complement

19. Teaching that the nurse will plan for the patient with SLE includes:
1/1
A. Ways to avoid exposure to sunlight

B. Increasing dietary protein and carbohydrate intake


C. The necessity of genetic counseling before planning a family
D. The use of no pharmacologic pain interventions instead of analgesics
20. Which type of lupus mainly affects the skin?
0/1
A. Drug-induced lupus
B.Systemic lupus erythematosus
C. Peripheral lupus erythematosus
D. Cutaneous lupus erythematosus

Correct answer
B.Systemic lupus erythematosus
CHRONIC
OBSTRUCTIVE
PULMONARY
DISEASE
BY: ANGELICA CRISTALES
CHRONIC OBSTRUCTIVE PULMONARY DISEASE (COPD)
A pulmonary disease that causes a chronic obstruction of
airflow from the lungs
It is irreversible. Airflow limitation is usually progressive
Abnormal inflammatory response of the lung to noxious
particles or gases = narrowing of airways,
hypersecretion of mucus and changes in the pulmonary
vasculature
CHRONIC
EMPHYSEMA
BRONCHITIS impaired gas exchange (oxygen, carbon dioxide)
presence of cough and sputum production for results from destruction of the walls of
at least 3 months in each of 2 consecutive years overdistended alveoli

"BLUE BLOATERS" "PINK PUFFERS"

Big and blue skin - Pink skin and pursed lip


cyanosis breathing
Long term chronic Increased barrel chest
cough and sputum No chronic cough or
Unusual lung sound - minimal cough
wheezing or crackles Cachexia - extreme wt
Edema loss and muscle wasting
RISK FACTORS

01. Smoking and Passive Smoking


Irritates the goblet cells and mucous glands, causing an increased accumulation =
produces more irritation, infection, and damage to the lung

02. Prolonged and intense exposure to


occupational dusts and chemicals

03. Air pollution

04. Genetic - Alpha 1-antitrypsin deficiency


Alpha 1-antitrypsin protects the lungs from damage caused by protease enzyme
such as elastase and trypsin.
Clinical Manifestations
CHRONIC COUGH
SPUTUM PRODUCTION
WHEEZING/CRACKLES
CHEST TIGHTNESS
DYSPNEA
WEIGHT LOSS
RISK FOR RESPIRATORY INFECTIONS
ASSESSMENT AND
DIAGNOSTIC FINDINGS

PATIENT HISTORY -
PULMONARY FUNCTION
STUDIES
SPIROMETRY
ARTERIAL BLOOD GAS
CHEST X-RAY
SCREENING FOR ALPHA1-
ANTIRYPSIN DEFICIENCY
PULMONARY FUNCTION TEST
Medical Management

Smoking cessation
Bronchodilators - relieve
bronchospasm and reduce airway
obstruction
Corticosteroids - improves the
symptoms
Oxygen administration
Surgery: Bullectomy - help reduce
dyspnea and improve lung function
Nursing Management
Patient Education Monitor and manage potential complications
include normal anatomy and physiology of the Monitor cognitive changes. The nurse should monitor for
lung, pathophysiology and changes with COPD, cognitive changes such as personality and behavior
medications and home oxygen therapy, nutrition, changes and memory impairment.
respiratory therapy treatments Monitor pulse oximetry values. Pulse oximetry values are
used to assess the patient’s need for oxygen and
Breathing Exercise administer supplemental oxygen as prescribed.
Pursed lip breathing helps to slow expiration,
Prevent infection. The nurse should encourage the
prevents collapse of small airways, and helps the
patient to be immunized against influenza and S.
patient to control the rate and depth of
pneumonia because the patient is prone to respiratory
respiration.
infection.
Activity Pacing
Manage daily activities. Daily activities must be
paced throughout the day and support devices
can be also used to decrease energy
expenditure.
Systemic Lupus
Erythematosus
affects multiple organs

Systemic Lupus
Erythematosus
affects multiple organs

Systemic Lupus
Erythematosus
reddening of skin
affects multiple organs

Systemic Lupus
Erythematosus
reddening of skin
diseases affecting skin
affects multiple organs

Systemic Lupus
Erythematosus
reddening of skin
Systemic Lupus
Erythematosus
a disease that is systemic, and affects a wide variety of organs,
but notably often causes red lesions on the skin.
TYPES OF
LUPUS
SYSTEMIC LUPUS ERYTHEMATOSUS

DISCOID LUPUS ERYTHEMATOSUS


TYPES OF
LUPUS
DRUG-INDUCED LUPUS

NEONATAL LUPUS
What Causes
SLE?
SLE is an autoimmune
disorder that develops when
the body’s immune system
begins to attack its own
tissues.
SLE is an autoimmune
disorder that develops when
the body’s immune system
begins to attack its own
tissues.

UNKNOWN
but likely a combination of the following factors:

GENETIC

ENVIRONMENTAL

HORMONAL

MEDICATION
PATHOPHYSIOLOGY
SLE IS A RESULT OF DISTURBED
IMMUNE REGULATION THAT
CAUSES AN EXAGGERATED
PRODUCTION OF
AUTOANTIBODIES.
In SLE, the increase in autoantibody production is
thought to result from abnormal suppressor T-cell
function, leading to immune complex deposition
and tissue damage. Inflammation stimulates
antigens, which in turn stimulate additional
antibodies, and the cycle repeats.
CLINICAL
MANIFESTATIONS
GENERAL SYMPTOMS

FEVER JOINT PAIN RASH

WEIGHT LOSS FATIGUE


CARDIAC
Endocarditis
Myocarditis
Pericarditis

GASTROINTESTINAL
Abdominal pain
Nausea & vomiting

DERMATOLOGICAL
Alopecia
Mucous membrane lesion
Photosensitivity
Purpura
Raynaud's phenomenon
DIAGNOSIS
Diagnosis is based on clinical
symptoms & lab findings

MALAR RASH DISCOID RASH PHOTOSENSITIVITY


ULCERS SEROSITIS ARTHRITIS RENAL DISORDERS

NEUROLOGICAL HEMATOLOGIC ANGTINUCLEAR OTHER


DISORDERS DISORDER ANTIBODY AUTOANTIBODY
MEDICAL MANAGEMENT
Lupus is characterized by periods of flare-ups and periods of
remittance, treatment is often aimed at preventing flares or
limiting how severe they are when they do happen.
To help prevent flares, some patients may have to avoid
sunlight exposure with hats and long-sleeved clothes.
To reduce severity of flares, corticosteroids may be used
to help limit the immune response, and finally, if
symptoms are really severe, certain immunosuppressive
drugs might be used.
GOAL OF TREATMENT

Prevent progressive loss of organ function.


Reduce the likelihood of acute disease.
Minimize disease-related disabilities.
Prevent complications from therapy.
THANK YOU :))
caused by disfunction of immune system

-immunological…

-hypersensitivity reactions
Exaggerated or inappropriate state of normal immune response

-autoimmune disorder

Type 1 hypersensitivity reaction

-State of rapidly developing anaphylactic type of antigen w/in 15-30 mins.

Dermatitis medicamentosa

Type 2 cytotoxic

Neural antibodies that attack antibodies

Blood transfusion reactions 15-30 mins.

Typpe3 immune complex hypersensitivity

Reaction may be general other organs may take 3-10 hours after exposure to antigen

immunodefficiency disorders
-infectious disease is compromised or entirely upset

type1-primary

-congenital

type 2-secondary

-acquired

hiv, malnutrition and immunosuppression

skin test

Epidermis
wheal 15 degree angle

30 minutes

Autoimmune disorder
Attacks body by mistake

Type 1 diabetes

-Glucose to produce energy

Ibd

-Chronic inflammation of the gi tract( part. Large intestine)

Psoriasis

-Causes red flaky patches of skin covered with silver scales

Multiple sclerosis

-Can affect brain and spine, problems vision movement sensation and balance, serious disability

Glayn barey syndrome???

Damages th nerves causes myscle weakness and paralysis

-Rheumatois arthritis

-Joint, relase of inflamma and joint bec distorted…..

Sle

ks its own tissues, causing widespread inflammation and tissue damage in the affected organs. It can
affect the joints, skin, brain, lungs, kidneys, and blood vessels.

Chronic disease , period of mild symtoms

Severe fatigue, joint pains with swelling, butterfly rash, pathognomonic sign, hair loss

, anemia, blood clotting probs, finger white/blue, reynauds phenomenon

Genetics, environmental factors (uv rays, meds, viruses, physical/emotional distress and trauma),
sex(affects women more, estrogen MIGHT play a role) and hormones

Diagnosed by: phys exam, signa and symtoms, mucous mebranes, arthritis

Treatment: no cure, goal is to treat symptoms


Antimalarial drusg for skin and joint problems

Vasculaitis (inflamm of blood vessesl)

Kidney inflamm to kidney failure

Pericarditis to heart attack or stroke

Relief of pain and discomfort

Fatigue

Maintenance of skin integrity

Compliance with prescribed meds

Increase regarding disease knowledge

Absence of complications

1. What is the most common coping problem for individuals diagnosed with lupus?
1 point

A) Pain
B) Fear
C) Guilt
D) Depression

Clear selection

2. Mild SLE is often treated with


1 point

A) antimalarials.
B) corticosteroids.
C) cytotoxic drugs.
D) nonsteroidal anti-inflammatory drugs.

Clear selection

3. Which test is the most specific and sensitive test for SLE?
1 point
A) Anti-DNA antibody test
B) Complete blood count (CBC)
C) Antinuclear antibody (ANA) test
D) Erythrocyte sedimentation rate (ESR)

Clear selection

4. Which of the following is a comorbidity that is recognized to increase the risk for
morbidity and mortality in patients with SLE?
1 point

A. Inflammatory bowel disease


B. Osteoporosis
C. Atrial fibrillation
D. Pregnancy

Clear selection

5. Which of the following are among the most common symptoms in new cases or
recurrent active SLE flares?
1 point

A. Fatigue and arthralgia


B. Cranial neuropathy
C. Pleuritis
D. Peptic ulcer disease

Clear selection

6. Which of the following cutaneous manifestations of SLE are included in lupus


diagnostic criteria?
1 point

A. Alopecia
B. Raynaud phenomenon
C. Malar rash
D. Livedo reticularis

Clear selection

7. Management of SLE depends on disease severity and manifestations. However,


which of the following medications is considered the cornerstone of SLE
management?
1 point

A. Nonsteroidal anti-inflammatory drugs (NSAIDs)


B.Hydroxychloroquine
C. Methotrexate
D. Belimumab

Clear selection

8. A 30-year-old woman has been diagnosed with early systemic lupus erythematosus
(SLE) involving her joints. In teaching the patient about the disease, the nurse
includes the information that SLE is a(n):
1 point

A. Hereditary disorder of women but usually does not show clinical symptoms unless a woman
becomes pregnant.
B. Autoimmune disease of women in which antibodies are formed that destroy all nucleated cells
in the body.
C. Disorder of immune function, but it is extremely variable in its course, and there is no way to
predict its progression.
D. Disease that causes production of antibodies that bind with cellular estrogen receptors,
causing an inflammatory response.

Clear selection

9. A patient with an acute exacerbation of systemic lupus erythematosus (SLE) is


hospitalized with incapacitating fatigue, acute hand and wrist pain, and proteinuria.
The health care provider prescribes prednisone (Deltasone) 40 mg twice daily. Which
nursing action should be included in the plan of care?
1 point

A. Institute seizure precautions.


B. Reorient to time and place PRN.
C. Monitor intake and output.
D. Place on cardiac monitor.

Clear selection

10. Following instruction for a patient with newly diagnosed systemic lupus
erythematosus (SLE), the nurse determines that teaching about the disease has been
effective when the patient says:
1 point

A. "I should expect to have a low fever all the time with this disease."
B. "I need to restrict my exposure to sunlight to prevent an acute onset of symptoms."
C. "I should try to ignore my symptoms as much as possible and have a positive outlook."
D. "I can expect a temporary improvement in my symptoms if I become pregnant."

Clear selection
11. A client is suspected of having systemic lupus erythematous. The nurse monitors
the client, knowing that which of the following is one of the initial characteristic sign of
systemic lupus erythematosus.
1 point

A. Weight gain
B. Subnormal temperature
C. Elevated red blood cell count
D. Rash on the face across the bridge of the nose

Clear selection

12. The nurse is assigned to care for a client with systemic lupus erythematosus
(SLE). The nurse plans care knowing that this disorder is:
1 point

A. A local rash that occurs as a result of allergy


B. A disease caused by overexposure to sunlight
C. An inflammatory disease of collagen contained in connective tissue
D. A disease caused by the continuous release of histamine in the body

Clear selection

13. The nurse is assigned to care for a client admitted to the hospital with a diagnosis
of systemic lupus erythematosus (SLE). The nurse reviews the health care provider's
prescriptions. Which of the following medications would the nurse expect to be
prescribed?
1 point

A. Antibiotic
B. Antidiarrheal
C. Corticosteroid
D. Opioid analgesic

Clear selection

14. The pathophysiology of systemic lupus erthematosus (SLE) is characterized by:


1 point

A. Destruction of nucleic acids and other self-proteins by autoantibodies


B. Overproduction of collagen that disrupts the functioning of internal organs
C. Formation of abnormal IgG that attaches to cellular antigens, activating complement
D. Increased activity of T-suppressor cells with B-cell hypoactivity, resulting in an
immunodeficiency

Clear selection
15. Teaching that the nurse will plan for the patient with SLE includes:
1 point

A. Ways to avoid exposure to sunlight


B. Increasing dietary protein and carbohydrate intake
C. The necessity of genetic counseling before planning a family

1. Which of the following is a non-organ-specific (systemic) autoimmune disease:

1/1

A Myasthenia gravis.

B Systemic lupus erythematosus (SLE).

C Pernicious anemia.

D Insulin-dependent diabetes mellitus.

2. Which of these is an autoimmune disease?

1/1

A. Type 1 diabetes

B. Rheumatoid arthritis

C. Psoriasis

D. AIDS

E. A, B, and C

3. If you have an autoimmune disease, what happens with the immune system?

1/1
A. Your immune cells die

B. Antibodies from your immune system mistakenly attack tissues in the body

C. Your immune system makes too many immune cells

D. None of the above

4. Autoimmune diseases strike which group more often?

1/1

A. Children younger than 12

B. Men older than 40

C. Women of childbearing age

D. Women older than 50

5. What tissues, organs, or body systems can be affected by autoimmune diseases?

1/1

A. Skin

B. Joints

C. Thyroid

D. All of the above

6. How does a person develop an autoimmune disease?

1/1

A. It may be triggered by a virus, such as mumps


B. It may be a complication of an existing infection, such as strep throat

C. It may be caused by exposure to an environmental agent

D. Most do not have an obvious cause

E. All of the above

7. Why are some autoimmune diseases difficult to diagnose?

1/1

A. Symptoms may be vague

B. No specific lab tests exist to confirm a diagnosis

C. Symptoms may come and go, making it hard to pinpoint the problem

D. All of the above

8. Which of these autoimmune diseases can be cured?

1/1

A. Lupus

B. Multiple sclerosis

C. Scleroderma

D. None of the above

9. The transfusion of mismatched blood groups leads to transfusion reaction. This hypersensitivity
reaction is of following type:

1/1

A. Type-III hypersensitivity
B. Type-IV hypersensitivity

C. Type-II hypersensitivity

D. Type-I hypersensitivity

10. Which of the following is not an allergic reaction?

1/1

A. Rhesus antigen incompatibility

B. Atopic dermatitis

C. Allergic Rhinitis

D. Food allergy

11. Malar rash is a characteristic symptom of:

1/1

A. SLE

B. Anaphylaxis

C. Guilliane Barre Syndrome

D. Multiple Sclerosis

12. The inappropriate response of immune system towards a relatively harmless antigen causing harm
to the host is referred as

1/1

A. hypersensitivity
B. auto-immune diseases

C. immunodeficiency

D. tolerance

13. All of the following are autoimmune disorders except;

1/1

A. Graves disease

B. Herpes Zoster

C. Rheumatoid arthritis

D. Addison’s disease

14. The most rapid effects of exposure to an allergen may be due to which of the following?

1/1

A. leukotrienes.

B. granulocytic infiltration.

C. metabolites of arachadonic acid.

D. histamine.

15. Which of the following are among the most common symptoms in new cases or recurrent active SLE
flares?

1/1

A. Fatigue and arthralgia

B Cranial neuropathy
C.Pleuritis

D. Peptic ulcer disease

What are the 4 types of hypersensitivity reactions


1. Type I (Anaphylactic Reactions)
2. Type 2 (Cytotoxic Reactions)
3. Type 3 (Immune Complex Reactions)
4. Type 4 (Cell-Mediated or Delayed-type Reactions)
Type I ( Anaphylatic) Hypersensitivity
a. Time before chemical signs
b. Characteristics
c. Examples
a. Less then 30 minutes immediate
b. IgE binds to mast cells; this causes degranulation of the mast cell and release of
histamine (reactive)
c. examples
*Anaphylactic shock from drug injections & insect venom
*allergic conditions (hay fever, asthma)
Type 2 (Cytotoxic) Hypersensitivity
a. Time before chemical signs
b. Characteristics
c. Examplebs
a. 5-12 hours
b. Antigen causes formation of IgM & IgG antibodies that bind to the target cell; when
combined with action of complement destroys target cell
c. examples
* Transfusion reaction (IgM-agglutination)
*Rh incompatibility (IgG cross placenta)
Type III ( Immune Complex) Hypersensitivity
a. Time before chemical signs
b. Characteristics
c. Examples
a. 3-8 hrs
b. Antibodies & antigens form complexes that cause damaging inflammation
c. Examples
*Arthus reaction
* Serum sickness
Type IV (Cell-mediated or delayed type) Hypersensitivity
a. Time before chemical signs
b. Characteristics
c. Examples
a.24-48 hrs
b. Antigens activate Tc that kill target cells
c. Examples
*Rejection of transplanted tissues contact DERMATITIS (Poision Ivy)
* Chronic diseases (Tuberculosis)
Everything above is a summary of Hypersensative Reactions
Everything below is the lecture from DR. Harris on type 1 & 4 Hypersensitivity
What are the effector molecules & cells involved in hypersensitivty, immediate or
allergic reactions?
1. Effector molecule-IgE
2. Cells-Mast cells or basophils which do the same things
What are high IgE levels a indication of and what % of the population have the tendency
of producing high IgE?
1. Parasitic infecions
2. 20% of the population
Condition in which people of Type I Hypersensitivity are suffering from? And what are
allergens>
1. Atopy
2. Individuals said to be allergic & their antigens
Where are basophils found?
Circulating blood granulocytes
Where are mast cells found & what do they secrete?
1. They are located throughout connective tissues near blood & lymphatic vessles
2. Granulocytes & cytokines
What are the similarities of mast cells & basophils
They contain large number of granules who contain pharmacological active mediators
What leads to Degranulation in Hypersensative Reactions? & What are its steps
1. IgE crosslinking due to the allergen
*The crosslinkage of IgE is required for the degranulation
What are other stimuli other than IgE that can degranulate the mast cell
1. anti-IgE antibody
2. Chemicals/Drugs
3. Anaphylatoxins (C3a, C4a, C5a)
After IgE is crosslinked and it degranulates the mast cell what happens
1. Formation of Ca2+ channels
2. Ca2+ leads to the formation of arachidonic acid& formation & contraction of
microfilaments
What is the significance of the formation of Arachidonic acid
1. It is a precursor to prostaglandins & leukotrienes
*both vasoactive substances (change blood vessel diameter)
What is the significance in the inductuon of microtubule formation
It makes the granlules more permeable so it can move to the plasma membrane.
What are the principal primary mediators involved in type I hypersensitivity
*Mediators activated by mast cells
1. Histamine, Heparin
2. Serotonin
3. Eosinophil Chemotactic factor
4. Neutrophil Chemotactic Factor
5. Proteases-involved in mucus secretion & complement
Histamine, Heparin, & Serotonin all share a common feature which is;
1. Increase vascular permeability & smooth muscle contraction
What are the secondary principal mediators in Type 1 hypersensitivity
1. Platelet-activity factor
2. Leukotrines
3. Prostaglandins
4. Bradykinin
5. Cytokines
Function of Platelet-activating factor
1. Platelet aggregation & degranulation; contraction of pulmonary smooth muscle
Leukotrienes
*have the same function of histamine, heparin & serotonin
1. Increased vascular permeability & contraction of pulmonary smooth muscles
Prostaglandins
Vasodilation; contraction of pulmonary smooth muscle & platelet aggregation
Bradykinin
Increased vascular permeability; smooth muscle contraction
Cytokines
a. IL-1 & TNF (alpha)-Systemic anaphylaxis (increased expression of CAMs on venular
endothelial cells)
b. IL-4 & IL-13-Increased IgE production
What are the two types of Type I-Hypersensitive Reactions?
1. Systemic
2. Local
What is systemic Anaphylaxis
1. It occurs when an allergen is introduced directly into the blood stream or absorbed
from skin
a. vasodilation & smooth muscle contraction induced by mediatores released by mast
cells is the outcome
What are the allergens inducing systemic anaphylaxis & how is it treated?
1. Allergens include in humans
a. venom from bee, wasp, hornet and ant
b. Drugs (penicillin, insulin, & antitoxins)
c. Sea Food & Nuts
2. Treated with epinephrine
How does epinephrine regulate systemic Anaphylaxis
Counteracts effects of histamine, leukotrienes by relaxing smooth muscles & reducing
vascular permeability
Whats Localized Anaphylaxis (Atopy)?
Anaphylaxisife threatning allergic reaction
1. A allergic reaction limited to a specific tissue
2. It involves epithelial surfaces at the site of allergen entry
3. It can be inherited
What are the 4 types of Localized Anaphylaxis
1. Allergic Rhinitis (hayfever)
2. Atopic dermatitis (Exema)
3. Food Allergies
4. Asthma
Allergic Rhinitis (Hay Fever)
1. It is the inhalation of airborne allergen
2. Causes liquid from eyes, nose, sneezing & coughing
Atopic Dermatitis (Allergic Eczema
1. Inflammation of the skin
2. Skin eruptions that are red with pus are formed; (contain Th2 cells & increased # of
eosinophils)
Food Allergies
1. Mast cell activation occurs along the upper & lower GI tract
*This leads to granulation along the gut; increase permeability of mucus membranes so
that allergens enters blood
What occurs in Asthama
1. The passage of airborne or blood antigens into the lower respiratory track
2. Bronchoconstriction occurs due to smooth muscle contraction, airway edema, mucus
secretion & inflammation
What is the diffrence between the two asthma responses (early, late)
1. Late (Hours), Early (Minutes)
2. Late involves additional mediators
a. Cytokines (IL-4, IL-6, TNFalpha, etc..
(Early involves histamine (vasodilation)
3. Late (Migration & Activation of leukocyte)
Early (Bronchostriction, Mucus Secretion)
Effects of early Asthma responses
1. Bronchoconstriction, Vasodilation & Mucus build-up
Effects of Late Asthma responses
1. Neutrophils & Eosinophils cause tissue injury by releasing toxic enzymes (oxygen
radicals & cytokines)
2. This forms a mucus plug that attaches to the bronchial wall.
Late Phase Type I Hypersensativity Reactions
*Eosinophils play a major role in late phage responses
1. 4-6 hours after initial reactions, a localized inflammation develops in response to
mediators
2. Infiltration of myeloid linage cells
3. TNF IL-1 increase adhesion molecule expression on endothelial cells
What are the drugs that are used to treat type 1 hypersensitivity
1. Antihistamines
2. Cromolyn Sodium
3. Theophylline
4. Epinephrine (Adrenaline(
5. Corisone
Action of Antihistamines
1. Block H1 & H2 receptors
Cromolyn Sodium
1. Block Ca2+ influx into mast cells
Thepphylline
1. Prolongs high cAMP levels in mast cells by inhibiting phosphodiesterase.
Epinephrine (Adrenaline)
1. Stimulates cAMP production by binding to Beta Adrenergic receptors on mast cells
*High cAMP prevents degranualation
Cortisone
1. Reduce histamine levels
2. Stimulate mast-cell production of cAMP
Wheal & Flare Reaction
*Skin test for an allergy
1. Allergy is placed under skin
2. And sensitized mast cells are activated causing fluid build-up & swelling
3. The size of the reaction is an indicator of the sensitivity of the individual to the
allergen
Step 1 of Delayed-Type Hypersensitivity
1. Sensitization Phase
a. encounter with the antigen by APC
b. Th1 cells activated
Step 2 of Delayed-Type Hypersensitivity
*cytokines induce the immune response
2. Effector Phase
a. This is the second encounter with the antigen
b. Secretion of cytokines; recruitment & activation of macro-phages
What is the primary effector cell of type 4 hypersensitivity
*type 1; mast cells & basophils
1. Marcophages
What is purpose of having a delayed type hypersensitivity
1. It reflects the time needed for cytokines & chemokines to recruit & activate
macrophages
a. It forms an amplified response by cytokines of macrophages
What is the significance of having a activated macrophages
*The clearing of a pathogen
1. Activated macrophage have increased;
a. phagocytosis
b. lytic enzymes (Nitric Oxide, O radicals)
c. MHC II & adhesion molecules
2. Better APC cell, phagocyte & killer cell
What are the some of the Antigen Presenting Cells of Hypersensiivity
1. Marcropage
2.Dendritic Cells
3. B-Cells
4. Langerhans Cell (In Skin)
What is a DTH response?
1. If the antigen is not cleared completely
a. prolonged response occurs & becomes destructive to the host
What occurs in Granuloma formation
1. Fusion of macrophages forming giant cells. (Why does it cause tissue necrosis)
a. giant cells release high amounts of lytic enzymes which destroy surrounding tissue
What are the intracellular bacteria that induce delayed-type (type IV) hypersensitivty
* 2 mycobacterium, listerine & a abortion
1. Mycobacterium Tuberculosis
2. Mycobacterium Leprae
3. Listeria Monocytogenes
4. Brucella Abortus
What are the intracellular fungi that induce delayed-type (type IV) hypersensitivty
1. Pneumocytis carinii
2. Candida albicans
3. Histoplasma Capsulatum
4. Cryptococcus Neoformans
What are the intracellular parasites that induce delayed-type (type IV) hypersensitivty
1. Leishmania sp.
What are the viruses that induce delayed-type (type IV) hypersensitivty
1. Herpes Simples
2. Variola (small pox)
3. Measles Virus
What are the contact antigens that induce delayed-type (type IV) hypersensitivty
1. Picrychloride
2. Hair dyes (Mentioned)
3. Nickel Salts (Mentioned)
4. Posion Ivy
5. Poison Oak (Mentioned)
What are the role of Cytokines in Delayed Type-Hypersensitivity
1. IL-3; Induce hematopoiesis
2. IL-1, TNF(alpha)
*Upregulate adhesion molecule expression on endothelial cells to facilitate trafficking &
extravasation of monocytes into tissue site
What are the roles of interferons TNF & chemokines in Type 4 hypersensitivty
1. Activate macrophages (INF& TNF)
2. Attract monocytes to the site & hold them there
How is a Type 4 Hypersensitivity Skin Test done? What are the signs of a positive test
& what implications do they have?
1. Antigen is injected int skin & individual is observed for a reaction
2. Positive skin test-indicates that the person has sensitived T-cells
Examples of DTH Skin test & what do they test for?
1. PPD Test; Mycobacterium Tuberculosis
*Protein from bacteria cell wall is injected
*Presence of a red, firm lesion between 2-3 days after injection is an indicator of
exposure.

give an example of a type I reaction


(IgE mediated-->mast cell)
allergy (rhinitis, hay fever), anaphylaxis (bee sting, some food/drug)
presentation of type I disorder
(IgE mediated-->mast cell)
immediate, anaphylactic, atopic (genetically mediated predisposition to an excessive
IgE reaction)
example of type II reaction
(Ab binds foreign antigen- activates compliment)
hemolytic anemia
pernicious anemia
rheumatic fever
goodpasture's syndrome
Graves' disease
Myasthenia gravis
example of Type III reaction
(deposit immune complex in membrane--recruit complement). The reaction is a
complication of an underlying autoimmune disease (not the primary cause).
autoantibody is already being made--> when it builds up to a high enough level and is
deposited as IC in tissues-->leads to type III
autoimmune diseases that can lead to type III include:
-SLE
-Rheumatoid arthritis
-chronic hepatitis
-alcoholic cirrhosis
-sjorgren's syndrome

-serum sickness (a type III reaction that results from the injection of heterologous or
foreign protein or serum-- ie horse)
arthus reaction (localized swelling and inflammation following tetanus vaccine- The end
result is a localized area of redness and induration that typically lasts a day or so.)
example of type IV reaction
T cell mediated (no Ab) Delayed Type
T binds antigen-->activates macrophage
* do not appear until 24 hours after antigen contact

Granulomatous hypersensitivity (Th1 recruiots macrophages--giant cells)


Type I Diabetes Mellitus
Multiple sclerosis
Guillain-Barre
Hashimoto's throiditis
PPD test for tuberculosis
Graft vs host disease
contact dermatitis (eg poison ivy, nickel, chemical agents, haptens)
presentation of Type II reactions
disease tends to be specific to tissue or site where antigen found
presentation of Type III reactions
can be associated with vasculitis and systemic manifestations
what is a type I hypersensitivity reaction?
soluble IgE mediated cross linking the Fc-epsilonRI receptor on mast cells and
basophils. Mast cells degranulate and release histamine and leukotrienes. Most
common form of hypersensitivity.
what is type II hypersensitivity reaction?
*this usually causes an autoimmune disease
antibody-mediated diseases - is where antibodies IgG and IgM against cells or
extracellular matrix components may react with any tissue that expresses the specific
antigen, but are usually specific for a particular tissue; antibodies are most often self-
reactive and may cause tissue injury by inducing local inflammation; examples include
autoimmune hemolytic anemia, graves disease and myasthenia gravis.

IMPORTANT: the Ab specificity is related to the disease presentation unlike Type III. Ie
in goodpasture or Graves or myasthenia gravis. The diseases only happen in specific
organs/vs specific receptors. IC not deposited just anywhere. It determines how the
disease presents.
What is type III hypersensitivity reaction?
* this is usually a complication of an underlying primary autoimmune disease

immune complex deposition (ag-Ab) activate complement through classical pathway--


>attract neutrophils-->release lysosomal enzymes-->tissue desctruction.

serum sickness happens when IC deposited in membranesb


ex: serum sickness, RA, SLE.

Important: the Ab specificity is NOT related to the disease presentation. IC is deposited


wherever Ag is encountered. (serum sickness... not like type II-- goodpasture only
happens against Ab on glomerular membrane)
what is the major difference between type II and Type III hypersensitivity reactions?
Ab specificity determines clinical presentation in type II (graves, myasthenia gravis,
goodpasture) but in type III it doesn't matter. IC is deposited anywhere it encounters
antigen. In type II these diseases always present a certain way bc the Ag is only against
Ab in those specific places (glomerular membrane)/against those receptors (TSH, ACH)
atopy
hereditary tendency to develop allergies. Atopic individuals susceptible to a wide range
of allergegns
asthma
severe allergic reaction caused by constriction of bronchioles
anaphylaxis
acute type I reaction caused by histamine and other vasoactive substanes usually in
association with IgE. May be local, cutaneous, or systemic.
which cytokines does a TH2 cell need to secrete for a B cell to class switch and secrete
IgE?
IL4 and IL13
what part of the IgE allows it to bind to the mast cell?
Its 4th constant region
what substances are released from mast cells?
histamine, protease (degrade blood vessel basement membrane), Heparin (anticoag),
ECF-A (eosinophil chemotaxis), Neutrophil chemotactic factor
deficiency of which antibody is common in people with type I hypersensitivity?
IgA
a lack of which type of T cell is common in people with type I hypersensitivity?
T regulatory cells
what triggers mast cell (and basophil) degranulation?
crosslinking of at least 2 high affinity IgE Fc receptors by allergen.
or mast cells can be activated by C3a and C5a, drugs, cold, exercise, and certain
neuropeptides.
what type of receptor is the Fc receptor on mast cells?
protein tyrosine kinase -->activation leads to 2nd messenger production
levels of which intracellular messenger must decrease below baseline in order for mast
cell degranulation to occur?
cAMP must decrease. A continued increase in cAMP will slow or halt degranulation.
clinical tests for allergies
scratch test (skin wheal and flare)
and radioallergosorbent Test (RAST) an (ELISA) for patients IgE specific antibodies
what enzyme produces cAMP and what enzyme degrades it?
adenyl Cyclase produces cAMP
Cyclic nucleotide phosphodiesterase breaks it down to AMP
what type of enzyme inhibitor would you give to someone in the ER with anaphylaxis?
Theophylline and other AMP specific phosphodiesterase inhibitors to keep cAMP levels
in mast cells high (stop degranulation)
why do we use Beta adrenergic inhalers (like Albuterol) for type I hypersensitivity?
B2 adrenergic receptor activation stimulates adenyl cyclase (which converts ATP to
cAMP). High cAMP blocks degranulation.
Also leads to bronchodilation.
alpha adrenergic leads to vasoconstriction of leaky arterioles.
what is allergy desensitization?
- inject tiny amounts of allergen subcutaneously or intradermally. The exact mechanism
is not fully understood but it is accepted that immunotherapy causes modification of the
immune system. try to induce allergen-specific IgG to neutralize the allergen in place of
allergen-specific IgE.-->convert Th2 to T reg cells. Over time replace and inhibit IgE
response.
why does a type IV hypersensitivity not need Antibody involvement?
Langerhans cells can take up and process antigen and bring it directly to TCD4+ rich
areas. afterTh1 cells are sensitized to antigen in lymp node they can release cytokines
and recruit macrophages without activating B cells

(if there is no free antigen-- only chemicals that must be bound to presenting cells and
no B cells are activated, no Ab will be produced)
what is the pathogenesis of Granuloma formation?
macrophage phagocytoses some foreign invader (ie bacteria, virus, fungus). but the
organism doesn't die inside the phagocyte. It is resistant to the macrophages. More are
recruited along with Cytotoxic T cells and TH1 cells. Fuse to form multinucleate giant
cells. Over time fibroblasts and calcification wall off the granuloma =.

Type III IC's can progress to this type IV hypersensitivity rxn.


risk factors for developing allergies
Inheriting MHC II molecules from parents who were allergic
Th2 cells that produce low IL-10
lack of Treg cells
IgA deficiency

What are the Four Type of Hypersensitivity Reactions


Type I (IgE Mediated)
Type II (cytotoxic) incompatible blood type
Type III (Immune Complex Mediated) serum sickness
Type IV (cell-mediated, not antibody)
Type I, II and III are known as
Antigen-Antibody Responses (immediate Hypersensitivity)
Type IV are known as
Antigen-Lymphocyte Reactions (delayed hypersensitivity)
What are the two most common types of Hypersensitivity reactions?
Types I and IV
Type I hypersensitivity reactions produces what?
IgE antibodies that bind to membrane receptors located on mast cells in connective
tissues and basophils
In subsequent exposures in Type I are
allergic responses that are immediate
Why makes these subsequent exposures in Type I immediate?
release of histamine and other powerful mediators
What are examples of a Type I response?
Allergic asthma, Allergic rhinitis, Anaphylactic shock
Type I anaphylactic shock is what
life threatening due to laryngospasms and hemolysis
Type II Hypersensitivity Responses what type of response
Cytotoxic
Type II Cyotoxic hypersensitivity occurs when
antibody-antigen complex destroys target cells
Examples of a Type II response is
Blood transfusion to an incompatible blood type
Transplant tissue
Autoimmune Diseases
Drug Reaction
Type III Hypersensitivity Responses involve
IgG or IgM antibody-antigen in circulation
Type III: IgG and IgM complexes do what?
deposit in the walls of small blood vessels, kidney and joints
Type III: What happens after the complexes deposit?
complements are activated and chemical mediators of inflammation are released
Type III: What do the neutrophils do?
they are attracted to the site and attempt to phagocytize the complexes
Type III: What do the lysosomal enzymes do?
when they are released they increase tissue damage
Type III: What are examples of this response?
Systemic Lupus
Rheumatoid Arthritis
Type IV Hypersensitivity Response is what type of a response?
cell-mediated
Type IV: What type of cells does this response involve?
T-cells
Type IV: What type of a reactive does this response have?
delayed hypersensitivity (24-48 hrs)
Type IV: is a result from what?
exaggerated interaction between an antigen and normal cell-mediated mechanisms
Type IV: What happens with this response?
Inflammatory and immune mediators are released, and Killer T-cells cause local tissue
destruction
Type IV: What are examples of this type of reaction?
Contact dermatitis (latex allergy, poison ivy)
Graft rejection episodes

Type 1: Rapid Hypersensitivity, IgE mediated


characterized by production of igE after exposure to an antigen
Type 1: Rapid Hypersensitivity, IgE mediated
1st response is minor, re exposure produces severe allergic reaction
Type 1: Rapid Hypersensitivity, IgE mediated
caused by a release of histamine
Type 1: Rapid Hypersensitivity, IgE mediated
symptoms last about 10 minutes
Type 1: Rapid Hypersensitivity, IgE mediated
EX- anaphylaxis & allergic reaction
Type 1: Rapid Hypersensitivity, IgE mediated
clinical manifestations: urticia, angioedema, allergic rhinitis, nasal discharge
Type II: cytotoxic
antibody reacts with antigen on surface cells
Type II: cytotoxic
types:

Hemolytic Anemia - antibodies produced against RBC

Transfusion reactions- antibodies produced against donor blood cells


*flank pain is most common sign

Erythroblastosis Fettles- maternal antibodies produced against fetal blood cells


Type III: immune complex reactions
antigens cause immune complexes to form in the blood
Type III: immune complex reactions
not organ specific! activates complement & mast cell degeneration which causes tissue
and capillary damage
Type III: immune complex reactions
EX- systemic Lupus erythematous, rheumatoid arthritis
Type III: immune complex reactions
treatment: anti inflammatory agents, glucocorticoids
Type IV: Cell Mediated
reactions do not involve antibodies; mediated by sensitized T lymphocytes
Type IV: Cell Mediated
delayed response (hrs/days)
Type IV: Cell Mediated
EX- poison ivy, transplant rejection, insect stings

Type I definition + example


Type I reactions (ie, immediate hypersensitivity reactions) involve immunoglobulin E
(IgE)-mediated release of histamine and other mediators from mast cells and basophils.
[2] Examples include anaphylaxis and allergic rhinoconjunctivitis.

Type II definition + example


Type II reactions (ie, cytotoxic hypersensitivity reactions) involve immunoglobulin G or
immunoglobulin M antibodies bound to cell surface antigens, with subsequent
complement fixation. An example is blood transfusion reaction.

Type III definition + example


Type III reactions (ie, immune-complex reactions) involve circulating antigen-antibody
immune complexes that deposit in postcapillary venules, with subsequent complement
fixation. An example is seen in SLE.
Type IV definition + example
Type IV reactions (ie, delayed hypersensitivity reactions, cell-mediated immunity) are
mediated by T cells rather than by antibodies. An example is contact dermatitis from
poison ivy or nickel allergy.

Systemic Lupus Erythematosus patho


Systemic lupus erythematosus (SLE) is a prototypic autoimmune disease characterised
by the production of antibodies to components of the cell nucleus in association with a
diverse array of clinical manifestations. The primary pathological findings in patients
with SLE are those of inflammation, vasculitis, immune complex deposition, and
vasculopathy.

Clinical manifestations of SLE


Dermatologic- butterfly rash
Musculoskeletal- arthritis and joint pain. Swan neck in fingers caused by joint swelling
Cardiopulmonary- hypertension, hyperlipidemia, raynauds syndrome
Renal- microscopic hematuria, proteinuria, elevated creatinine, glomerulonephritis
CNS- headaches, stroke (HTN, clotting disorders), fever
Hematologic- clotting issues, anemic
GI- pancreatitis, diarrhea, pain
General- sun will exacerbate the symptoms as well as pregnancy and stress.
DX of SLE
Malar rash
Discoid rash
Photosensitivity
Ulcers: mouth/nose
Arthritis in two or more joints
Serositis
Kidney dysfunction
Neurologic problems
Positive ANA
Immunologic changes
Anti-Smith Ab
Anti double stranded DNA titer
EIA: LE immunoassay
Anemia, leukopenia, thrombocytopenia

Tx goals of SLE
Decrease inflammation
Decrease immune system response
Decrease complications
Reduce pain
Prevent flares
Tx of SLE
Meds:
NSAIDS
Steroids
Plaquenil
-Prevent
-Educate
Scleroderma S&S
Calcinosis
Raynauds
Esophageal hypomotility
Sclerodactyly
Telangiectasia

Scleroderma Tx
D-penicillamine
Symptomatic
PT/OT
Stretching
No smoking: will make Raynaud's worse.
DO NOT DO FINGER STICKS. Protect the fingers.

Systemic Sclerosis (Scleroderma) Overview


Disorder of connective tissue becoming more and more fibrotic.
Cardinal sign: THICKENING AND TIGHTENING OF SKIN
Patho not fully understood. Genetics may have connection.
Excess production of collagen is major issue.
A patient with a history of asthma develops shortness of breath and stridor and
becomes hypotensive during allergy skin testing. Which medication would the nurse
expect to administer in this situation?
1
Zileuton
2
Epinephrine
3
Fexofenadine
4
Cromolyn sodium
2
Epinephrine

The patient is experiencing an anaphylactic reaction, and epinephrine is a first-line


sympathomimetic used to treat anaphylaxis. Fexofenadine is a nonsedating
antihistamine and is not a first-line drug to treat anaphylaxis. Cromolyn sodium is a mast
cell-stabilizing drug; it is used to prevent symptoms of allergic rhinitis but is not useful
during an acute episode. Zileuton is a leukotriene antagonist; it is also used to prevent
symptoms of allergic rhinitis but is likewise not useful during an acute episode.
Which instruction would the nurse give to a patient who has had an allergic reaction to
environmental airborne allergens?
1
Be cautious when eating unprocessed honey.
2
Thoroughly clean cloth drapes, carpeting, and upholstered furniture.
3
Intermittent exposure to known allergens will produce immunity.
4
Wash fruits and vegetables with mild soap and water before eating.
2
Thoroughly clean cloth drapes, carpeting, and upholstered furniture.

Thoroughly cleaning (or removing) cloth drapes, carpet, and upholstery will reduce
airborne pollen, dust mites, and mold. This may help prevent further allergic response.
Washing fruits and vegetables pertains to food allergies. Patients do not develop
immunity to known allergens by direct exposure; common interventions include
avoidance therapy, desensitization therapy, and symptomatic therapy. Honey is only
said to help some people with allergies to pollen; it does not have an impact on airborne
allergens.
A patient is experiencing reduced chest movement and impaired airflow, hypotension,
and a rapid, weak, irregular pulse. What should be the nurse's first priority?
1
Assessing gas exchange
2
Elevating the bed about 10 degrees
3
Starting an IV and running normal saline
4
Ensuring that intubation equipment is ready
1
Assessing gas exchange

Assessing gas exchange should be the nurse's first priority since emergency respiratory
management is critical during an anaphylactic reaction. Starting an IV and running
normal saline, elevating the bed above 10 degrees if hypotension is present, and
ensuring that intubation equipment is ready are all important components of emergency
care of the patient with anaphylaxis, but these can take place once the airway has been
assessed.
The nurse plans to assess a patient with type I hypersensitivity for which clinical
manifestation?
1
Poison ivy
2
Allergic asthma
3
Rheumatoid arthritis
4
Autoimmune hemolytic anemia
2
Allergic asthma

Allergic asthma is a manifestation of type I hypersensitivity. Poison ivy is a type IV


delayed mechanism of hypersensitivity. Autoimmune hemolytic anemia is a type II
cytotoxic mechanism of hypersensitivity. Rheumatoid arthritis is a type III immune
complex-mediated mechanism of hypersensitivity.
A patient has developed serum sickness. What is the most likely cause?
1
Antibiotic therapy
2
Antipyretic drugs
3
Antihistamine therapy
4
Corticosteroid therapy
1
Antibiotic therapy

Serum sickness is a group of symptoms that occurs after receiving serum or certain
drugs. Serum sickness is often caused by penicillin or other antibiotics. Antipyretic
drugs may be useful in treating the fever that accompanies serum sickness.
Antihistamines and corticosteroids are also useful to treat symptoms.
A patient receiving antibiotic therapy for 2 months reports high fever, arthralgia, and
rashes. Diagnostic reports of the patient indicate polyarthritis and enlarged lymph
nodes. What is the best explanation for these clinical manifestations?
1
Secretion of vasoactive amines
2
Accumulation of macrophages in the joints
3
Deposition of immune complexes in blood vessel walls
4
Production of immunoglobulin G (IgG) antibodies against self blood cells
3
Deposition of immune complexes in blood vessel walls

The prolonged use of antibiotics may cause the deposition of immune complexes (drug-
antibody complex) in the blood vessel walls. This deposition results in fever, arthralgia,
polyarthritis, etc. Secretion of vasoactive amines does not cause some of the patient's
symptoms. Macrophages do not accumulate in joints. Production of IgG antibodies to
destroy self cells does not cause these symptoms.
The nurse is caring for a patient experiencing an anaphylactic reaction to an
intravenous (IV) drug. Which nursing intervention is priority for this patient?
1
Apply oxygen using a high-flow non-rebreather mask.
2
Prepare to administer epinephrine 1:10,000 concentration.
3
Keep the head of the bed flat if the patient experiences hypotension.
4
Discontinue the IV immediately and start a new IV with normal saline.
1
Apply oxygen using a high-flow non-rebreather mask.

The nurse should apply oxygen using a high flow, non-rebreather mask at 90% to 100%
before arterial blood gas results are obtained following the guideline of Airway,
Breathing and Circulation (ABC) management for an emergency event. The IV drug
should be promptly discontinued and normal saline administered through the same IV
after the tubing is changed. The nurse should prepare to administer epinephrine 1
mg/mL, which is a 1:1000 concentration, not a 1:10,000 concentration, 0.3 to 0.5 mL
intramuscularly; epinephrine constricts blood vessels, improves cardiac contraction, and
dilates bronchioles. The head of the bed is kept at 10 degrees if the patient experiences
hypotension. Test-Taking Tip: Become familiar with reading questions on a computer
screen. Familiarity reduces anxiety and decreases errors.
A patient has developed a type II hypersensitivity reaction. Which condition is
associated with this type of hypersensitivity?
1
Hay fever
2
Sarcoidosis
3
Serum sickness
4
Autoimmune hemolytic anemia
4
Autoimmune hemolytic anemia

Hemolytic anemia is a type II hypersensitivity. In this reaction, the body makes special
autoantibodies to kill the self cells (or blood cells), causing anemia due to loss of blood
cells. Hay fever, serum sickness, and sarcoidosis are type I, type III, and type IV
hypersensitivities respectively.
Which diseases can be categorized as autoimmune diseases? Select all that apply.
1
Pyelonephritis
2
Cholecystitis
3
Scleroderma
4
Rheumatic fever
5
Goodpasture's syndrome
3
Scleroderma
4
Rheumatic fever
5
Goodpasture's syndrome

Autoimmunity is a process in which an individual becomes oversensitive and starts


producing antibodies against self cells or proteins. Scleroderma, rheumatic fever, and
Goodpasture's syndrome are autoimmune diseases that occur when immune cells start
destroying self cells. Pyelonephritis is an infection of the kidneys and does not have an
autoimmune cause. Cholecystitis is inflammation of the gallbladder; it is not caused by
autoimmunity.
A patient has developed a type II hypersensitivity. Which mechanism is involved in this
type of allergic reaction?
1
Activation of T-cells
2
Phagocytosis of self cells
3
Stimulation of thyroid cells
4
Secretion of histamines from mast cells
2
Phagocytosis of self cells

In type II reactions, an autoantibody is produced against the self cell that contains an
antigen or foreign protein on its surface. The self cells are then destroyed by
phagocytosis or lysis. Activation of T-cells takes place in delayed hypersensitivity.
Stimulation of thyroid cells and secretion of histamines are the mechanisms of type V
and type I hypersensitivities, respectively.
A patient presents with fever, arthralgia, rash, malaise, lymphadenopathy, nephritis, and
polyarthritis. The patient has been admitted to the hospital several times in the past
month and was last discharged one week ago. What does the nurse suspect this patient
is experiencing?
1
Serum sickness
2
Rheumatoid arthritis
3
Systemic lupus erythematosus
4
Hemolytic transfusion reaction
1
Serum sickness

The symptoms of fever, arthralgia, rash, malaise, lymphadenopathy, nephritis, and


polyarthritis are classic signs and symptoms of serum sickness, which develops after
receiving serum or certain drugs. The patient would have received these in previous
hospital visits. Symptoms of rheumatoid arthritis include tender, warm, swollen joints;
joint stiffness (especially in the morning and during periods of inactivity); fatigue; fever;
and weight loss. Symptoms of systemic lupus erythematosus are swollen or painful
joints, unexplained fever, and a red rash (most commonly on the nose and cheeks).
Symptoms of a hemolytic transfusion reaction include back pain, bloody urine, chills,
fainting or dizziness, fever, flank pain, and flushing of the skin.
A patient is diagnosed with a type I hypersensitivity. Which antibody level increases
during this type of allergy?
1
IgM
2
IgD
3
IgE
4
Immunoglobulin G (IgG)
3
IgE

In a patient exposed to an allergen, elevated levels of IgE antibodies are produced. The
elevated level of antibodies causes inflammation, erythema, or edema in the affected
region. IgG antibodies play a crucial role in type II allergies. IgM antibodies act at the
first stage of an infection caused by viruses, microorganisms, or toxins. The role of IgD
antibodies is not clearly known, but they activate B-cells during infection.
A patient has been ordered by the health care provider to carry a "bee sting kit" at all
times. What does the nurse teach the patient about the care and use of the device?
1
Carry a drug-filled device at all times.
2
Inject the drug into the forearm when required.
3
Obtain a replacement device if the drug is discolored.
4
Go to the nearest hospital for a 2-hour observation after using the device.
3
Obtain a replacement device if the drug is discolored.

The patient should obtain a replacement device if the drug is discolored; the drug must
be clear, with sufficient time left before the expiration date. The patient must keep at
least two drug-filled devices on hand in case more than one dose is needed. The drug is
injected into the thigh muscle, not the forearm. The patient must go to the nearest
hospital for observation for at least 4 to 6 hours after the device is used.
A patient is undergoing a skin test for allergies. Which symptoms are associated with an
anaphylactic reaction? Select all that apply.
1
Red blotches
2
Itching and urticaria
3
High blood pressure
4
Diarrhea and vomiting
5
Erythema and angioedema
1
Red blotches
2
Itching and urticaria
5
Erythema and angioedema

Red blotches, itching, urticaria, erythema, and angioedema are the common symptoms
associated with anaphylactic reactions. During this reaction, vasodilation occurs and
blood is lost from the vascular system; therefore, the patient becomes hypotensive, not
hypertensive. Diarrhea and vomiting are not directly related to this reaction.
A woman who is breastfeeding is diagnosed with rheumatoid arthritis. She does not
want to stop breastfeeding. Which medication is safest for her to take?
1
Anakinra (Kineret)
2
Abatacept (Orencia)
3
Adalibumab (Humira)
4
Tocilizumab (Actemra)
3
Adalibumab (Humira)

Adalibumab (Humira) is a monoclonal antibodies to tumor necrosis factor. Because it is


large with high molecular weight, it is unlikely to enter the breast milk. Anakinra (Kineret)
is monoclonal antibodies to interleukin-1. Abatacept (Orencia) is a CTLA4 agonist.
Toclizumab (Actemra) is monoclonal antibodies to interleukin-6. There is not enough
sufficient evidence for any of these drugs to know how they affect breastfeeding
outcomes so use is not recommended.
Which type of hypersensitivity is anaphylaxis?
1
Rapid hypersensitivity
2
Delayed hypersensitivity
3
Cytotoxic hypersensitivity
4
Immune-complex mediated hypersensitivity
1
Rapid hypersensitivity

Type 1 hypersensitivity is also known as rapid hypersensitivity. Anaphylaxis is an


example of type I hypersensitivity reaction, which occurs rapidly and systemically.
Anaphylaxis is due to the reaction of immunoglobulin E (IgE) antibodies on mast cells
with antigen, resulting in the release of mediators, especially histamine, which causes
the allergic reaction. Anaphylaxis is not an example of delayed, cytotoxic, or immune-
complex mediated hypersensitivity.
What is the role of decongestants in the treatment of a type I hypersensitivity?
1
Reduce edema
2
Clear the allergen
3
Prevent the release of vasoactive amines
4
Prevent the synthesis of vasoactive amines
1
Reduce edema

Decongestants usually contain ephedrine, phenylephrine, or pseudoephedrine. These


drugs cause vasoconstriction in the inflamed area and reduce edema. Antihistamines
clear the allergens and inhibit the release of vasoactive amines. Corticosteroids inhibit
the synthesis of vasoactive agents.
A patient is suffering with type I hypersensitivity. Which questions does the nurse ask
during the assessment? Select all that apply.
1
"Do you have any food or drug allergies?"
2
"What is your favorite food and beverage?"
3
"Have you undergone a transplantation surgery before?"
4
"Do you have any family members with a type I allergy?"
5
"When did this problem begin and how long does each episode last?"
1
"Do you have any food or drug allergies?"
4
"Do you have any family members with a type I allergy?"
5
"When did this problem begin and how long does each episode last?"

In order to identify the causative allergen, the nurse asks if the patient has any allergy to
some foods, drugs, or other chemical compounds. The exposure to such compounds
can result in mild to severe type I reactions. This type of hypersensitivity can be a
hereditary problem; therefore, asking about family history of the disease will be useful.
In order to understand the severity of the disease, information about onset and duration
can be beneficial. Rather than asking about favorite foods or beverages, the nurse
should inquire about any allergies to food items or beverages. Transplantation-related
problems are associated with autoimmunity, not with type I hypersensitivity.
A patient recently admitted to the hospital is to receive an antibiotic intravenously for the
first time for a urinary tract infection. Before checking the five rights prior to
administration, what is the nurse's first action?
1
Check the patient's identification band for any allergies.
2
Check with the pharmacy for any known allergies for this patient.
3
Review the clinical records and ask the patient about any known allergies.
4
Ask the nurse who previously cared for the patient about any known allergies.
3
Review the clinical records and ask the patient about any known allergies.

The clinical record should have all known hypersensitivities listed for the patient. The
patient should also be asked directly about any known allergies. The pharmacy is not
responsible for obtaining information on all of the patient's known allergies. Checking
the patient's identification band for allergies is part of the "five rights" process at the
bedside before the medication is given. Asking the previous nurse is not an appropriate
safety measure before medication administration.

What is the most common type of allergy?


atopic allergy (type 1 hypersensitivity reactions)
What is an atopic allergy caused by?
increased production of immunoglobulin E
Angioedema is what type of reaction?
type 1 - severe (atopic)
What assessment finding will a nurse see with angioedema?
deep, firm swelling (of the face, lips, tongue, neck)
What are the priority problems for patients with angioedema?
risk for airway obstruction due to mucosal swelling and anxiety due to cerebral hypoxia
What type of allergy is anaphylaxis?
life-threatening type 1
What is the best treatment for anaphylaxis?
prevention (carry alert bracelet and 2 epi pens)
What are some assessment findings that the nurse would see with anaphylaxis?
uneasiness, itching, angioedema, hives, dyspnea
What are priority interventions with anaphylaxis?
assess gas exchange first, stabilize airway, epinephrine, antihistamines, oxygen,
corticosteroids
What drugs most commonly cause angioedema?
ACE inhibitors
What is rhinosinusitis?
inflammation of the sinus cavities (hay fever)
What are some allergens that trigger hay fever?
pollen, molds, dust, animal dander, wool, etc
What is a cytotoxic reaction?
When the body makes antibodies directed at self cells that have foreign protein
attached to them
What is type 2 hypersensitivity?
cytotoxic reaction
What is type 3 hypersensitivity?
immune complex reactions
What is an immune complex reaction?
when excess antigens cause immune complexes to form in blood (that get lodged in
small blood vessels)
What are usual sites of immune complex reactions?
kidneys, skin, joints
Name two examples of immune complex reactions:
rheumatoid arthritis and lupus erythematosus
What is type 4 hypersensitivity?
delayed hypersensitivity
What is a delayed hypersensitivity reaction?
when lymphocytes and macrophages collect and cause edema and tissue damage
Name some examples of delayed hypersensitivity reactions:
contact dermatitis, poison ivy, local response to insect stings
What is autoimmunity?
when the immune system targets self cells
A patient comes to the ED with shortness of breath and facial edema, reporting
rhinorrhea (runny nose) and a severe headache. The triage nurse notes that the
patient's lips and eyes are very swollen. What assessment history questions should be
asked at this time?
ask about onset and duration of problems, ask about possible environmental exposures,
ask is close relatives have any allergies
The nurse asks the patient if she has any allergies. The patient states that she has an
allergy to peanuts, but has not had anything to eat today. The patient also states that
she worked earlier today as a nursing assistant. Which laboratory tests does the nurse
anticipate the provider will order?
Immunoglobulin E and CBC with differential
The patient states she has an allergy to peanuts. Has not eaten anything today but
worked earlier as a CNA. Based on the patient's history of new onset of shortness of
breath and edema of the eyes and lips, what does the nurse suspect is the problem?
anaphylactic reaction (peanut allergy has been linked to latex allergy)
The patient states she has an allergy to peanuts. Has not eaten anything today but
worked earlier as a CNA. What questions are appropriate to ask this patient?
if she used latex gloves at work and what procedures she may have performed before
symptoms started
Five minutes later, the patient continues to experience shortness of breath. She states
that she is feeling anxious and frightened. What is the appropriate nursing intervention?
assess the patient for signs of respiratory distress
The patient is experiencing an anaphylactic reaction. What therapies does the nurse
anticipate the provider will order?
epinephrine, diphenhydramine, IV fluids, oxygen
A patient has been admitted to the ED with bilateral eyelid swelling and subsequent
difficulty seeing. What is the priority nursing assessment?
airway
A nurse is administering a new medication intravenously to a patient. The patient
becomes short of breath and begins to experience itching and hives. What is the priority
nursing response?
stop the infusion
Actinomycin
an antimicrobial medication that acts on susceptible pathogens by inhibiting nucleic acid
synthesis
Penicillin acts on?
susceptible pathogens by inhibiting cell wall synthesis.
erythromycin acts on?
susceptible pathogens by inhibiting biosynthesis and reproduction.
cephalosporin acts on?
susceptible pathogens by inhibiting cell wall synthesis.
Syphilis is an STI caused by?
Treponema pallidum
Gonorrhea is caused by?
Neisseria gonorrhoeae
What causes genital warts?
Klebsiella granulomatis
Haemophilus ducreyi
What can cause vulvovaginitis?
Herpes simplex virus, Trichomonas vaginalis, and Candida albicans.
Hypersensitivity reactions
Type 1-4
Type I: anaphylaxisType II: Cytotoxic (antibody-IgM and IgG mediated)Type III: Immune
complex mediated Type IV: Delayed type (T cell mediated)

An example of a type I hypersensitivity reaction is anaphylaxis. Serum sickness is a


type III immune complex reaction. Contact dermatitis is a type IV delayed
hypersensitivity reaction. A blood transfusion reaction is a type II cytotoxic reaction.
Dermatitis related to a sexually transmitted infection
Phthirus pubis
IgA presents in?
tears, saliva, and breast milk.
IgE and IgG immunoglobulins are present in?
plasma and interstitial fluids.
IgM immmunoglobulin is present in?
plasma
What cytokine increases growth and maturation of myeloid stem cells?
Granulocyte-macrophage colony-stimulating factor.
What is Interleukin-2?
A cytokine that increases growth and differentiation of T-lymphocytes.
What is Thrombopoietin?
A cytokine that increases growth and differentiation of platelets.
What is Granulocyte colony-stimulating factor?
A cytokine that increases numbers and maturity of neutrophils.
Hepatitis A virus spreads through?
Contaminated food and water.
Hepatitis B,C, and D viruses spread through?
contaminated needles, syringes, and blood products.
Cervicitis caused by?
Chlamydia trachomatis
Gonorrhea caused by
Neisseria gonorrhoeae
Genital warts caused by
Human papillomavirus
Genital herpes is caused by
Herpes simplex virus
Which cells are affected in DiGeorge syndrome?
This is a primary immune deficiency disorder in which T-cells are affected.
The first antibody formed after exposure to an antigen?
IgM, provides the primary immune response.
What is IgA responsible for?
Has very low circulating levels and is responsible for preventing infection in the upper
and lower respiratory tracts, and the gastrointestinal and genitourinary tracts.
IgE is associated with?
IgE has variable concentrations in the blood and is associated with antibody-mediated
hypersensitivity reactions.
IgG is expressed...
Heavily expressed on second and subsequent exposures to antigens to provide
sustained, long-term immunity against invading microorganisms.
IgD is present on? and function?
Presents on the lymphocyte surface. Differentiates B-lymphocytes.
Name a leukotriene antagonist drug that prevents the synthesis of leukotriene's and
helps in managing and preventing hay fever?
Zileuton.
Cromolyn sodium stabilizes?
This stabilizes mast cells and prevents the opening of mast cell membranes in response
to allergens binding to immunoglobulin E.
Which virus is responsible for mononucleosis and possibly Burkitt's lymphoma?
Epstein-Barr virus
Which drug is considered to be the first-line of medication for the management of
anaphylaxis?
Isoproterenol, a beta-adrenergic. Sympathomimetic drug
Microglial
alveolar macrophages
Kupffer cells
Mesangial cells

located...
brain
lungs
liver
kidneys
What bacteria causes toxic shock syndrome?
Staphylococcus aureus

Which statement made by the nurse is most appropriate in teaching patient


interventions to minimize the effects of seasonal allergic rhinitis?

A. "You will need to get rid of your pets."


B. "You should sleep in an air-conditioned room."
C. "You would do best to stay indoors during the winter months."
D. "You will need to dust your house with a dry feather duster twice a week."
B. "You should sleep in an air-conditioned room."

Seasonal allergic rhinitis is most commonly caused by pollens from trees, weeds, and
grasses. Airborne allergies can be controlled by sleeping in an air-conditioned room,
daily damp dusting, covering the mattress and pillows with hypoallergenic covers, and
wearing a mask outdoors.
When caring for a patient with a known latex allergy, the nurse would monitor the
patient closely for a cross-sensitivity to which foods (select all that apply)?

A. Grapes
B. Oranges
C. Bananas
D. Potatoes
E. Tomatoes
A, C, D, E

Because some proteins in rubber are similar to food proteins, some foods may cause an
allergic reaction in people who are allergic to latex. The most common of these foods
are bananas, avocados, chestnuts, kiwi fruit, tomatoes, water chestnuts, guava,
hazelnuts, potatoes, peaches, grapes, and apricots.
Ten days after receiving a bone marrow transplant, a patient develops a skin rash on
his palms and soles, jaundice, and diarrhea. What is the most likely etiology of these
clinical manifestations?

A. The patient is experiencing a type I allergic reaction.


B. An atopic reaction is causing the patient's symptoms.
C. The patient is experiencing rejection of the bone marrow.
D. Cells in the transplanted bone marrow are attacking the host tissue.
D. Cells in the transplanted bone marrow are attacking the host tissue.

The patient's symptoms are characteristic of graft-versus-host-disease (GVHD) in which


transplanted cells mount an immune response to the host's tissue. GVHD is not a type I
allergic response or an atopic reaction, and it differs from transplant rejection in that the
graft rejects the host rather than the host rejecting the graft.
A patient's low hemoglobin and hematocrit have necessitated a transfusion of packed
red blood cells (RBCs). Shortly after the first unit of RBCs starts to infuse, the patient
develops signs and symptoms of a transfusion reaction. Which type of hypersensitivity
reaction has the patient experienced?

A. Type I
B. Type II
C. Type III
D. Type IV
B. Type II

Transfusion reactions are characterized as a type II (cytotoxic) reaction in which


agglutination and cytolysis occur. Type I hypersensitivity reactions are IgE-mediated
reactions to specific allergens (e.g., exogenous pollen, food, drugs, or dust). Type III
reactions are immune-complex reactions that occur secondary to antigen-antibody
complexes. Type IV reactions are delayed cell-mediated immune response reactions.
On initial assessment of an older patient, the nurse knows to look for certain types of
diseases because which immunologic response increases with age?

A. Autoimmune response
B. Cell-mediated immunity
C. Hypersensitivity response
D. Humoral immune response
A. Autoimmune response

With aging, autoantibodies increase, which lead to autoimmune diseases (e.g., systemic
lupus erythematosus, acute glomerulonephritis, rheumatoid arthritis, hypothyroidism).
Cell-mediated immunity decreases with decreased thymic output of T cells and
decreased activation of both T and B cells. There is a decreased or absent delayed
hypersensitivity reaction. Immunoglobulin levels decrease and lead to a suppressed
humoral immune response in older adults.
A 21-year-old student had taken amoxicillin once as a child for an ear infection. She is
given an injection of Penicillin V and develops a systemic anaphylactic reaction. What
manifestations would be seen first?

A. Dyspnea
B. Dilated pupils
C. Itching and edema
D. Wheal-and-flare reaction
C. Itching and edema

A systemic anaphylactic reaction starts with edema and itching at the site of exposure to
the antigen. Shock can rapidly develop with rapid, weak pulse; hypotension; dilated
pupils; dyspnea, and possible cyanosis. The wheal-and-flare reaction occurs with a
localized anaphylactic reaction such as a mosquito bite.
The patient with an autoimmune disease will be treated with plasmapheresis. What
should the nurse teach the patient about this treatment?

A. It will gather platelets for use later when needed.


B. It will cause anemia because it removes whole blood and RBCs are damaged.
C. It will remove the IgG autoantibodies and antigen complexes from the plasma.
D. It will remove the peripheral stem cells in order to cure the autoimmune disease.
C. It will remove the IgG autoantibodies and antigen complexes from the plasma.

Plasmapheresis removes plasma that contains autoantibodies (usually IgG class) and
antigen-antibody complexes to remove the pathologic substances in the plasma without
causing anemia. Plateletpheresis removes platelets from normal individuals for use by
patients with low platelet counts. Apheresis is used to collect stem cells from peripheral
blood that does not cure autoimmune disease.
The patient with diabetes mellitus has been ill for some time with a severe lung infection
needing corticosteroids and antibiotics. The patient does not feel like eating. The nurse
understands that this patient is likely to develop

A. major histoincompatibility.
B. primary immunodeficiency.
C. secondary immunodeficiency.
D. acute hypersensitivity reaction.
C. secondary immunodeficiency.

Secondary immunodeficiency is most commonly caused by immunosuppressive drugs,


such as corticosteroids. It can also be caused by diabetes mellitus, severe infection,
malnutrition, and chronic stress, all of which are present in this patient. The other
options are not possible for this patient. Histoincompatibility occurs when the human
leukocyte antigen (HLA) system of the donor is not compatible with the recipient's HLA
genes. Primary immunodeficiency is rare and includes phagocytic defects, B cell
deficiency, T cell deficiency, or a combination of B cell and T cell deficiency. Acute
hypersensitivity reaction is an anaphylactic-type allergic reaction to an antigen.
Which statement by the patient who has had an organ transplant would indicate that the
patient understands the teaching about the immunosuppressive medications?

A. "My drug dosages will be lower because the medications enhance each other."
B. "Taking more than one medication will put me at risk for developing allergies."
C. "I will be more prone to malignancies because I will be taking more than one drug."
D. "The lower doses of my medications can prevent rejection and minimize the side
effects."
D. "The lower doses of my medications can prevent rejection and minimize the side
effects."

Because immunosuppressants work at different phases of the immune response, lower


doses of each drug can be used to produce effective immunosuppression while
minimizing side effects. The use of several medications is not because they enhance
each other and does not increase the risk of allergies or of malignancies.

three categories for disorders of the immune system:


hypersensitivity reactions, autoimmune disorders, immune deficiencies.
examples of hypersensitivity reactions:
anaphylaxis, hemolytic transfusion reactions, measles, transplant rejections.
examples of autoimmune disorders:
rheumatoid arthritis, ulcerative colitis, multiple schlerosis.
examples of immune deficiencies:
hypogammaglobulinemia, acquired immunodeficiency syndrome (AIDS).
immune system can cause injury to the body because of its:
exaggerated response.
hypersensitivity reactions are classified according to:
the way the tissue is injured.
type I hypersensitivity reaction:
anaphylactic reaction, must have had a previous exposure to the antigen, occurs when
the patient has a positive reaction to a scratch test.
type I reactions can range from:
mild to severe and life threatening.
during the first exposure to an antigen in type I reactions:
immunoglobulin E (IgE) antibodies are made.
in type I reactions when subsequent exposure occurs, the antigen causes IgE to:
trigger mast cells to release their contents ex. histamine.
histamine causes:
vasodilation, changes in vascular permeability, an increase in mucous production,
contraction of various smooth muscles.
in type I reactions if the second antigen exposure is localized:
the reaction is mild and remains local.
in type I reactions if the second antigen exposure is systemic:
the reaction is massive and widespread.
severest form of type I reactions:
anaphylaxis, urticarial, angioedema.
most common form of allergy:
allergic rhinitis.
allergic rhinitis is the result of:
an antigen-antibody reaction, ciliary action decreases and mucous secretions increase,
vasodilation and local tissue edema occur.
signs and symptoms of allergic rhinitis:
sneezing, nasal itching, profuse watery rhinorrhea (runny nose), itchy red eyes, allergic
shiners.
allergic shiners:
dark circles under the eyes caused by venous congestion in the maxillary sinuses.
diagnostic test for allergic rhinitis:
skin testing to identify the specific offending allergens.
therapeutic measures for allergic rhinitis:
eliminating the stimuli, antihistamines, nasal decongestants, corticosteroids,
immunotherapy.
immunotherapy:
allergy shots, reserved for severe or debilitating symptoms of allergic rhinitis.
atopic dermatitis (eczema):
inflammatory skin response.
signs and symptoms of eczema:
pruritus, edema, extremely dry skin, blisters.
diagnostic tests for eczema:
no tests to confirm this diagnosis, detailed history and physical examination.
therapeutic measures for eczema:
lukewarm soaks, topical corticosteroids, topical calcineurin inhibitors.
if the skin lesions from eczema become infected:
topical or systemic antibiotics are used.
severe systemic type I reaction:
anaphylaxis.
pathophysiology of anaphylaxis:
IgE antibodies produced from the previous antigen sensitization are attached to mast
cells throughout the body.
anaphylaxis causes a widespread release of;
histamine and other chemical mediators contained within the mast cells.
signs and symptoms of anaphylaxis:
dyspnea, laryngeal edema, respiratory arrest, hypotension, tachycardia.
diagnostic tests for anaphylaxis needed to guide symptom treatment:
arterial blood gases, electrocardiogram (ECG) monitoring.
anaphylaxis is diagnosed based on:
physical assessment and history.
therapeutic measures for anaphylaxis:
IV access, epinephrine, vasopressor, fluids to increase BP, oxygen therapy, mechanical
ventilation, Antihistamines, corticosteroids.
urticaria:
hives, release of histamine.
urticaria, anaphylaxis, angioedema, eczema, allergic rhinitis are type _ reactions:
I.
signs and symptoms of urticaria:
lesions are raised, pruritic, nontender, erythematous wheals on the skin, concentrated
on trunk and proximal extremities.
diagnostic tests for urticaria are based on:
physical examination and history.
in severe cases of urticaria what may be given:
epinephrine.
therapeutic measures for urticaria:
corticosteroids (given orally, topically, or IV), Antihistamines, histamine (H2) blockers.
form of urticaria:
angioedema.
signs and symptoms of angioedema:
eruptions are nonpruritic and painless, are of the dermal and subcutaneous layers of the
skin, mucous membrane edema.
unlike epidermal eruption of urticaria, angioedema eruption are of the:
dermal and subcutaneous layers of the skin.
diagnostic test for angioedema:
comprehensive history and physical examination, skin testing.
type II hypersensitivity reaction:
destruction of a cell or substance that has an antigen attached to its cell membrane.
type II reaction causes lysis of the cell of accelerated phagocytosis (engulfing and
ingestion) which is normally good except :
when a red blood cell (RBC) is sensed as foreign.
hemolytic transfusion reaction, is an example of type _ reaction:
II.
in hemolytic transfusion reactions incompatible surface antigens on RBCs are:
transfused.
in hemolytic transfusion reactions the recipient's antibodies attach to the foreign
antigens on the transfused RBCs causing:
rapid lysis (destruction) of the RBC.
in hemolytic transfusion reactions rapid RBC lysis result in:
a massive amount of cellular debris that blocks blood vessels throughout the body.
in hemolytic transfusion reactions cellular debris that block blood vessels throughout the
body leads to:
ischemia, necrosis of tissue and organs, can be life threatening.
signs and symptoms of hemolytic transfusion reaction:
low back/chest pain, hypotension, fever rising more than 1.8F, chills, tachycardia,
tachypnea, wheezing, dyspnea, urticaria, anxiety, headache, nausea.
diagnostic tests for hemolytic transfusion reaction:
direct Coombs test.
therapeutic measures for hemolytic transfusion reaction:
Antihistamines, corticosteroids, epinephrine, diuretics, sympathominetics.
vital sins are taken when during a blood transfusion:
before the start of the transfusion, 15 minutes into the transfusion, and when the
transfusion is completed.
how long do you stay with the patient during a blood transfusion:
first 15 minutes.
direct Coombs' test:
small amount of pt's RBC are washed, antihuman globulin is added, if agglutination
(clumping) occurs an immune reaction is occurring.
serum sickness is a type _ reaction:
III.
type III hypersensitivity reaction:
involves immune complexes formed by antigen and antibodies.
symptoms of type III hypersensitivity reaction ranging from:
red edematous skin lesion to hemorrhage and necrosis.
reaction of type III reaction is:
localized and evolves over several hours.
serum sickness occurs occasionally after:
administration of penicillin or sulfonamide.
serum sickness:
antigen-antibody form and lodge in small vessels-leads to inflammation, tissue damage,
and necrosis.
signs and symptoms of serum sickness:
severe urticaria/angioedema, fever, malaise, muscle soreness, splenomegaly, nausea,
vomiting, diarrhea, lymphadenopathy.
signs and symptoms of serum sickness usually occur __ after the exposure:
7 days to 3 weeks.
diagnostic test for serum sickness:
slight elevation in the white blood cell (WBC) count, sedimentation rate and C-reactive
protein, IgG and IgM increase substantially.
therapeutic measures for serum sickness:
antipyretics, antihistamines and epinephrine may be given for urticaria and angioedema.
treatment for serum sickness is focused on:
symptoms.
contact dermatitis and transplant rejection are type __ reactions:
IV.
type IV hypersensitivity reaction is also called a:
delayed reaction.
type IV reaction:
sensitized T lymphocyte comes in contact with antigen, cell-mediated immune response
results in necrosis.
contact dermatitis:
chemical comes in contact with the skin.
with contact dermatitis with the first contact there is:
no reaction of symptoms.
with contact dermatitis with subsequent exposures:
T memory cells secrete the chemicals that may cause symptoms.
the ____ cause symptoms in contact dermatitis:
subsequent exposures.
common irritants that cause contact dermatitis:
poison ivy, poison oak, latex rubber.
signs and symptoms of contact dermatitis:
area of contact becomes red and pruritic with fragile vesicles.
therapeutic measures for contact dermatitis:
tepid baking soda baths, aveeno baths, wash with brown soap, topical drying agents,
antihistamines, corticosteroids, avoid scratching skin.
transplant rejection is a type ___ reaction:
IV.
lifelong immunosuppression is needed to help prevent:
transplant rejection.
transplant rejection is any form of:
transplanted living tissue that is sensed as foreign.
pathophysiology of transplant rejection:
lymphocytes become sensitized immediately and invade the transplanted tissue and
destroy it.
complications of transplant rejection:
total failure and loss of transplanted tissue or organ, death.
greatest cause of death following a transplant rejection:
infection.
nursing care during transplants:
observing for signs of rejection.
diagnostic tests for transplant rejection:
biopsy, scans, blood tests, arteriography, ultrasonography.
therapeutic measures for transplant rejection:
depends on the type of transplant. preventive preop prep with medications,
transfusions, or radiation to minimize the risk of rejection.
priority nursing diagnoses for transplant rejection:
grieving, fear, deficient knowledge, other diagnoses depend on which organ is failing.
pernicious anemia is an example of an:
autoimmune disorder.
pathophysiology of autoimmune disorders:
immune response destroys cells, immune system recognizes body's own cells as
foreign.
pernicious anemia:
antibodies destroy gastric parietal cells and intrinsic factor, vitamin B12 deficiency may
result causing decreased production of RBCs.
causes of pernicious anemia:
gastric or small-bowel restrictions.
signs and symptoms of pernicious anemia:
increasing weakness, loss of appetite, glossitis, pallor, irritability, confusion, numbness
or tingling in the extremities (peripheral neuropathy).
diagnostic tests for pernicious anemia:
folic acid deficiency, low vitamin B12 levels, macrocytic anemia.
therapeutic measures for pernicious anemia:
corticosteroids, vitamin B12 therapy is needed usually for life.
nursing care for pernicious anemia:
vitamin B12 & folic acid are administered as ordered, care related to fatigue and safety,
ambulation, frequent rest periods, providing assistance with ADLs.
idiopathic autoimmune hemolytic anemia is an example of an:
autoimmune disorder.
idiopathic autoimmune hemolytic anemia:
autoantibodies attach to RBCs, cause lyse or agglutinate (clump), agglutinations occurs,
occlusions in the small blood vessels followed by tissue ischemia.
signs and symptoms of idiopathic autoimmune hemolytic anemia:
fatigue, pallor, hypotension, dyspnea, jaundice.
diagnostic tests for idiopathic autoimmune hemolytic anemia:
low: RBC, hemoglobin (Hgb), hematocrit (Hct). elevated: lactate dehydrogenase (LDH),
serum bilirubin levels. fragmented RBCs.
therapeutic measures for idiopathic autoimmune hemolytic anemia:
folic acid, immunosuppressant meds, corticosteroids, blood transfusions,
erythrocytapheresis, splenectomy.
erythrocytapheresis:
abnormal RBCs are removed and replaced with normal RBCs.
nursing care for idiopathic autimmune hemolytic anemia:
signs and symptoms should be monitored and reported, frequent rest periods, blood
products are administered as ordered.
hashimoto's thyroiditis is an example of an:
autoimmune disorder.
hashimoto's thyroiditis:
autoantibodies for thyroid-stimulating hormone (TSH) form, thyroid gland is
overstimulated, autoantibodies destroy thyroid-hypothyroidism, lifelong thyroxine.
initial signs and symptoms of hashimoto's thyroiditis:
hyperthyroidism, restlessness, tremors, chest pain, increased appetite, diarrhea, moist
skin, heat intolerance, weight loss.
later signs and symptoms of hashimoto's thyroiditis:
hypothyroidism, fatigue, bradycardia, hypotension, dyspnea, anorexia, constipation, dry
skin, weight gain, sensitivity to cold, facial puffiness, slowing of mental processes.
diagnostic tests for hashimoto's thyroiditis:
hard to diagnose, elevated: serum TSH levels. low: T3, T4 levels. thyroid scan.
lupus erythematosus is an example of an:
autoimmune disorder.
types of lupus:
neonatal, drug-induced, discoid, systemic.
what type of lupus is life threatening because it is progressive:
systemic type.
African American, Hispanics, Native Americans, and Asians are more likely to develop:
systemic lupus erythematosus (SLE).
lupus most often affects:
women between the ages of 15 and 40.
systemic lupus:
chronic, inflammatory, multisystem disorder.
early symptoms of systemic lupus are:
vague, then fatigue, fever.
complications of lupus:
renal failure, thrombocytopenia, emboli, sepsis.
signs and symptoms of discoid lupus:
skin lesion, patch crusty sharply defined skin plaques, occur on face or sun-exposed
areas.
signs and symptoms of drug-induced lupus:
pleuropericardial inflammation, fever, rash, arthritis.
symptoms of systemic lupus:
butterfly rash (face), photosensitivity, alopecia, pain, pruritus, bruising, arthralgia,
arthritis, anemia, leukocytopenis, thrombocytopenis, pericarditis, myocarditis,
myocardial infarction, vasculitis, pleurisy, valvular heart disease.
leukocytopenia:
decreased WBC.
thrombocytopenia:
decreased platlets.
systemic lupus affects what areas:
dermatological, musculoskeletal, hematologic, cardiopulmonary, renal, central nervous
system, gastrointestinal, opthalmological.
treatment of lupus:
IV immunoglobulin, symptomatic management, antimalarials, corticosteroids, NSAIDs,
immunosuppressants.
nursing care for lupus:
need at least 8 hours of sleep, prevention of exacerbations (flares), warm baths, well
balanced diet, exercise to keep range of motion.
education for lupus:
explains the signs of bleeding, regular eye exams, medical alert bracelet, stop smoking,
facial puffiness, foamy or coke-colored urine.
Type I hypersensitivity reactions:
Allergic rhinitis, eczema, anaphylaxis, urticaria, angioedema.
Type II hypersensitivity reactions:
Hemolytic transfusion reaction.
Type III hypersensitivity reactions:
Serum sickness.
Type IV hypersensitivity reactions:
Contact dermatitis, transplant rejection.
Autoimmune disorders:
Pernicious anemia, idiopathic autoimmune hemolytic anemia, Hashimoto's thyroiditis,
lupus erythematosus.

Decongestants
Cause systemic vasoconstriction, so increases BP
Hypersensitivity/Allergy
Increased or excessive response to the presence of an antigen to which the client has
been exposed. Overproduction of IgE. Degree of reaction ranges from uncomfortable to
life threatening.
Type 1: Rapid Hypersensitivity Reactions
Most common type. AKA atopic allergy. Can be local; just in the areas exposed to the
antigen. OR systemic; affecting the whole body.
Examples of Type 1 Reactions
latex allergy
Allergic asthma
Anaphylaxis
Ways that we contact allergens
*Inhaled (plant pollens, fungal spores, animal
dander, house dust, grass, ragweed)
*Ingested (foods, food additives, drugs)
*Injected (bee venom, drugs, biologic substances)
*contacted (pollens, foods, environmental
proteins)
Collaborative Management of Allergies
History / close relative with allergies,
Physical/Assesments / s/s
Lab Assessments:
Increased eosinophils
Possibly increased WBCs
Increased IgE
RAST (radioallergosorbant test)
RAST (radioallergosorbant test)
a type of blood test used to test for allergies. These tests, which use radioactivity, have
largely been abandoned by doctors in favor of newer, more accurate forms of allergy
testing.
Allergy Testing
*Skin testing/scratch and intradermal
*Oral food challenge
*Avoidance therapy
*Symptomatic therapy/premedicate
*Drug therapy/Zyrtec, Allegra, Singulair..
Drug therapy
Decongestants/ not for pt. with glaucoma, high
BP or urinary retention
Antihistamines dries secretions. not for
breastfeeding mothers
Corticosteroids (discontinue 5 days before
allergy testing) MONITOR BLOOD SUGAR
Mast cell stabilizers
Leukotriene antagonists
Complementary and alternative therapy/Essential
Oils, honey
Desensitization therapy/allergy shot 2-3 times per week for 5 years. Stay 20 minutes
after injection.
Anaphylaxis
The widespread reaction that occurs in response to contact with a substance to which
the person has a severe allergy (antigen); characterized by blood vessel and
bronchiolar smooth muscle involvement causing widespread bold vessel dilation,
decreased cardiac output, and bronchoconstriction; results in cell damage and the
release of large amounts of histamine, severe hypovolemia, vascular collapse,
decreased cardiac contraction, and dysrhythmias and causes extreme whole-body
hypoxia.
Anaphylaxis facts
Not common, episodes vary in severity, and it can be fatal. The major factor in fatal
outcomes is a delay in the administration of epinephrine.
*Drugs and dyes are common causes in acute care setting
*Food and insect stings are common causes in community settings.
Initial manifestations of anaphylaxis
feelings of uneasiness, apprehension, weakness, impending doom, anxious, frightened.
These feelings are followed quickly by generalized itching, urticaria.
Manifestations of anaphylaxis
Initial manifestations are followed by erythema and sometimes angioedema of the eyes,
lips, or tongue. Intensely itchy skin wheals or hives may appear and sometimes merge
to form large, red blotches.
Respiratory manifestations of anaphylaxis
Histamine and other mediators cause inflammation, bronchoconstriction, mucosal
edema, and excess mucus production. Symptoms include. Symptoms include
congestion, rhinorrhea, dyspnea, and increasing respiratory distress with audible
wheezing. Crackles, wheezing, and reduced breath sounds are heard. Pt may have
laryngeal edema as a lump in the throat, hoarseness, and stridor. Distress increases as
the tongue and larynx swell and more mucus is produced. Stridor increases as the
airway begins to close. Increasing bronchoconstriction can lead to reduced chest
movement and impaired airflow. Respiratory failure may follow from laryngeal edema,
suffocation, or lower airway constriction causing hypoxemia.
Cardiac manifestations of anaphylaxis
Patient is usually hypotensive and has a rapid, weak, irregular pulse from extensive
capillary leak and vasodilation. Patient is faint and diaphoretic with increasing anxiety
and confusion.

A patient with a family history of allergies has suffered an allergic response based on a
genetic predisposition. This atopic response is usually mediated by what
immunoglobulin?

A)

Immunoglobulin A

B)

Immunoglobulin M

C)

Immunoglobulin G

D)

Immunoglobulin E
D)

Immunoglobulin E
An office worker takes a cupcake that contains peanut butter. He begins wheezing, with
an inspiratory stridor and air hunger and the occupational health nurse is called to the
office. The nurse should recognize that the worker is likely suffering from which type of
hypersensitivity?

A)

Anaphylactic (type 1)

B)

Cytotoxic (type II)

C)

Immune complex (type III)

D)

Delayed-type (type IV)


A)

Anaphylactic (type 1)
A patient is learning about his new diagnosis of asthma with the asthma nurse. What
medication has the ability to prevent the onset of acute asthma exacerbations?

A)

Diphenhydramine (Benadryl)

B)

Montelukast (Singulair)

C)
Albuterol sulfate (Ventolin)

D)

Epinephrine
B)

Montelukast (Singulair)
A nurse is preparing a patient for allergy skin testing. Which of the following
precautionary steps is most important for the nurse to follow?

A)

The patient must not have received an immunization within 7 days.

B)

The nurse should administer albuterol 30 to 45 minutes prior to the test.

C)

Prophylactic epinephrine should be administered before the test.

D)

Emergency equipment should be readily available.


D)

Emergency equipment should be readily available


A patient who is scheduled for a skin test informs the nurse that he has been taking
corticosteroids to help control his allergy symptoms. What nursing intervention should
the nurse implement?

A)

The patient should take his corticosteroids regularly prior to testing.


B)

The patient should only be tested for grass, mold, and dust initially.

C)

The nurse should have an emergency cart available in case of anaphylaxis during the
test.

D)

The patients test should be cancelled until he is off his corticosteroids.


D)

The patients test should be cancelled until he is off his corticosteroids.


A patient has developed severe contact dermatitis with burning, itching, cracking, and
peeling of the skin on her hands. What should the nurse teach the patient to do?

A)

Wear powdered latex gloves when in public.

B)

Wash her hands with antibacterial soap every few hours.

C)
Maintain room temperature at 75F to 80F whenever possible.

D)
Keep her hands well-moisturized at all times.
D)
Keep her hands well-moisturized at all times.
A patient with severe environmental allergies is scheduled for an immunotherapy
injection. What should be included in teaching the patient about this treatment?

A)
The patient will be given a low dose of epinephrine before the treatment.

B)

The patient will remain in the clinic to be monitored for 30 minutes following the
injection.

C)

Therapeutic failure occurs if the symptoms to the allergen do not decrease after 3
months.

D)

The allergen will be administered by the peripheral intravenous route.


B)

The patient will remain in the clinic to be monitored for 30 minutes following the
injection.
The nurse in an allergy clinic is educating a new patient about the pathology of the
patients health problem. What response should the nurse describe as a possible
consequence of histamine release?

A)

Constriction of small venules

B)

Contraction of bronchial smooth muscle

C)

Dilation of large blood vessels

D)

Decreased secretions from gastric and mucosal cells


B)

Contraction of bronchial smooth muscle


The nurse is providing care for a patient who has experienced a type I hypersensitivity
reaction. What condition is an example of such a reaction?

A)

Anaphylactic reaction after a bee sting

B)

Skin reaction resulting from adhesive tape

C)

Myasthenia gravis

D)

Rheumatoid arthritis
A)

Anaphylactic reaction after a bee sting


A nurse is caring for a teenage girl who has had an anaphylactic reaction after a bee
sting. The nurse is providing patient teaching prior to the patients discharge. In the
event of an anaphylactic reaction, the nurse informs the patient that she should self-
administer epinephrine in what site?

A)

Forearm

B)

Thigh

C)
Deltoid muscle

D)

Abdomen
B)

Thigh
A nurse has included the nursing diagnosis of Risk for Latex Allergy Response in a
patients plan of care. The presence of what chronic health problem would most likely
prompt this diagnosis?

A)

Herpes simplex

B)

HIV

C)

Spina bifida

D)

Hypogammaglobulinemia
C)

Spina bifida
A patient has a documented history of allergies presents to the clinic. She states that
she is frustrated by her chronic nasal congestion, anosmia (inability to smell) and
inability to concentrate. The nurse should identify which of the following nursing
diagnoses?

A)
Deficient Knowledge of Self-Care Practices Related to Allergies

B)

Ineffective Individual Coping with Chronicity of Condition and Need for Environmental
Modification

C)

Acute Confusion Related to Cognitive Effects of Allergic Rhinitis

D)

Disturbed Body Image Related to Sequelae of Allergic Rhinitis


B)

Ineffective Individual Coping with Chronicity of Condition and Need for Environmental
Modification
A patients decline in respiratory and renal function has been attributed to Goodpasture
syndrome, which is a type II hypersensitivity reaction. What pathologic process
underlies the patients health problem?

A)

Antigens have bound to antibodies and formed inappropriate immune complexes.

B)

The patients body has mistakenly identified a normal constituent of the body as foreign.

C)

Sensitized T cells have caused cell and tissue damage.

D)

Mast cells have released histamines that directly cause cell lysis.
B)

The patients body has mistakenly identified a normal constituent of the body as foreign.
A child is undergoing testing for food allergies after experiencing unexplained signs and
symptoms of hypersensitivity. What food items would the nurse inform the parents are
common allergens?

A)

Citrus fruits and rice

B)

Root vegetables and tomatoes

C)

Eggs and wheat

D)

Hard cheeses and vegetable oils


C)

Eggs and wheat


A patient has been admitted to the emergency department with signs of anaphylaxis
following a bee sting. The nurse knows that if this is a true allergic reaction the patient
will present with what alteration in laboratory values?

A)

Increased eosinophils

B)

Increased neutrophils

C)
Increased serum albumin

D)

Decreased blood glucose


A)

Increased eosinophils
A nurse is aware of the need to assess patients risks for anaphylaxis. What health care
procedure constitutes the highest risk for anaphylaxis?

A)

Administration of the measles-mumps-rubella (MMR) vaccine

B)

Rapid administration of intravenous fluids

C)

Computed tomography with contrast solution

D)

Administration of nebulized bronchodilators


C)

Computed tomography with contrast solution


After the completion of testing, a childs allergies have been attributed to her familys cat.
When introducing the family to the principles of avoidance therapy, the nurse should
promote what action?

A)

Removing the cat from the familys home


B)

Administering OTC antihistamines to the child regularly

C)

Keeping the cat restricted from the childs bedroom

D)

Maximizing airflow in the house


A)

Removing the cat from the familys home


The nurse is providing health education to the parents of a toddler who has been
diagnosed with food allergies. What should the nurse teach this family about the childs
health problem?

A)

Food allergies are a life-long condition, but most families adjust quite well to the
necessary lifestyle changes.

B)

Consistent use of over-the-counter antihistamines can often help a child overcome food
allergies.

C)

Make sure that you carry a steroid inhaler with you at all times, especially when you eat
in restaurants.

D)

Many children outgrow their food allergies in a few years if they avoid the offending
foods.
D)

Many children outgrow their food allergies in a few years if they avoid the offending
foods.
A child has been diagnosed with a severe walnut allergy after suffering an anaphylactic
reaction. What is a priority for health education?

A)

The need to begin immunotherapy as soon as possible

B)

The need for the parents to carry an epinephrine pen

C)

The need to vigilantly maintain the childs immunization status

D)

The need for the child to avoid all foods that have a high potential for allergies
B)

The need for the parents to carry an epinephrine pen


An adolescent patients history of skin hyperreactivity and inflammation has been
attributed to atopic dermatitis. The nurse should recognize that this patient consequently
faces an increased risk of what health problem?

A)

Bronchitis

B)

Systemic lupus erythematosus (SLE)

C)
Rheumatoid arthritis

D)

Asthma
D)

Asthma
The nurse is planning the care of a patient who has a diagnosis of atopic dermatitis,
which commonly affects both of her hands and forearms. What risk nursing diagnosis
should the nurse include in the patients care plan?

A)

Risk for Disturbed Body Image Related to Skin Lesions

B)

Risk for Disuse Syndrome Related to Dermatitis

C)

Risk for Ineffective Role Performance Related to Dermatitis

D)

Risk for Self-Care Deficit Related to Skin Lesions


A)

Risk for Disturbed Body Image Related to Skin Lesions


A patient has been brought to the emergency department by EMS after being found
unresponsive. Rapid assessment reveals anaphylaxis as a potential cause of the
patients condition. The care team should attempt to assess for what potential causes of
anaphylaxis? Select all that apply.

A)
Foods

B)

Medications

C)

Insect stings

D)

Autoimmunity

E)

Environmental pollutants
A)

Foods

B)

Medications

C)

Insect stings
A school nurse is caring for a child who appears to be having an allergic response.
What should be the initial action of the school nurse?

A)

Assess for signs and symptoms of anaphylaxis.

B)
Assess for erythema and urticaria.

C)

Administer an OTC antihistamine.

D)

Administer epinephrine.
A)

Assess for signs and symptoms of anaphylaxis.


A patient is receiving a transfusion of packed red blood cells. Shortly after initiation of
the transfusion, the patient begins to exhibit signs and symptoms of a transfusion
reaction. The patient is suffering from which type of hypersensitivity?

A)

Anaphylactic (type 1)

B)

Cytotoxic (type II)

C)

Immunecomplex (type III)

D)

Delayed type (type IV)


B)

Cytotoxic (type II)


Which of the following individuals would be the most appropriate candidate for
immunotherapy?
A)

A patient who had an anaphylactic reaction to an insect sting

B)

A child with allergies to eggs and dairy

C)

A patient who has had a positive tuberculin skin test

D)

A patient with severe allergies to grass and tree pollen


D)

A patient with severe allergies to grass and tree pollen


A nurse has asked the nurse educator if there is any way to predict the severity of a
patients anaphylactic reaction. What would be the nurses best response?

A)

The faster the onset of symptoms, the more severe the reaction.

B)

The reaction will be about one-third more severe than the patients last reaction to the
same antigen.

C)

There is no way to gauge the severity of a patients anaphylaxis, even if it has occurred
repeatedly in the past.

D)
The reaction will generally be slightly less severe than the last reaction to the same
antigen.
A)

The faster the onset of symptoms, the more severe the reaction.
A nurse knows of several patients who have achieved adequate control of their allergy
symptoms using over-the-counter antihistamines. Antihistamines would be
contraindicated in the care of which patient?

A)

A patient who has previously been treated for tuberculosis

B)

A pregnant woman at 30 weeks gestation

C)

A patient who is on estrogen-replacement therapy

D)

A patient with a severe allergy to eggs


B)

A pregnant woman at 30 weeks gestation


A patient has been living with seasonal allergies for many years, but does not take
antihistamines, stating, When I was young I used to take antihistamines, but they
always put me to sleep. How should the nurse best respond?

A)

Newer antihistamines are combined with a stimulant that offsets drowsiness.

B)

Most people find that they develop a tolerance to sedation after a few months.
C)

The newer antihistamines are different than in years past, and cause less sedation.

D)

Have you considered taking them at bedtime instead of in the morning?


C)

The newer antihistamines are different than in years past, and cause less sedation.
A child has been transported to the emergency department (ED) after a severe allergic
reaction. The ED nurse is evaluating the patients respiratory status. How should the
nurse evaluate the patients respiratory status? Select all that apply.

A)

Facilitate lung function testing.

B)

Assess breath sounds.

C)

Measure the childs oxygen saturation by oximeter.

D)

Monitor the childs respiratory pattern.

E)

Assess the childs respiratory rate.


B)

Assess breath sounds.


C)

Measure the childs oxygen saturation by oximeter.

D)

Monitor the childs respiratory pattern.

E)

Assess the childs respiratory rate.


A patient with multiple food and environmental allergies tells the nurse that he is
frustrated and angry about having to be so watchful all the time and wonders if it is
really worth it. What would be the nurses best response?

A)

I can only imagine how you feel. Would you like to talk about it?

B)

Lets find a quiet spot and Ill teach you a few coping strategies.

C)

Thats the same way that most patients who have a chronic illness feel.

D)

Do you think that maybe you could be managing things more efficiently?
A)

I can only imagine how you feel. Would you like to talk about it?
A nurse at an allergy clinic is providing education for a patient starting immunotherapy
for the treatment of allergies. What education should the nurse prioritize?

A)
The importance of scheduling appointments for the same time each month

B)

The importance of keeping appointments for desensitization procedures

C)

The importance of avoiding antihistamines for the duration of treatment

D)

The importance of keeping a diary of reactions to the immunotherapy


B)

The importance of keeping appointments for desensitization procedures


A patient has presented with signs and symptoms that are consistent with contact
dermatitis. What aspect of care should the nurse prioritize when working with this
patient?

A)

Promoting adequate perfusion in affected regions

B)

Promoting safe use of topical antihistamines

C)

Identifying the offending agent, if possible

D)

Teaching the patient to safely use an EpiPen


C)

Identifying the offending agent, if possible


A patient was prescribed an oral antibiotic for the treatment of sinusitis. The patient has
now stopped, stating she developed a rash shortly after taking the first dose of the drug.
What is the nurses most appropriate response?

A)

Encourage the woman to continue with the medication while monitoring her skin
condition closely.

B)

Refer the woman to her primary care provider to have the medication changed.

C)

Arrange for the woman to go to the nearest emergency department.

D)

Encourage the woman to take an OTC antihistamine with each dose of the antibiotic.
B)

Refer the woman to her primary care provider to have the medication changed.
A patient has sought care, stating that she developed hives overnight. The nurses
inspection confirms the presence of urticaria. What type of allergic hypersensitivity
reaction has the patient developed?

A)

Type I

B)

Type II
C)

Type III

D)

Type IV
A)

Type I
The nurse is providing care for a patient who has a diagnosis of hereditary angioedema.
When planning this patients care, what nursing diagnosis should be prioritized?

A)

Risk for Infection Related to Skin Sloughing

B)

Risk for Acute Pain Related to Loss of Skin Integrity

C)

Risk for Impaired Skin Integrity Related to Cutaneous Lesions

D)

Risk for Impaired Gas Exchange Related to Airway Obstruction


D)

Risk for Impaired Gas Exchange Related to Airway Obstruction


A junior nursing student is having an observation day in the operating room. Early in the
day, the student tells the OR nurse that her eyes are swelling and she is having trouble
breathing. What should the nurse suspect?

A)
Cytotoxic reaction due to contact with the powder in the gloves

B)

Immune complex reaction due to contact with anesthetic gases

C)

Anaphylaxis due to a latex allergy

D)

Delayed reaction due to exposure to cleaning products


C)

Anaphylaxis due to a latex allergy


A nurse is caring for a patient who has allergic rhinitis. What intervention would be most
likely to help the patient meet the goal of improved breathing pattern?

A)

Teach the patient to take deep breaths and cough frequently.

B)

Use antihistamines daily throughout the year.

C)

Teach the patient to seek medical attention at the first sign of an allergic reaction.

D)

Modify the environment to reduce the severity of allergic symptoms.


D)

Modify the environment to reduce the severity of allergic symptoms.


The nurse is creating a care plan for a patient suffering from allergic rhinitis. Which of
the following outcomes should the nurse identify?

A)

Appropriate use of prophylactic antibiotics

B)

Safe injection of corticosteroids

C)

Improved skin integrity

D)

Improved coping with lifestyle modifications


D)

Improved coping with lifestyle modifications


A 5-year-old boy has been diagnosed with a severe food allergy. What is an important
parameter to address when educating the parents of this child about his allergy and
care?

A)

Wear a medical identification bracelet.

B)

Know how to use the antihistamine pen.

C)

Know how to give injections of lidocaine.

D)
Avoid live attenuated vaccinations.
A)

Wear a medical identification bracelet.


A patient is brought to the emergency department (ED) in a state of anaphylaxis. What
is the ED nurses priority for care?

A)

Monitor the patients level of consciousness.

B)

Protect the patients airway.

C)

Provide psychosocial support.

D)

Administer medications as ordered.


B)

Protect the patients airway.

Which factor indicates to the nurse the only similarity between discoid lupus
erythematosus (DLE) and systemic lupus erythematosus (SLE)?
a. Feeling tired and having a temperature that runs about 100° F (37.8° C) during the
day
b. Disfiguring and embarrassing rash
c. Peripheral neuropathies and cranial nerve palsies
d. High risk for renal inflammation
b. Disfiguring and embarrassing rash
Skin lesions associated with disfiguring and embarrassing rash are common to SLE and
DLE.
Fatigue and fever are common only to SLE. Neurologic manifestations and inflammation
of the kidneys are common in SLE.
The nurse assesses a client diagnosed with Sjögren's syndrome. The nurse anticipates
that the client will also have which common condition?
a. Dry eyes
b. Abdominal bloating after eating
c. Excessive production of saliva in the mouth
d. Intermittent episodes of diarrhea
a. Dry eyes
Clients with Sjögren's syndrome experience dry eyes
Before administering low-molecular-weight heparin (LMWH) to an older adult client after
total knee arthroplasty, the nurse notes that the client's platelet count is 50,000/mm3 (50
x 109/L). What action is most important for the nurse to take?
a. Notify the health care provider of the platelet count.
b. Administer the prescribed LMWH on schedule.
c. Assess the activated partial thromboplastin time (aPTT).
d. Assess the international normalized ratio (INR).
a. Notify the health care provider of the platelet count.
Normal platelet count is between 150,000 mm³ (150 x 109/L) and 400,000 mm³ (400 x
109/L), so 50,000 mm³ (50 x 109/L) is quite low. If the platelet count falls below
20,000/mm3 (20 x 109/L) spontaneous bleeding could occur. Notifying the health care
provider before the LMWH is given is essential.
Before administering prednisone IV push to a middle-aged adult with rheumatoid
arthritis (RA), the nurse notes that the client's random blood glucose level is 139 mg/dl
(7.7 mmol/L). Which action is most important for the nurse to take?
a. Instruct the client to drink diet soda to prevent elevation of blood sugar.
b. Administer the prescribed prednisone on schedule.
c. Notify the health care provider of the random blood glucose result.
d. Review the client's antinuclear antibody (ANA) level.
b. Administer the prescribed prednisone on schedule.

it is only slightly elevated so it just needs to be monitored during therapy.


When preparing a client for allergy testing, the nurse provides the client with which
instruction?
a. "Antihistamines should be discontinued 2 weeks before the test to avoid suppressing
the test response."
b. "It is okay to use your fluticasone propionate (Flonase) nasal spray before testing."
c. "Aspirin in a low dose may be taken before testing."
d. "You can take antihistamine nasal sprays before testing."
a. "Antihistamines should be discontinued 2 weeks before the test to avoid suppressing
the test response."
Assessment findings reveal that a client admitted to the hospital has a contact type I
hypersensitivity to latex. Which preventive nursing intervention is best in planning care
for this client?
a. Report the need for desensitization therapy.
b. Convey the need for pharmacologic therapy to the health care provider.
c. Communicate the need for avoidance therapy to the health care team.
d. Discuss symptomatic therapy with the health care provider.
c. Communicate the need for avoidance therapy to the health care team.
The nurse is reviewing discharge teaching with a client who suffered an anaphylactic
reaction to a bee sting. Which statement by the client indicates the need for further
teaching?
a. "I must wear a medical alert bracelet stating that I am allergic to bee stings."
b. "I need to carry epinephrine with me."
c. "My spouse must learn how to give me an injection."
d. "I am immune to bee stings now that I have had a reaction."
d. "I am immune to bee stings now that I have had a reaction."
An alert, middle-aged client is admitted to the emergency department with wheezing,
difficulty breathing, angioedema, blood pressure of 70/52 mm Hg, and apical pulse of
122 beats/min and irregular. The nurse makes an immediate assessment using the
"ABCs" for any client experiencing anaphylaxis. What nursing intervention is the
immediate priority?
a. Raise the lower extremities.
b. Start intravenous (IV) administration of normal saline.
c. Reassure the client that appropriate interventions are being instituted.
d. Apply oxygen using a high-flow non-rebreather mask at 40% to 60%.
d. Apply oxygen using a high-flow non-rebreather mask at 40% to 60%.
A client recently admitted to the hospital with a UTI is to receive the first dose of an
antibiotic intravenously. Before checking the five rights prior to administration, what is
the nurse's first action?
a. Review the clinical records and ask the client about any known allergies.
b. Check with the pharmacy for any known allergies for this client.
c. Check the client's identification band for any allergies.
d. Ask the nurse who previously cared for the client about any known allergies.
a. Review the clinical records and ask the client about any known allergies.
A client with a history of asthma is admitted to the clinic for allergy testing. During skin
testing, the client develops shortness of breath and stridor and becomes hypotensive.
What is the most appropriate drug for the nurse to give in this situation?
a. Epinephrine (Adrenalin)
b. Fexofenadine (Allegra)
c. Cromolyn sodium (Nasalcrom)
d. Zileuton (Zyflo)
a. Epinephrine (Adrenalin)
The nurse prepares to administer zafirlukast (Accolate) to a client with allergic rhinitis.
Zafirlukast works by which mechanism?
a. Blocking histamine from binding to receptors
b Preventing synthesis of mediators
c. Preventing mast cell membranes from opening
d. Blocking the leukotriene receptor
d. Blocking the leukotriene receptor
The nursing instructor asks the student nurse to explain a type IV hypersensitivity
reaction. Which statement by the student best describes type IV hypersensitivity?
a. "It is a reaction of immunoglobulin G (IgG) with the host cell membrane or antigen."
b. "The reaction of sensitized T cells with antigen and release of lymphokines activate
macrophages and induce inflammation."
c. "It results in release of mediators, especially histamine, because of the reaction of
immunoglobulin E (IgE) antibody on mast cells."
d. "An immune complex of antigen and antibodies is formed and deposited in the walls
of blood vessels."
b. "The reaction of sensitized T cells with antigen and release of lymphokines activate
macrophages and induce inflammation."
The nurse is assessing a client with suspected serum sickness. Which symptoms are
consistent with serum sickness?
a. Arthralgia
b. Blurred vision
c. Lymphadenopathy
d. Malaise
e. Ptosis
a, c, d
A client is being discharged from the hospital after an allergic reaction to environmental
airborne allergens. Which instruction is most important for the nurse to include in this
client's discharge teaching plan?
a. Wash fruits and vegetables with mild soap and water before eating.
b. Intermittent exposure to known allergens will produce immunity.
c. Remove cloth drapes, carpeting, and upholstered furniture.
d. Be cautious when eating unprocessed honey.
c. Remove cloth drapes, carpeting, and upholstered furniture.
Which nursing action is most appropriate for the nurse working in an allergy clinic to
delegate to a nursing assistant?
a. Plan the schedule for desensitization therapy for a client with allergies.
b. Monitor the client who has just received skin testing for signs of anaphylaxis.
c. Educate a client with a latex allergy about other substances with cross-sensitivity to
latex.
d. Remind the client to stay at the clinic for 30 minutes after receiving intradermal
allergy testing
d. Remind the client to stay at the clinic for 30 minutes after receiving intradermal
allergy testing
A client in the allergy clinic develops all of these clinical manifestations after receiving
an intradermal injection of an allergen. Which symptom requires the most immediate
action by the nurse?
a. Anxiety
b. Urticaria
c. Pruritus
d. stridor
d. stridor
The nurse plans to assess a client with type I hypersensitivity for which clinical
manifestation?
a. poison ivy
b. autoimmune hemolytic anemia
c. allergic asthma
d. rheumatoid arthritis
c. allergic asthma
The nurse is reviewing the medical record of a client who is prescribed a decongestant.
The nurse plans to contact the client's health care provider if the client has which
condition?
a. Cataracts
b. Crohn's disease
c. Diabetes mellitus
d. Hypertension
d. hypertension
A client is prescribed prednisone for treatment of a type I hypersensitivity reaction. The
nurse plans to monitor the client for which adverse effects?
a. Fluid retention
b. Gastric distress
c. Hypotension
d. Infection
e. Osteoporosis
a. b. d. e.
A client is admitted to the hospital with suspected Goodpasture's syndrome. Which
findings does the nurse expect to observe?
a. Bradycardia
b. Hemoptysis
c. Increased urine output
d. Weight gain
b. Hemoptysis
The nurse is caring for a middle-aged client diagnosed with rheumatoid arthritis. Which
client statement requires further assessment for unproductive coping strategies?
a. "I'm letting my husband do most of the cooking, but I help plan the menus."
b. "Since I started taking etanercept (Enbrel), I can walk up and down the stairs of my
home easier."
c. "My husband is getting used to having sex only once a month."
d. "I worry about what's going to happen to me if my husband cannot take care of me,
but he says he'll hire someone if he must."
c. "My husband is getting used to having sex only once a month."
A client diagnosed with rheumatoid arthritis (RA) is started on methotrexate
(Rheumatrex). Which statement made by the client indicates to the nurse that further
teaching is needed regarding drug therapy?
a. "Drinking alcoholic beverages should be avoided."
b. "The health care provider should be notified 3 months before a planned pregnancy."
c. "Any side effects of this drug will be mild."
d. "I will avoid any live vaccines."
c. "Any side effects of this drug will be mild."
The nurse is teaching a client about the difference between rheumatoid arthritis (RA)
and osteoarthritis (OA). Which statement by the client indicates a need for further
teaching?
a. "RA is inflammatory. OA is degenerative."
b. "The risk factors or causes of RA are probably autoimmune, whereas OA may be
caused by age, obesity, trauma, or occupation."
c. "The typical onset of RA is seen between 35 and 45 years of age, whereas the typical
onset of OA is seen in clients older than 60 years."
d. "The disease pattern of RA is usually unilateral and is seen in a single joint, whereas
OA is usually bilateral and symmetric, and is noted in multiple joints."
d. "The disease pattern of RA is usually unilateral and is seen in a single joint, whereas
OA is usually bilateral and symmetric, and is noted in multiple joints."

OA is unilateral and usually affects a single joint, whereas RA is bilateral and affects
multiple joints.
A client has symptoms of rheumatoid arthritis (RA). Which laboratory finding indicates to
the nurse that the client may have RA?
a. Total serum complement, 75 units/mL
b. Positive total antinuclear antibody (ANA)
c. Erythrocyte sedimentation rate (ESR), 20 mm/hr
d. Beta-globulin level, 1.0 g/dL (10 g/L)
b. Positive total antinuclear antibody (ANA)

Elevation of total ANA is common in systemic lupus erythematosus, systemic sclerosis,


and RA.
The nurse is reviewing the medication history for a client diagnosed with rheumatoid
arthritis (RA) who has been ordered to start sulfasalazine (Azulfidine) therapy. The
nurse plans to contact the health care provider if the client has which condition?
a. Glaucoma
b. Hypertension
c. Hypothyroidism
d. Sulfa allergy
d. Sulfa allergy
Which instructions for joint protection does the nurse recommend for a client with a
connective tissue disease?
Select all that apply.
a. Use long-handled devices such as a reacher.
b. When getting out of bed, use fingers to push off.
c. Sit in a low back chair.
d. Bend at the waist while keeping the back straight.
e. Use adaptive devices such as Velcro closures.
f. Turn a doorknob clockwise.
a, e
For clients with a connective tissue disease, the use of long-handled devices such as a
reacher and other adaptive devices, such as Velcro closures, helps to protect the joints.

autoimmune disease
is an immune response against the self
-the immune system will fail to differentiate between self and non-self, therefore attack
its own cells
causes of autoimmunity
-unknown
-age
-gender (more common in women)
-genetics
-environmental triggers such as viral infections and certain drugs
autoimmune classification
can be:
-organ specific
-systemic
organ specific
includes the blood, CNS, muscle, heart, endocrine, GI, kidney, liver, eye
systemic
includes: scleroderma, rheumatoid arthritis, systemic lupus
hemolytic anemia
-hematological autoimmune disorders
-autoimmune destruction of RBCs at a rate exceeding that which the body can replace
-results in severe anemia usually requiring blood transfusions, corticosteroids, and a
splenectomy
immune thrombocytopenic purpura
-hematological autoimmune disorder
-is the autoimmune tagging and sequestration of platelets in the spleen
-resulting in the shortened lifespan of platelets
-increased risk of bleeding
-may be acute or chronic
multiple sclerosis
-central nervous system autoimmune disease
-defined as the chronic-progressive, degenerative demyelinating disease of the CNS
-attacks the brain and spinal cord resulting in impaired transmission of nerve impulses
between brain and body
guillain barre
-central nervous system autoimmune disease
-occurs after viral/bacterial infection
-defined as the attack/demyelination of the peripheral nervous system (affecting
transmission of nerves)
-can cause pain, numbness/tingling, lack of reflexes, weakness of extremities
-respiratory muscles can be affected and may require mechanic ventilation
myanthesia gravis
-neuromuscular autoimmune disease
-defined as the attack of voluntary muscles
-characterized by fluctuating weakness of skeletal muscles
-muscles used to move eye/eyelids, chew, swallow, speak, breathe (eyes go first)
-muscles are strongest in the morning and fatigue with activity (so rest will restore
strength)
rheumatic fever
-cardiac autoimmune disease
-defined as the acute inflammatory disease of the heart
-complication of inadequately treated strep infection
-can cause permanent damage to the heart, including damaged heart valves and heart
failure
celiac disease
-GI autoimmune disease
-defined as the ingestion of gluten which leads to damage in the small intestine
-can cause foul smelling stool, diarrhea, flatus, abdominal distention, dermatitis,
herpetiformis
inflammatory bowel disease
-GI autoimmune disease
-defined as the chronic inflammation of the digestive tract such as ulcerative colitis, and
chron's disease
pernicious anemia
-GI autoimmune disease
-defined as when the body cannot absorb enough vitamin B2 (which is required to
produce healthy RBCs)
glomerulonephritis
-renal autoimmune disease
-defined as the inflammation of the glomeruli
-glomeruli normally function to remove excess fluid, electrolytes, and waste from the
bloodstream and pass them into the urine
-causes fluid retention and a decreased glomerular filtratio
-edema, htn, oliguria, hematuria, proteinuria
rhuematoid arthritis
-systemic autoimmune diseases
-characterized by the inflammation of connective tissue in synovial joints
-marked by periods of exacerbation and remission
-joints will become painful and stiff with limited mobility
-most common in the small joints of the hands and feet
-the inflammation will eventually lead to bone deformity
scleroderma
-systemic autoimmune disease
-defined as the disease of the connective tissue resulting in hardening of the skin
-CREST
CREST
c= calcinosis
Deposits of Ca2+ in skin (painful)
R= Raynaud's phenomenon
Vasospasm of fingertips in response to cold
E= Esophageal dysfunction
Difficulty swallowing d/t internal scarring
S= Sclerodactyly
Tightening of skin on fingers and toes
T= Telangiectasia
Red spots from capillary dilation
lupus
-systemic autoimmune disease
-defined as an autoimmune disease where the immune system of the body mistakenly
attacks healthy tissue
-is a chronic disease that affects multiple organs such as the skin, muscles, heart,
lungs, nervous tissue, kidneys
SLE cuases
-genetic
-hormones: often correlates with meses, pregnancy, oral contraceptives
-environmental: UV light exposures, stress, viruses, medications
SLE prevalence
-affects women 10 more times than men
-african americans, asians, latinas, native americans
-most often diagnosed in childbearing years (20-40)
SLE clinical manifestations
-tachypnea, cough, pleural inflammation/effusion
-weight loss, fatigue, fever, infection, arthritis, emotional liability
-hematological disorders ,neurological disorders, raynaud's phenomenon, pericarditis,
vascular inflammation
SLE lab tests
-increased ESR and CRP
-CBC
-BMP/CMP
SLE pharmacological interventions
-NSAIDs
-antimalarial drugs
-corticosteroids
-immunosuppressive drugs
-anticoagulants
SLE management
-plasmapheresis to remove blood plasma from the body in order to remove antibodies in
treating autoimmunity
-avoid sunlight
SLE patient teaching
-disease process
-drug actions, dosing, side effects
-pain management
-stress avoidance
-minimize exposure to Infection
-use of sunscreen
-lab follow ups
-pregnancy counseling
-support groups
treatment of autoimmunity
-aphresis
-platelet pheresis
-leukocytopheresis
-plamsapheresis
aphresis
procedure that separates and removes one or more blood components
platelets pheresis
removal of platelets
leukocytapheresis
remove white blood cells
pasmapheresis
removes plasma

accumulation of uric acid crystals in joints is what


gout
eating too much food that contains what can cause gout?
purine
which joint is most commonly affected by gout? how many joints is common?
big toe
less than 4
what can occur with chronic gout?
tophi
whats the uric acid level in gout?
>6
colsigine is given and if what happens, they are diagnosed with gout?
pain is relieved
what's another pain med given for gout?
NSAIDS
binimed is what?
a steroid injection
what foods are high inpurine? (6)
spinach, asparagus, shellfish, beeef, chicken, pork
what's lupus
chronic inflammation of multiple systems
cause of lupus?
unkonwn
how does their face look with people with lupus
butterfly rash
antiphospholipid syndrome is what and seen with what disorder
clots formed
lupus
how might their joints look in a person with lupus?
swan neck
specific lab diagnostic test for lupus?
Anti-DNA antibody and anti-Sm
what's another name for systemic sclerosis? what is it? cause?
sclerodema
skin thickening
too much collagen
what are some known triggers for scleroderma/
exposure to coal dust and plastics
Limited Cutaneous Disease Systemic Sclerosis affects what?
hands and face
Diffuse cutaneous disease Systemic Sclerosis affects what?
extremities and trunk
manifestations for systemic sclerosis?
Calcinosis (ca deposits)
Raynauds
Esophageal dysfunction
Sclerodactyly (tight skin)
Telangiectasia (red spots)
what are patients at risk for with calcinosis?
r/f infection
what labs do you want to look at for scleroderma?
scleroderma antibody
renal function (elevated BUN creatnine)
urinalsysi
Sjögren's Syndrome is what
anything that produces secretions is dry
treatment for Sjögren's Syndrome?
do things to keep hydrated and moist, like lotions, lubes, eye drops
what's fibromyalgia?
Wide-spread musculoskeletal pain and fatigue
people with fibromyalgia have multiple what?
tender points
diagnosis of fibromyalgia requires what?
11 of 18 tender points
pain for at least 3 months
what's flexural? used for what?
muscle relaxant
fibromyalga
what things should people with fibromyalgia avoid? why?
sugar, caffeine, alochol
irrates muscles
Innate Immunity
nonspecific --- that is, it is not directed against specific invaders but against any
pathogens that enter the body
Acquired Immunity
Able to recognize and destroy specific substances. The defensive reaction of the
adaptive immune system.
The immune response is a reaction of the adaptive immune system in defense of the
body.
Antigen or immunogen ( a toxin or enzyme in the invading microorganism) that the body
sees as foreign initiates this response
lymphocytes
White blood cells
Primary component of adaptive immune system
Antigen-presenting cells work with lymphocytes by trapping antigens and bringing them
to the attention of lymphocytes so that they can mount their attack.
Immunologic memory
When an antigen invades the body, normally only those lymphocytes with receptors that
fit the contours of that particular antigen take part in the immune response.
Daughter cells are generated that have receptors identical to those found on the original
lymphocytes
If the same type of antigen invades the body a second time, there are many more
lymphocytes specific for that antigen ready to meet the invader
Lymphocyte generation
"Lymphocytes are made from stem cells in the bone marrow
Lymphocytes then undergo a second stage of development, or processing, in which
they acquire their antigen-specific receptors.
Some lymphocytes are created with receptors that happen to be specific to normal,
healthy components of the body".
Where are B cells and T cells processed?
B Cells - bone marrow
T Cells - Thymus
B Cell and T cell function
B-cells involved in humoral immunity (takes place in body fluids)
T-cell immunity is known as cellular immunity as it takes place directly between T-cell
and antigen
When a B-cell is stimulated by an antigen that it encounters in the body fluids, it
transforms into a larger cell called a blast cell with the aid of a type of T-cell called a
helper T-cell, The blast cell begins to divide rapidly, forming a clone of identical cells
Complement
Works with both innate immunity and acquired humoral immunity as it can be triggered
by antibodies
It stimulates inflammation, facilitates antigen phagocytosis, and lyses some cells
directly. Because it is such a powerful inflammatory agent, its activity is tightly regulated
autoimmunity
A healthy immune system purges itself of the lymphocytes that are sensitized to self,
leaving only lymphocytes that ignore normal body components but react to foreign
intruders.
If this purging process is not completely successful, the result is an autoimmune
(literally "self-immune") disease in which the immune system attacks normal
components of the body as though they were foreign antigens, destroying healthy
molecules, cells, or tissues
Immune system malfunction
Antigen specific T-cells are activated→
Autoantibodies produced by B-cells that have been activated by Th2 cells→
Complement is activated→
Tissue is damaged/prolonged inflammation initiated→ Th1 cells activate macrophages
which promote inflammation and are cytotoxic
Specific disease initiation is not yet identified
Checks and balances
The function of regulatory T cells to see self-antigen based on the major
histocompatibility complex (MHC) and control potentially damaging autoreactive cells.
Type 1 diabetes Etiology
Genetic variants in HLA-DQA1, HLA-DQB1, and HLA-DRB1 genes.
Part of Human Leukocyte Antigen Complex gene family
This family of genes helps immune system identify "self" from "non-self" proteins
Changes in these genes is associated with an increased risk in the development of type
1 DM
Possible Triggers of DM1
Viral Infections
Vaccines
Low Vitamin D
Cow's milk
Increased insulin demand
Mechanism of Action DM1
T-cells attack the beta cells in islets of pancreas
Insulin production ↓ and BS ↑
Slow progressive diseases with relapses and remissions taking years to manifest
Due to excess islets of langerhans in human body more than ½ of the 1,000,000 islets
need to be destroyed before disease noticed
Thyroid Hormone Production
Thyroxine (T4)
80% of hormone produced by thyroid
Triiodothyronine (T3)
20% of hormone produced by thyroid
Has four times the hormone "strength" as T4
Grave's Disease
Thyroid specific (organ specific autoimmunity)
Antibodies to thyroid-stimulating hormone receptor (TSHR) created
Interferes with the body's ability to shut off TSH production to maintain normal thyroid
levels
Antibodies act as agonists, stimulating the release of thyroid hormone
Hyperthyroidism occur
Signs and Symptoms of Graves
Goiter
Tachycardia
Weight loss
Trembling
Ophthalmopathy
Pretibial myxedema

r/t both hyperthyroidism and autoimmunity


Signs and Symptoms of Graves
Goiter from inflammation and lymphocyte infiltration
Exacerbation of the autoimmune disorder by thyroid cells
Retro-orbital swelling from inflammation and lymphocyte infiltration
Apoptosis (cellular suicide without tissue damage) not regulated appropriately
Can be vertically transmitted from mother to baby as IgG antibodies cross the placenta
and attack the baby's thyroid
Risk Factors of Graves
Female
40-60 y/o
Caucasian/Asian
Traumatic event/stress often proceeds onset
Smoking (persistant ophthalmopathy)
Genetics
Low serum Vit D (genetic)
Labs Related to Graves' Disease
TSH: Decreased Level
Free T3: Increased Level
Free T4: Increased Level
TSI (thyroid stimulating immunoglobulin)
Antibody responsible for hyperthyroidism in Graves' disease
"Normal" patients do not produce any TSI therefore the presence of TSI indicates
Graves' Disease is present or imminent
Pharmacology for Graves
Restore normal thyroid function by compensating for the hyperthyroidism
Radioiodine treatment to destroy thyroid tissue
Anti-thyroid drugs successfully send the disease into remission in 30%-40% patients for
10 or more years
Radioactive iodine therapy
Thyroid cells require iodine to produce thyroid hormone
Radioactive iodine enters thyroid cells and destroys the overactive cells
Treatment takes time to be effective
Anti-thyroid drugs (thionamides)
Work by interfering with the thyroid's use of iodine to produce hormones.
When used alone it is possible to relapse.
Taking the drug for >1 year may give better results over time.
Anti-thyroid drugs may also be used in conjunction with radioiodine therapy
Beta blockers
Do not treat the high levels of thyroid hormone
Reduce symptoms related to hyperthyroidism
Tremors
Anxiety/Irritibility
Heat intolerance
Sweating
Dysrhythmias
diarrhea
Hashimoto's Thyroiditis
First described in 1912 by Dr. Hakaru Hashimoto
Most common cause of hypothyroidism in US
Genetic link
Associated with type 1 DM and celiac disease
Antibodies present to thyroid peroxidase in the thyroid gland
Inflammation develops destroying thyroid gland
Goiter d/t inflammation
Type 1 polyglandular autoimmune syndrome 1 (PGA I)
When Hashimoto's disease occurs with
Hypoparathyroidism
Adrenal insufficiency
Fungal infections of the mouth and nails
Type 2 polyglandular autoimmune syndrome (PGAII)
When Hashimoto's disease occurs with
Adrenal insufficiency
Type 1 diabetes
Signs and Symptoms of Hypothyroidism
Depression, modest weight gain, excessive, sleepiness, constipation, LE edema,
decreased concentration, aches and pains, fatigue, tendonitis, cold intolerance, dry,
coarse hair, dry skin, high cholesterol
Clinical Signs of Hashimoto's
Thyroid gland enlarged and symmetrical with firm, pebbly texture (90%)
Atrophy of thyroid gland (10%)
Lab diagnostics of Hashimotos
Free T4 test (low)
Serum TSH (high)
T3 (low or normal)
Thyroid autoantibodies are present 90%-95% of the time:
Antithyroid peroxidase antibody
Antithyroglobulin antibody
Late signs of Hashimoto's
Orbital edema
Bradycardia
Heart failure
Myxedema coma
Thyroid hormone replacement
Desiccated thyroid or thyroid extract (Armor Thyroid, Nature-Throid)
Natural thyroid hormone is derived from beef or pork.
Levothyroxine sodium (Synthroid, Levoxyl, Levothroid)
Synthetic thyroxine (T4)
Liothyronine sodium (Cytomel, Triostat)
Synthetic version of T3
Liotrix ( Thyrolar) is a synthetic thyroid hormone containing a mixture of T4and T3.
Rheumatoid Arthritis
Inflammatory form of arthritis that attacks synovium
Inflammation results
Swelling →pain→deformity
No cure
Etiology of RA
Autoimmune (IgG rheumatoid factor)
Synovium thickens following release of proteins in immune system attack
Cartilage, bone, tendons and ligaments damaged leading to loss of shape and
alignment and eventual destruction of joint
Risk Factors of RA
Women
Age 40-60 y/o
Family history
Smoking
Signs and Symptoms of RA
Joint pain
Joint swelling
Joint tender to touch
Red/edematous hands
Rheumatoid nodules
Fatigue
Morning stiffness X 30 min
Weight loss
Joint dysfunction
Difficulty with ADL's
Diagnositcs of RA
Elevated sedimentation rate
Presence of rheumtoid factor
Anti-cyclic citrullinated peptide (anti-CCP) antibodies
Joint fluid analysis
X-rays to determine amount of joint damage
Pharmacology of RA
NSAIDS
Steroids
Disease-modifying antirheumatic drugs (DMARDs).
Immunosuppressants
Biologics
DMARDs
slows the process of RA by blocking the inflammation
Hydroxychloroquine sulfate (Plaquenil, generic)
Comes in tablet only. Antimalarial drugs are commonly used to treat rheumatoid arthritis
and can help improve the skin lesions of lupus, and can hold off disease recurrence and
prevent organ damage. Serious side effects for antimalarial drugs are rare
Leflunomide (Arava, generic)
Comes as a pill taken once a day. People who cannot tolerate methotrexate may take
leflunomide. It can also be taken in combination with methotrexate.
Methotrexate (Rheumatrex, Trexall, Otrexup, Rasuvo, generic)
This drug is taken once a week and comes in tablet or as a self-injectable. It is for adults
with active RA and children with active juvenile idiopathic arthritis with more than one
affected joint.
Biologics (TNF-alpha inhibitors)
Injectable drugs
Block specific inflammatory pathways made by immune cells causing reduced
inflammation
Can reduce immune response
Added to DMARDS treatment if needed
Humira
Orencia
Remicade
Enbrel
Immunosuppressants
Increase risk of serious infection
cyclophosphamide (Cytoxan)
cyclosporine (Gengraf, Neoral, Sandimmune)
azathioprine (Azasan, Imuran)
Hydroxychloroquine (Plaquenil)
Surgical Treatment for RA
Total joint replacement
Tendon repair
Synovectomy
Patient Education in RA
Exercise
Healthy diet
Protect joints
Use assistive devices
Apply heat
Apply cold
Relax
Systemic Lupus Erythematosus (SLE) Risk Factors
Women
Age 14-45
Sunlight exposure
Long term use prescription certain Rx meds
Epsein barr virus
Exposure to mercury or silica
Primary Damage Systemic Lupus Erythematosus (SLE)
Joints
Skin
Kidneys
Heart
Lungs
Blood vessels
Brain
Complications of SLE
Kidney damage leading cause of death in lupus patients
CNS damage
Blood and blood vessels
Lungs
Heart
Infection
Cancer
Avascular necrosis
Pregnancy complications
Butterfly
Signs and Symptoms of Lupus
Fatigue
Fever
Weight loss or gain
Joint pain, stiffness and swelling
Butterfly-shaped rash (malar rash) on the face that covers the cheeks and bridge of the
nose
Skin lesions that appear or worsen with sun exposure
Mouth sores
Fingers and toes that turn white or blue when exposed to cold or during stressful
periods (Raynaud's phenomenon)
Shortness of breath
Chest pain
Dry eyes
Easy bruising
Anxiety
Depression
Memory loss
Diagnostics of Lupus
Requires 4 symptom:
Butterfly rash
Discoid rash
Sun related rash
Painless mouth sores
Joint pain>/= 2 joints
Swelling around lungs/heart
Kidney disease
Seizures/psychosis
Diagnostic Testing for Lupus
CBC
Sedimentation rate
Kidney/liver tests
UA
Antinuclear antibody
CXR
ECG
Syphilis test
Treatment for Lupus
NSAIDS
Antimalarial drugs -
improves life expectancy
Corticosteroids
Immunosuppressive drugs
DMARDS
No Advil for kidney problems
Azathioprine (Azasan, Imuran, generic)
Comes in tablet only; used most commonly in lupus. This drug depends on a specific
enzyme to work and some people lack enough of this enzyme to make the drug
effective. Your doctor will test your levels before prescribing azathioprine.
Cyclophosphamide (generic only)
Comes in capsule, tablet or infusion. Can be used in lupus in patients who do not
respond to traditional therapy or who experience kidney damage
Cyclosporine (Neoral, Gengraf, Sandimmune, generic)
Comes in capsule or syrup. This medicine is used sometimes for lupus in people who
do not respond to other therapies.
Patient Education with Lupus
Rest
Avoid sunlight
Exercise regularly
Don't smoke
Eat healthy diet

What is RA?
Inflammation of connective tissue within synovial joints
What are Rheumatoid Factors (RF)?
auto-antibodies we see that the body is attacking itself - abnormal antibodies
What are clinical manifestations of RA?
fatigue, anorexia, weight loss, generalized stiffness, low grade fevers

Warmth, inflammation, and sometimes redness at joints


When is pain the worst for RA patients?
in the morning, these patients need time in the morning - need to warm up their joints
and to get them up and moving
What are late manifestations of RA?
deformities, immobility
What is Sjorgen's syndrome (complication of RA)?
decrease salivary production, photo-sensitivity
What is Felty syndrome (complication of RA)?
very rare, enlarged spleen
What is the goal of care for RA patients?
Goal of care: trying to decrease pain, maintain their mobility and independence,
symptom management, nutrition
What type of exercise should RA patients do?
Lighter weights, swimming, nothing hard on the joints like pounding on the ground
What is Ankylosing Spondylitis (AS)?
Chronic, inflammatory disease affecting the axial skeleton
Clinical manifestations of AS?
sacroilitis & progressive, inflammatory arthritis of axial skeleton

Low back pain, stiffness, & limited motion


What makes AS pain feel better?
Pain improves with mild activity
What is Cauda Equina Syndrome (complication of AS)?
weakness in your lower extremities
What is important with AS?
Airway and breathing - patient becomes hunched over as the spine fuses improperly so
this can become an issue
What position should a person with AS be in?
Sitting up erect as possible
What is Bell's Palsy? What causes it?
facial nerve paralysis in one side of the face

They don't really know could be a virus


Clinical manifestation of Bell's Palsy?
inflammation, ischemia, & demyelination of facial nerve (7th CN) on one side of the face
What is Lyme Disease?
disease due to a bite of an infected deer tick
What is the first "classic" sign of Lyme Disease?
erythema migrans aka "Bull's eye rash"
What are late stage manifestations of lyme disease?
fatigue, memory/thinking problems, forgetfulness, confusion, difficulty with gait
What is lupus?
complex Inflammatory autoimmune disorder, affects multiple organ systems
What type of rash do you see with lupus?
Malar rash aka "Butterfly rash"
What can trigger lupus?
super sensitive to the sun, trigger exacerbation periods
How are you classified as having lupus?
You have to have at least 4 of the listed symptoms
What population does lupus most effect?
Women of childbearing ages
What is scleroderma?
is a chronic disease with skin changes causes thickening, hardening, or tightening of
the skin, blood vessels, and internal organs
What is localized scleroderma?
commonly on extremities
What is Systemic scleroderma?
Goes down through the skin and effecting the vessels and the organs
Limited vs Diffuse Scleroderma
Limited - thicker skin from the elbow down... reynauds symdrome appears later on

Defuse - everything happens all at once - worse than limited, reynauds happening at the
same time
What is Sclerodactyly?
tight taunt skin
What is fibromyalgia?
Widespread burning pain and tenderness
Clinical manifestations of fibromyalgia?
Chronic syndrome with widespread musculoskeletal pain & fatigue

Peripheral numbness and tingling due to all of the tenderness points happening
What do you need to have to be diagnosed with Fibromyalgia?
Pain in 11 of 18 tender points on palpation

History of Widespread pain for at least 3 months


Which of the following interventions is most important to teach a patient with
fibromyalgia?
Use stress management techniques such as meditation & deep breathing
During an assessment of a patient with scleroderma, which of the following assessment
findings would the nurse expect to find?
Thickening of the skin of the fingers and hands
What is Multiple Sclerosis (MS)?
chronic, progressive, degenerative disorder of the CNS "lesions"
S/S of MS?
fatigue, headaches, seizures..etc
Why do we do MRIs for MS? What qualifications do you need to be diagnosed with MS?
Want to identify the lesions
At least two inflammatory demyelinating lesions in at least 2 different areas in CNS
(present on MRI)

2 separate symptomatic events


The nurse is assessing a patient with (MS) who is experiencing an acute exacerbation.
Which of the following assessment findings would the nurse expect to find?
Motor impairment, visual disturbance, & paresthesias
What is Myasthenia Gravis (MG)?
Affects the neuromuscular junction causing weakness of skeletal muscles

Muscle weakness from the diaphragm up - in easier terms


What are clinical manifestations of MG?
weakness of eyes, eyelids, chewing, swallowing, speaking, and breathing all effected
How does their strength improve with MG?
strength improves with rest
What causes Exacerbations of Myasthenia Gravis?
Caused by emotional stress, pregnancy, menses, illnesses, temperature changes,
trauma, certain medications
What is Myasthenia Crisis?
Causes muscular weakness affecting normal swallowing and breathing
What is Cholinergic Crisis?
•Results from too much MG medications (Anticholinesterase drugs)

Think SLUGBAM (muscle twitching, sweating, secretions, bradycardia


What does IV Injection of an anticholinesterase agent (Tensilon) do for MG?
improves muscle contractility (Myasthenia Crisis)
If too much Tensilon is given what do you give?
When Tensilon is given it makes muscle weakness worse

Give Atropine!!
Where should the HOB be for a patient with MG?
Keep HOB elevated - risk for aspiration

Widespread inflammation
Autoimmune disorders are all similar due to their shared characteristic of... throughout
the body
Connective tissue

Synovial joints
(joints that allow for movement)
Rheumatoid Arthritis is the inflammation of... within the...
1. Chronic
2. Systemic
Rheumatoid Arthritis is a... (2) autoimmune disease
1. Exacerbation
2. Remission
Patients with autoimmune diseases will have periods of... (2)
Age
The incidence of Rheumatoid Arthritis increases with...
Symmetrical

Extra-articular
(inflammation outside the joints)
Rheumatoid Arthritis presents with inflammation of the joints that are ALWAYS...

Patients also present with... manifestations


Osteoarthritis is NOT autoimmune

(caused by wear and tear - athletes)


How is Rheumatoid Arthritis different from osteoarthritis?
Joint

Synovial joint

Massive inflammation
(VERY painful)
In Rheumatoid Arthritis, when triggered, abnormal immunoglobulins begin to attack
the...

This results in immune complexes forming in the... causing...


Genetic

1. Smoking
2. Stress
3. Infections
Rheumatoid Arthritis is a... disease that can also be caused by... (3)
Deformed
With Rheumatoid Arthritis, overtime, the joints and bones will become...
Morning

1. Take a hot shower


2. Put heat on their joints
When is Rheumatoid Arthritis pain at its worst?

Teach patients to... (2)


1. Fatigue
2. Weight loss
3. Low grade fevers
4. Pain/stiffness after inactivity
5. Warmth/inflammation at joints
6. Symmetrical swelling (synovitis)
S/S of Rheumatoid Arthritis include... (6)
Inactivity
Rheumatoid Arthritis pain gets WORSE with...
1. Slow
2. Progressive

Flu-like

1. Joint pain
2. Inflammation
Rheumatoid Arthritis is a... (2) disease that usually starts initially with... symptoms and
eventually... (2)
Rheumatoid Arthritis
Ulnar drift, Hallux valgus, Boutonniere and swan neck deformity are all seen in...
1. ADL performance
2. Nutrition (too much pain to feed)
3. Pain
4. Safety
Concerns as a nurse for a patient with Rheumatoid Arthritis include... (4)
Dry
(ex. eyes, skin, mouth)
Sjogren's syndrome is an autoimmune disorder where everything is...
Cold (ice)

Warm
During an Rheumatoid Arthritis exacerbation, use... therapy

In the morning use... therapy


Swimming
The best exercise for a patient with Rheumatoid Arthritis would be...
Lower back
(sacral ileac area)

Up
Ankylosing Spondylitis is a chronic, inflammatory disease that starts in the... and works
its way...
Spinal bones

1. "bamboo spines"
2. "RA of the spine"
In Ankylosing Spondylitis,... start to fuse together

This is sometimes called... (2)


Genetic
Ankylosing Spondylitis is a... disorder
1. Sacroilitis
2. Lower back pain
3. Stiffness
4. Weight loss
Clinical manifestations of Ankylosing Spondylitis include... (4)
At rest (morning)

Mild activity
When is Ankylosing Spondylitis pain worse?

How does the pain improve?


Ankylosing Spondylitis
Sacroilitis is seen in which condition?
1. Postural deformities
2. Cauda Equina syndrome*
Complications of Ankylosing Spondylitis include... (2)
Medical emergency

Lower extremity weakness

Bowel/bladder incontinence
Cauda Equina syndrome is considered a... that consists of bilateral... and...
Compression

Nerve root

Paralysis
Cauda Equina syndrome is caused due to major... over the... over the base of the spine

This can lead to permanent...


Emergency surgery to relieve the nerve root
How is Cauda Equina syndrome fixed?
Kyphosis
Safety concerns with Ankylosing Spondylitis involve monitoring the fixed position of the
patients spine which is called...
Arthralgia
Generalized joint pain is called...
Posture

1. Respiratory
2. Mobility
Interventions for Ankylosing Spondylitis involve correcting... and maintaining... (2)
function
Supine with a pillow under their knees

(keeps the spine aligned)


What is the best position for a patient with Ankylosing Spondylitis to sleep in?
Swimming
(hydrotherapy)
What type of exercise is the best for a patient with Ankylosing Spondylitis?
Paralysis

Facial nerve (CN7)

Viral infections
(ex. Herpes)
Bell's Palsy is... of the... caused by...
7 (CN)

ONE SIDE
Bell's Palsy involves inflammation, ischemia and demyelination of facial nerve... that
affects... of the face
Stroke

Trigger (virus)
Patients with Bell's Palsy may appear as though they are having a...

This is caused by a...


1. Sensory
2. Motor
Patients with Bell's Palsy experience pain with alterations in... (2) functions
Eye patch
If patients with Bell's Palsy cannot close their eye due to muscle weakness, teach them
to wear an...
It resolves on its own

1. Moist heat
2. Gentle massage
How is Bell's Palsy go away?

How can you help treat bells palsy? (2)


Infected deer tick
How it Lyme disease transmitted?
1. MS
2. Mono
3. Meningitis
Lyme disease clinical manifestations can mimic other diseases such as... (3)
Erythema migrans*
"bull's eye rash"

(presents 3-30 days after tick bite)


The first classic symptom of Lyme disease is... which is called...
1. "bull's eye rash"
2. Flu-like symptoms
3. Facial paralysis (facial droop)
4. Swollen joints (knee)
Clinical manifestations of Lyme disease include... (4)
1. Heart
2. Joints
3. CNS
If Lyme disease is not treated, what can become affected? (3)
1. Acute
2. Chronic
Lyme disease can be... (2)
Neurological

1. Fatigue
2. Memory/thinking issues
3. Gait difficulty
LATE clinical manifestations of Lyme disease include... presentations such as... (3)
2 step testing

1. Enzyme immunoassay (EIA, ELISA) - (NONSPECIFIC)


2. Western blot IgG, IgM
(to CONFIRM)
The CDC recommends... to diagnose Lyme disease

What are they called? (2)


Doxycycline (Vibramycin)
What is the drug of choice to treat Lyme disease?
Unpredictable
Lupus is a complex, chronic disorder that is... in nature
1. Genetic
2. Hormones
3. Environmental factors (ex. sun)
What are 3 suspected big triggers of Lupus?
Antibody

1. Serum
2. Tissue
Lupus is an overaggressive... response where immune complexes form in the... (2)
Women

Child bearing years


Lupus is most common in... of...
1. Butterfly rash*
2. Photosensitivity
3. Flu-like symptoms
4. Inflammation in areas of the body (-itis)
Clinical manifestations of lupus include... (4)
1. Gluten
2. Sugar
3. Dairy
4. Alcohol
Teach patients with Lupus to avoid inflammatory foods such as... (4)
Exacerbation

1. Miscarriage
2. Birth deformities
Pregnancy in a patient with Lupus can lead to severe...

There is a high risk for... (2)


Collagen

1. Tissue fibrosis
2. Blood vessel occlusion
Scleroderma is an overproduction of... resulting in... (2)
1. Genetic
2. Environmental
What is the cause of Scleroderma? (2)
1. Hardening/thickening of skin
2. "CREST" syndrome
Clinical manifestations of Scleroderma include... (2)
1. Localized
2. Systemic
What are the 2 types of Scleroderma?
Systemic

Calcinosis
Raynaud's
Esophageal dysfunction
Sclerodactyly
Telangiectasia
"CREST" syndrome involves... symptoms

CREST stands for...


1. Shiny and taut skin
2. Arthralgias
3. Joint contractures (stuck)
4. "CREST" syndrome
Clinical manifestations of Scleroderma include... (4)
To prevent Raynaud's
For a patient with Scleroderma, why should we teach them to protect their hands and
feet from cold exposure?
Musculoskeletal
Fibromyalgia is a chronic syndrome with widespread... pain and fatigue
1. Widespread burning pain and tenderness
2. Difficulty concentrating/memory
3. Peripheral numbness/tingling
Clinical manifestations of Fibromyalgia include... (3)
11

18

3 months
For diagnosis of Fibromyalgia, the patient must be tender inattentive least... out of...
points

This must be present for... or longer


Fibromyalgia
Which disease is diagnosed by the presence of tenderness in 11 out of 18 pressure
points?
1. Pain in 11/18 tender points
2. Widespread pain for 3 months
Which 2 main criteria must be met to diagnose Fibromyalgia?
1. Massage with ultrasound
2. Restrict inflammatory foods
3. Relaxation techniques
(stress management, cognitive training)
How is Fibromyalgia treated/managed? (3)
Triggers

Stress factors
Fibromyalgia involves many... and...
Degenerative

CNS

1. Brain
2. Spinal cord
Multiple Sclerosis is a chronic, progressive and... disorder of the... characterized by
disseminated demyelination of the nerve fibers of the... (2)
Terminal (no known cure)

Unknown
Autoimmune
Multiple Sclerosis is a... disease

The cause is..., but is believed to be...


1. Inflammation
2. Demyelination
3. Gliosis/sclerosis
(scarring/plaques)
What are the 3 pathologic processes within the CNS in Multiple Sclerosis?
Different locations on the spinal cord are affected
Why is the presentation of Multiple Sclerosis different with each patient?
Neurologic (CNS)
Multiple Sclerosis is chronic, progressive deterioration of... function
Improve

Exacerbations
In Multiple Sclerosis, a patients symptoms will... with pregnancy

There is a high risk for severe... after birth


1. Motor
2. Sensory
3. Cerebellar
4. Cognitive/emotional
Clinical manifestations of Multiple Sclerosis include... (4) symptoms
Plaques/scarring
An MRI of the brain and spine on a patient with Multiple Sclerosis will show the location
of...
1. At least 2 inflammatory demyelinating lesions in at least 2 areas in CNS (seen in
MRI)
2. 2 separate symptomatic events
3. Rule out other diseases
The 3 diagnostic criteria of Multiple Sclerosis include...
No cure (terminal)

Symptom relief
What is the cure for Multiple Sclerosis?

What is the treatment goal?


Skeletal muscles

ACh receptors

Muscle weakness
Myasthenia Gravis affects the NM junction causing weakness of...

Antibodies attack... causing...


Muscles

Muscle weakness
ACh makes... move

Decreased ACh seen in Myasthenia Gravis presents with...


Morning
When are patients with Myasthenia Gravis at their best?
Use

Rest
Myasthenia Gravis weakness gets worse with... and stronger with...
Diaphragm
Myasthenia Gravis involves fluctuating weakness of skeletal muscles from the... and up
UNDER medicated

LOW

DRY
A patient experiencing a myasthenia crisis is...

ACh levels are...

These patients are...


OVER medicated
(too many anticholinergic drugs)

HIGH

WET
(SLUGBAM)
A patient experiencing a cholinergic crisis is...

ACh levels are...

These patients are...


1. Swallowing
2. Breathing

Aspiration
A myasthenia crisis causes muscular weakness affecting normal... (2)

These patients are at risk for...


Tensilon test
Which test is performed to diagnose between a Myasthenia vs cholinergic crisis?
ACh
Tensilon is a short-acting... inhibitor
Myesthenia crisis

UNDER medicated
If the patient gets BETTER with tensilon, they were experiencing a... which means they
were...
Cholinergic crisis

OVER medicated

GIVE ATROPINE
If the patient gets WORSE (weaker) with tensilon, they were experiencing a... which
means they were...

What should you do?


Muscle contractility
When we are administering Tensilon, we are looking for improvement/worsening of...
1. Respiratory support
2. Atropine
Before performing a tensilon test, make sure there is... (2) at the bedside
Anticholinergic/cholinergic
When treating Myasthenia Gravis, which drugs should be avoided?
1. C-reactive protein (CRP)
2. ESR

(nonspecific)
What are examples of 2 inflammatory markers?

Diseases in which the body see's itself as nonself and attacks


Autoimmune diseases
Examples of autoimmune diseases (6)
Lupus
Gillian-Barre
Multiple Sclerosis
Rheumatoid Arthiritis
Scleroderma
Myasthenia Gravis
Results of autoimmune diseases?
Tissue damage
Inflammation
Pain
Chronic, systemic autoimmune diseases that results in inflammation and eventually
destorys the joint tissues
Rheumatoid Arthiritis
What population does RA affect?
Women more than men, ages 25-50
What increases the risk of developing RA?
Smoking
Patho of RA
Neutrophils accumulate in the synovial fluid of the joints.
Toxins are released and break down the tissue which then gets replaced with granular
tissues
Synovial fluid begins to thicken from attacks
Accumulation of tough tissue in the joiunts that cases joint deformation
Pannus
Symtpoms of first onset of RA
Flu like symptoms-
1.Fatigue
2.Malaise
3.Total body achyness
4.Weight loss
5.Low grade fever
What symptoms of RA kick in after the flu like symptoms resolve?
Joint swelling, discomfort, and stiffness
What joints does RA affect?
Hands
Wrists
Knees
Ankles
Toes
When do you begin to feel the pain of RA?
Upon walking and then resting, lasts up to 1-2hrs after
What are the two specific markers shown in labs when testing for RA?
Rheumatoid Factor
Anti CCP
What labs are generally drawn to determine RA?
CRP
ESR
RF
Anti CCP
Anemia factors
RA treatment that helps removal of fluid from the joint
Anthrocentesis
What is the goal in management of RA?
Increase/maintain function - avoid decreases in function or disablement
How do you pharmacologically manage RA?
1. NSAIDS-esp Aspirin
2. DMARDs
3.Corticosteroids
4.Immunsuppressants
What are DMARDS?
Disease modification Anti-Rheumatic Drugs
What are 2 examples of DMARDs?
Methotrexate
Aslavadine
How do corticosteroids help RA patients?
Help with acute flair ups, keep on low dose
When shouldn't you give corticosteroids to RA patients and why?
With immunsoppressants, will open up to opputunistic infections and depress the
immune system
What are the 8 main goals for caring for an RA patient?
1.Relief of pain and discomfort
2.Relief of fatigue
3.Promotion of restorative sleep
4.Increased mobility
5.Maintainence of self care
6.Improved body image
7.Effective coping
8.Absence of complications
How could being sedentary further affect an RA patient?
Could cause UTI and speed up joint degeneration
Disease that generates an immune attack on the myelanted fibers of the nerves, CNS,
brain, optic nerves
Multiple Sclerosis
What is the onset of MS and what population does it affect?
20-40 y/o, women more than men
What happens to the nerve impulses in a patient with MS?
Lack of myelin short circuits the nerve signals
What are the signs and symptoms of MS? (4)
1.Changes in vision (double/blurred vision, loss of visual field)
2.Weakness, gait issues
3.Parastesias- numbness in extremeities
4.Cooridination-loss of balance
What are the 3 types of MS?
1.Relapsing-Remitting
2.Secondary-Progressive
3.Primary-Progressive
Most common form of MS that is characterized by episodes of acute exacerbaions
followed by going back to semi-normal baseline state
Relapsing-Remitting
Type of MS in which the patient starts with relapsing and remitting RR and then
transitions into this type, downward progression in which patient begins to lose function
as oppose to relapsing
Secondary-progressive
Very rare form of MS that only affects about 10% of diagnosed people, steadydecline
from time of diagnosis with no period of relapse or remission
Primary-progressive
How would you diagnose a patient with MS? (5)
1.History and Physical
2.Neurological Exam
3.MRI
4.Visual Evoked Potential (VEP)
5.Cerebrospinal Fluid analaysis
What is the most common complaint of MS?
Fatigue
What exacerbates fatigue in a patient with MS?
Heat
What kind of pain to MS patient experience?
Sharp/electric shock down the spinal chord, pins and needles, muscle spasms
What are 5 complications associated with MS?
1.Pain
2.Ataxia
3.Emotional lability
4.Bowel and bladder issues
5.Immobility complications
What kind of emotional issues to people with MS experience?
Depression to euphoria
What kind of bladder and bowel complications to people with MS have?
UTI's
Continence issues
What are some immobility complications associated with MS?
Contractures
DVT
Pressure Ulcers
_____________ includes gait disturbances and balance issues associated with MS
Ataxia
What are the 5 immune-modulating therapies for MS?
Avonex
Betaseron
Copaxone
Rebif
Tysabri
Autoimmune disorder affecting the myoneural junction that affects motor function,
antibodies in thymus form on acetlycholine and block it, impairing the transmission of
impulses
Myasthenia Gravis
Where does myesthenias Gravis start?
Eyes- drooping eyelids and blurred vision
How does myesthenia gravis progress?
Goes from the eyes to the facial muscles causing weakness, progresses to the larynx
which causes hoarsness of the voice and muscle weakness of the throat, weakness
then progresses throughout the entire body- affects the intercostal muscles last which
leads to respiratory failure

Methotrexate
It can treat cancer of the blood, bone, lung, breast, head, and neck. It can also treat
rheumatoid arthritis and psoriasis.
Trade Name(s)
Metoject [Canada]
Otrexup
Rasuvo
Rheumatrex
Trexall
CAN CAUSE HEPATIC FIBROSIS AND CIRRHOSIS, BONE MARROW
SUPRESSION, ULCERATIVE STOMATITIS AND RENAL DAMAGE
Plaquenil/hydroxychloroquine
It can treat and prevent malaria. It can also treat lupus and arthritis.
Colchicine
It can treat and prevent gout attacks. It can also treat familial Mediterranean fever
(FMF).
Trade Name(s)
Colcrys
Mitigare
Allopurinol
It can treat gout and kidney stones.
Trade Name(s)
Aloprim
Lopurin
Zyloprim
Uloric (Febuxostat)
It can treat high uric acid in the blood of patients with gout.
Antihistamines
Antihistamines are drugs which treat allergic rhinitis and other allergies. Antihistamines
can give relief when a person has nasal congestion, sneezing, or hives because of
pollen, dust mites, or animal allergy.
Decongestants
A decongestant, or nasal decongestant, is a type of pharmaceutical drug that is used to
relieve nasal congestion in the upper respiratory tract. The active ingredient in most
decongestants is either pseudoephedrine or phenylephrine.
Epinephrine
Blood pressure support and vasoconstrictor
It can treat severe asthma attacks and allergic reactions (including anaphylaxis) in an
emergency situation.
Isoproterenol
Blood pressure support
It can treat certain types of heart problems. It can also improve breathing while a patient
is under anesthesia.
Ephedrine
Involuntary nervous system stimulant
It can treat low blood pressure caused by other medications. It can also improve
breathing.
Corticosteriods
Corticosteroids are a class of steroid hormones that are produced in the adrenal cortex
of vertebrates, as well as the synthetic analogues of these hormones.
Vasopressors
are a powerful class of drugs that induce vasoconstriction and thereby elevate mean
arterial pressure (MAP).
Things that contain latex
ACE bandages, Elastic/adhesive bandages, Ambu bag, Balloons, BP cuff tubing,
catheters, gloves, feminine hygiene pads, IV catheters, tubing, rubber injection ports
and syringes.
Anaphylaxis
Whole body: fainting, lightheadedness, low blood pressure, dizziness, or flushing
Respiratory: difficulty breathing, rapid breathing, shortness of breath, or wheezing
Skin: hives, swelling under the skin, blue skin from poor circulation, or rashes
Gastrointestinal: nausea or vomiting
Also common: fast heart rate, feeling of impending doom, itching, tongue swelling,
difficulty swallowing, facial swelling, mental confusion, nasal congestion, or impaired
voice
Stages of lyme disease
1, 2, 3
Stage 1 lyme disease
Several days to months following the bite, small red pimples develops that may spread
into ring shaped rash. Flu like symptoms occur, stiff neck, muscle aches and fatigue
Stage 2 lyme disease
This stage occurs several weeks following bite, joint pain occurs, neurological
complications occur and cardiac complications occur
Stage 3 lyme disease
Large joints become involves arthritis progresses
Arthroscopy
visualization of internal structures of a joint , infection and lack of joint mobility are
contraindications
Nuclear scans
evaluates entire skeletal system, radioactive isotope injected 2-3 hours prior, less
common due to MRI's
Gallium and thallium scan
A gallium scan is a test to look for swelling (inflammation), infection, or cancer in the
body. It uses a radioactive material called gallium and is a type of nuclear medicine
exam.
A thallium stress test is a nuclear imaging test that shows how well blood flows into your
heart while you're exercising or at rest. This test is also called a cardiac or nuclear
stress test. ... The radioisotope will flow through your bloodstream and end up in your
heart.
Nuclear scan preprocedure considerations
Informed consent, assess for allergy, pregnancy or kidney disease
DXA scan
is a special type of X-ray that measures bone mineral density (BMD). DEXA stands for
"dual energy X-ray absorptiometry". This type of scan may also be called: a DXA scan.
a bone density scan. Remove metallic objects prior..
EMG
measures the electrical activity of muscles when they're at rest and when they're being
used. Nerve conduction studies measure how well and how fast the nerves can send
electrical signals. Nerves control the muscles in the body with electrical signals called
impulses.
EMG post procedure
client can apply ice to prevent hematomas at needle insertion site and reduce swelling
use a warm compress for discomfort.
Osteoarthritis disease process
Cartilage destruction with bone spur growth at joint ends (degenerative)
Osteoarthritis findings
Pain with activity that improves w/ rest
Osteoarthritis effusions
localized inflammatory response
Osteoarthritis body size
usually overweight
Osteoarthritis system involvment
none, localized
Osteoarthritis nodes
heberdens and bouchards nodes
Osteoarthritis symetrical
No
Osteoarthritis diagnostic tests
Xray
RA disease process
Synovial membrane inflammation in cartilage destruction and bone erosion
inflammatory
RA findings
selling, redness, warmth, pain at rest after immobility
RA effusions
All joints
RA nodes
Swan neck and boutonniere of hands
RA body size
usually underweight
RA systemic
Yes, lungs, heart, skin and extra-articular
RA Symmetrical?
Yes
RA diagnostic tests
Xrays positive and Rheumatoid factor
Osteoarthritis health promotion
Encourage weight loss if needed, avoid repetitive strain like jogging or contact sports,
recommend wearing well fitting shoes.
Osteoarthritis risk factors
Aging
genetic
joint injury
obesity
metabolic disorders (DM, Sickle cell disease)
Osteoarthritis expected findings
Joint pain/stiffness
Pain on joint palpation or ROM
Crepitus
Enlarged joint
Bouchards nodes
joint effusion
vertebral radiating pain
limping gait
back pain
OA diagnostic procedures
Xrays, MRI, CT, NBS, CT myelogram, EMG, Arthrogram
Arthrogram
xray study of a joint
Patient assessment for OA
Pain monitoring, musculoskeletal assessment/muscle tone, Neuro/sensory ans
psychosocial assessment
Maximum administration of acetominophen
4,000mg/24 hr
NSAID side effects
black tarry stool, indigestion and SOB
Muscle relaxants can cause what in older adults?
Confusion
Tramadol use in OA?
Considered a weak opioid that can be use for short term management of OA and long-
term use in lower back pain
Glucosamine supplements
is a compound that is naturally made in humans. It is involved in the creation of
molecules that form cartilage.
Interprofessional care for OA
PT, nutritionist and occupational therapy
Total joint arthroscopy
is a surgical procedure to replace the weight-bearing surfaces of the knee joint to relieve
pain and disability. It is most commonly performed for osteoarthritis,[1] and also for
other knee diseases such as rheumatoid arthritis and psoriatic arthritis. In patients with
severe deformity from advanced rheumatoid arthritis, trauma, or long-standing
osteoarthritis, the surgery may be more complicated and carry higher risk. Osteoporosis
does not typically cause knee pain, deformity, or inflammation and is not a reason to
perform knee replacement.

Open diskectomy
removal of herniated disk
Laminectomy
Removal of part of the laminae and facet of joints
HIV infection stages
Stage 1-4
Stage 1
CD4: 500 cells/mm3 or more
%: 29
Stage 2
CD4: 200-499 cells/mm3
%: 14-28
Stage 3 AIDS
CD4: 200 or less
%:14
or one or more opportune infection
HIV manifestation can occur as early as
2-4 weeks after infection(flu like symptoms)
AIDS death usually occurs within....
5 years
HIV encourage
hygiene, frequent immunizations and drug regimen be followed
HIV expected findings
Chills, rash, anorexia, weight loss, nausea, weakness and fatigue, headache and sore
throat and night sweats
Diagnostic tests for HIV
Viral load test, HIV genotype or HIV tropism, Liver profile, biopsies and testing of stool
for parasites and brain or lung MRI or CT scan.
Neoplasms
Kaposi's sarcoma purplish-red lesions of internal organs and skin, B cell and non-
hodgkins lymphoma cervical cancer
Fusion inhibitors
enfuvirtide blocks fusion of HIV host in cell
Entry inhibitors
Maraviroc
Nucleoside reverse transcriptase inhibitors
Zidovudine interferes with the virus' ability to convert RNA into DNA
Non-nucleoside reverse transcriptase inhibitors
Delavirdine and efavirenz inhitbit viral replication in cells
Protease inhibitors
Atazanavir, nelfinavir, saquinavir and indinavir inhibit enzym needed for virus to
replicate
Integrase inhibitors
Raltegravir
Antineoplastic medication
Interleukin is an immunostimulant that enhances the immune response and reduces the
production of cancer cells used commonly with Kaposis sarcoma.
HIV interprofessional care
Infectious disease, resp. services, nutritional services, rehab services, support groups,
home health, longterm care and hospice.
Opportunistic bacterial diseases
TB, bacterial pneumonia, septicemia
Opportunistic malignancies
Kaposis sarcoma, lymphoma, abd squamous cell carcinoma
Opportunistic viral diseases
cytomeglovirus, herpes simplex and herpes zoster
Opportunistic fungal diseases
PCP(pnemonia) candidasis, cryptospoidosis, and penicilliosis
Opportunistic protazoal diseases
OCO, toxoplasmosis, microsporidosis, cryptosporidosis isoporiasis and leishmaniasis
Wasting syndrome
A general state of ill health involving marked weight loss and muscle loss.
HIV F and E
encourage 2,000ml-3,000ml of fluid intake
HIV encephalopathy
Seizure precautions
Arthrodesis/spinal fusion
fusion of two or more vertebras
DLE(Discoid lupus erythmatosus)
only effects the skin
Systemic lupus (SLE)
affects connective tissue of multiple organ systems and can lead to major organ failure
Medication induced (LE)
can be caused by procainamide, hydralizine, isoniazid. Resolved after medication is d/c
Lupus risk factors
women b/w 20-40
African american, asian and native americans
incidence declines in women who are post menopausal but remains steady in men
Can be harder to notice in adults b/c symptoms mimic normal aging process
Lupus expected findings
Fatigue
Alopecia
Blurred vision
pleuritic pain
anorexia/weight loss
Lupus physical assessment findings
Fever
Anemia
Lymphendopathy
Pericarditis
Raynauds phenomenon
Organ involvement
Raised dry scaly rash over nose and cheeks
Alopecia
Lupus lab tests
ANA antibody, serum complement c3 and c4 decrease, ESR elevated due to
inflammation, BUN and creatinine increase, Urinalysis + for protein and RBC's, CBC
pancytopenia (eduction in the number of red and white blood cells, as well as platelets)
Nursing care lupus
Assess and monitor pain, mobility and fatigue
HTN and edema
Urine output
Diminished breath sounds
Tachycardia
Rubor, pallor or cyanosis of hands and feet
Joint and connective tissue problems
Change in MS
BUN/ Creatinine and urinary output for renal involvement
Nutritional status
Encourage limit of salt intake for fluid retention secondary to steroid therapy
Lupus medication therapy
NSAID, Corticosteroids, immunosuppressant agents, antimalarial(hydroxychloroquine
used for suppression of synovitis, fever and fatigue)
Complications of lupus
lupus nephritis, pericardiatis and myocarditis
Gout
The most common inflammatory arthritis. Caused by disruption of purine metabolism in
which uric crystals are deposited in joints and body tissues can be primary or secondary
Primary gout
Most common uric acid production is greater than excretion of it by kidneys, genetic
component middle and older adults males ages 40 to 50 as well as post menopausal
Secondary gout
Caused by another disease or condition chronic kidney, failure, excessive diuretic use
of uric acid in blood. Treatment is based on treating the underlying condition can affect
people of any age
Assessment
Obesity
Cardiovasular disease
Trauma
Alchol ingestion
Starvation diet
Diuretic use
Some chemo agents
Chronic kidney failure
Gout expected findings
Severe joint pain in great toes (metatarsophalangeal) redness, swelling, and warmth of
affected joint(painful if touched) or chronic gout
Gout lab test
ESR: elevated
Serum uric acid: 6.5mg above
Urinary uric acid: elevated
BUN and creatinine: elevated
Acute gout medication
Antigout, NSAIDS and corticosteroids
Antigout agent
*Colchicine* (PO or parenteral) is used to decrease pain and inflammation. Use
cautiously on kidney patients
NSAIDS
Indomethacin or ibuprofen is used to decrease pain and inflammation
Corticosteroids
Prednisone used for inflammation, monitor for fluid retention, HTN and impaired kidney
function and do not take on empty stomach
Chronic gout medication
Xanthine oxidase inhibitor and uricosuric
Xanthine oxidase inhibitor
Allopurinal is used as maintenance medication to promote uric acid excretion and
decreased its production
increase fluid intake
Take after meals on a full stomach with a glass of water
Uricosuric
Probenecid is used as a maintenance medication to promote uric acid excretion.
Monitor uric acid levels. Do not use aspirin b/c it will decrease the effectiveness of
meds.
Fibromyalgia
Is often accompanied by fatigue and altered sleep, memory, and mood.
Widespread muscle pain and tenderness are the most common symptoms.
Medications, talk therapy, and stress reduction may help control symptoms.
Fibromyalgia risk factors
Females between 30-50
History of rhematologic conditions chronic fatigue syndrome or lyme disease
deep sleep deprivation
Fibromyalgia medications
Serotonin-norepinephrine re uptake inhibitors (SNRIs) and anticonvulsants
NSAIDS
Try cyclic antidepressants
Fibromyalgia interprofessional care
PT, support groups and alternative therapy (acupuncture and stress management)
Rheumatoid Arthritis
A chronic inflammatory disorder affecting many joints, including those in the hands and
feet.
Rheumatoid Arthritis Risk Factors
Females gender 3:1
Age 20-50
Genetic predisposition
Epstein-Barr virus
Stress
Environmental factors
Older age
Rheumatoid Arthritis Expected Findings
Pain at rest and with movement
Morning stiffness
Pleuritic pain upon inspiration
Dry mouth
Anorexia/wgt. loss
Fatigue
Paresthesias
Recent illness/stressor
Joint swelling
Subq nodules
Fever(low grade)
Muscle weakness/atrophy
Reddened sclera or abnormal pupils
Lymph node enlargement
RA lab tests
Anti-CCP antibodies
RA factor antibody
ESR
C-reactive positive
ANA titer
Elevated WBC
RA diagnostic procedures
Arthrocentesis
XRAY
RA nursing care
Apply heat and cold to the affected areas indicated based on client response
Assist w/ physical activity to maintain joint mobility
Monitor for indications of fatigue
Teach client measures to maximize functional activity
Provide a safe environment
Use progressive muscle relaxation
Administer meds and proper positioning
Monitor for medication effectiveness
Teach client signs/symptoms that need to be reported immediately
RA nutritional suggestions
Encourage foods high in vitamins, protein and iron also encourage small frequent meals
RA medications
NSAIDs, Cox 2 enzyme blockers (corticosteroids), disease modifying anti-rheumatic
drugs
DMARDS
Slow the progression of RA and suppress immune system
DMARD medication
Antimalarial agent: Hydroxychloroquine
Antibiotic: Minocycline
Sulfonamide: Sulfasalzine
Biologic response modifiers: Etanercept, infliximab, adalimumab, and cheltor
penicillamine
Cytotoxic medications: Methotrexate, leflunomide, cyclophosphamide and azathioprine
can cause severe adverse effects
Plasmapheresis
Removes circulating antibodies from plasma decreasing attacks on client tissues. May
be done for severe life threatening exacerbation. The process is similar to kidney
dialysis
RA complications
Sjogrens syndrome, secondary osteoporosis, vasculitis(organ ischemia)
Sjogrens syndrome
An immune system disorder characterized by dry eyes, vagina and dry mouth.
Secondary osteoporosis
is caused by certain medical conditions or treatments that interfere with the attainment
of peak bone mass and may cause bone loss.
Vasculitis
An inflammation of the blood vessels that causes changes in the blood vessel walls.
Monitor for skin lesions, decrease in vision and symptoms of cognitive dysfunction

A teenager is diagnosed with cellulitis of the right knee and fails to respond to oral
antibiotics. He then develops osteomyelitis of the right knee, prompting a detailed
diagnostic workup that reveals a phagocytic disorder. This patient faces an increased
risk of what complication?

A) Thrombocytopenia
B) HIV/AIDS
C) Neutropenia
D) Hemophilia
C
(Feedback:
Patients with phagocytic cell disorders may develop severe neutropenia. None of the
other listed health problems is a common complication of phagocytic disorders.)
A patient is admitted for the treatment of a primary immunodeficiency and intravenous
immunoglobulin (IVIG) is ordered. What should the nurse monitor for as a potential
adverse effect of IVIG administration?

A)
Anaphylaxis
B)
Hypertension
C)
Hypothermia
D)
Joint pain
A

(Feedback:
Potential adverse effects of an IVIG infusion include hypotension, flank pain, chills, and
tightness in chest, terminating with a slightly elevated body temperature and
anaphylactic reaction. Hypertension, hypothermia, and joint pain are not usual adverse
effects of IVIG.)
A nurse is admitting a patient with an immunodeficiency to the medical unit. In planning
the care of this patient, the nurse should assess for what common sign of
immunodeficiency?

A) Chronic diarrhea
B) Hyperglycemia
C) Rhinorrhea
D) Contact dermatitis
A

(Feedback:
The cardinal symptoms of immunodeficiency include chronic or recurrent severe
infections, infections caused by unusual organisms or organisms that are normal body
flora, poor response to treatment of infections, and chronic diarrhea. Hyperglycemia,
rhinorrhea, and contact dermatitis are not symptoms the patient is likely to exhibit.)
A young couple visits the nurse practitioner stating that they want to start a family. The
husband states that his brother died of a severe infection at age 6 months. He says he
never knew what was wrong but his mother had him undergo "blood testing" as a child.
Based on these statements, what health problem should the nurse practitioner suspect?

A) Severe neutropenia
B) X-linked agammaglobulinemia
C) Drug-induced thrombocytopenia
D) Aplastic anemia
B

(Feedback:
There is no evidence of drug-induced thrombocytopenia or aplastic anemia. The child
would have only suffered from severe neutropenia if there was evidence of bacterial or
fungal infections. The fact the mother of this individual had him tested for gamma-
globulin as a child would indicate that his sibling had X-linked agammaglobulinemia.
More than 10% of patients with X-linked agammaglobulinemia are hospitalized for
infection at less than 6 months of age. Since the condition is X-linked it is important for
the couple to undergo genetic testing.)
The parents of a 1-month-old infant bring their child to the pediatrician with symptoms of
congestive heart failure. The infant is ultimately diagnosed with DiGeorge syndrome.
What will prolong this infant's survival?

A) Stem cell transplantation


B) Long-term antibiotics
C) Chemotherapy
D) Thymus gland transplantation
D

(Feedback:
Transplantation of fetal thymus, postnatal thymus, or human leukocyte antigen (HLA)-
matched bone marrow has been used for permanent reconstitution of T-cell immunity in
infants with DiGeorge syndrome. Antibiotics and chemotherapy do not address the
etiology of the infant's disease. Stem cell transplantation is not a common treatment
modality.)
A patient who has received a heart transplant is taking cyclosporine, an
immunosuppressant. What should the nurse emphasize during health education about
infection prevention?

A) Eat a high-calorie, high-protein diet.


B) Limit physical activity in order to conserve energy.
C) Take prophylactic antibiotics as ordered.
D) Perform frequent handwashing.
D

(Feedback:
Hand hygiene is imperative in infection control. A well-balanced diet is important, but for
most patients this is secondary to hygiene as an infection-control measure. Prophylactic
antibiotics are not normally used. Limiting physical activity will not protect the patient
from infection.)
The nurse is caring for a patient who has a diagnosis of paroxysmal nocturnal
hemoglobinuria. When planning this patient's care, the nurse should recognize the
patient's heightened risk of what complication?

A) Venous thromboembolism
B) Acute respiratory distress syndrome (ARDS)
C) Myocardial infarction
D) Hypertensive urgency
A

(Feedback:
Patients with paroxysmal nocturnal hemoglobinuriahave a high incidence of life-
threatening venous thrombosis, which occurs most commonly in the abdominal and
cerebral veins. This health problem is not linked to ARDS, MI, or hypertensive urgency.)
A patient diagnosed with common variable immune deficiency (CVID) has been
admitted to the acute medicine unit. When reviewing this patient's laboratory findings,
the nurse should prioritize what values?

A) Creatinine and blood urea nitrogen (BUN)


B) Hemoglobin and vitamin B12
C) Sodium, potassium and magnesium
D) D-dimer and c-reactive protein
B

(Feedback:
A patient diagnosed with CVID often develops pernicious anemia; the patient's
hemoglobin and vitamin B12 levels would be used to assess for this common
complication of CVID. None of the other listed blood values directly relates to the signs
and complications of CVID.)
Patient teaching regarding infection prevention for the patient with an immunodeficiency
includes which of the following guidelines?

A) Cook all food thoroughly.


B) Refrain from using creams or emollients on skin.
C) Maintain contact only with individuals who have recently been vaccinated.
D) Take OTC vitamin supplements consistently.
A

(Feedback:
All foods must be cooked to avoid food-borne illness. The patient should avoid contact
with individuals who have recently been ill or vaccinated. The nurse should apply
creams and emollients to any dry, chaffed, or cracked skin. Vitamin supplements may or
may not be indicated.)
A nurse has admitted a patient diagnosed with severe combined immunodeficiency
disease (SCID) to the unit. The patient's orders include IVIG. How will the patient's dose
of IVIG be determined?

A) The patient will receive 25 to 50 mg/kg of body weight.


B) The dose will be determined by the patient's response.
C) The dose will be determined by body surface area.
D) The patient will receive a one-time bolus followed by 100- to 150-mg doses.
B

(Feedback:
The optimal dosage of IVIG is determined by the patient's response. In most instances,
an IV dose of 200 to 800 mg/kg of body weight is administered.)
The nurse is preparing to administer IVIG to a patient who has an immunodeficiency.
What nursing guideline should the nurse apply?

A) Do not exceed an infusion rate of 300 mL/hr.


B) Slow the infusion rate if the patient exhibits signs of a transfusion reaction.
C) Weigh the patient immediately after the infusion is complete.
D) Administer pretreatment medications as ordered 30 minutes prior to infusion.
D

(Feedback:
The nurse should administer pretreatment acetaminophen and diphenhydramine as
prescribed 30 minutes before the start of the infusion. The patient should be weighed
prior to the treatment and the IV infusion rate should not exceed 200 mL/hour. The
nurse should stop the transfusion in the event of any signs of a reaction.)
IVIG has been ordered for the treatment of a patient with an immunodeficiency. Which
of the following actions should the nurse perform before administering this blood
product?

A) Ensure that the patient has a patent central line.


B) Ensure that the IVIG is appropriately mixed with normal saline.
C) Administer furosemide before IVIG to prevent hypervolemia.
D) Weigh the patient before administration to verify the correct dose.
D

(Feedback:
The nurse should obtain height and weight before treatment to verify accurate dosing.
IVIG can be administered through a peripheral line. Diuretics are not normally given
prior to administration, and IVIG is not mixed with normal saline.)
A patient with a diagnosis of common variable immunodeficiency begins to develop
thick, sticky, tenacious sputum. The patient has a history of episodes of pneumonia at
least one time per year for the last 10 years. What does the nurse suspect the patient is
developing?

A) Pulmonary edema
B) A pulmonary neoplasm
C) Bronchiectasis
D) Emphysema
C

(Feedback:
Frequent respiratory tract infections in patients with CVID typically lead to chronic
progressive bronchiectasis and pulmonary failure. Pulmonary edema is often a result of
vascular insufficiency. A patient suffering from CVID is likely to develop gastric cancer,
not lung cancer. The patient is not at risk for emphysema.)
A nurse is admitting an adolescent patient with a diagnosis of ataxia-telangiectasis.
Which of the following nursing diagnoses should the nurse include in the patient's plan
of care?

A) Fatigue Related to Pernicious Anemia


B) Risk for Constipation Related to Decreased Gastric Motility
C) Risk for Falls Due to Loss of Muscle Coordination
D) Disturbed Kinesthetic Sensory Perception Related to Vascular Changes
C

(Feedback:
Ataxia-telangiectasia is an autosomal recessive neurodegenerative disorder
characterized by cerebellar ataxia (loss of muscle coordination), telangiectasia (vascular
lesions caused by dilated blood vessels), and immune deficiency. Decreased
coordination is likely to constitute a risk for falls. The patient does not characteristically
lose tactile sensation or experience pernicious anemia or constipation.)
A 20-year-old patient with an immunodeficiency is admitted to the unit with an acute
episode of upper airway edema. This is the fifth time in the past 3 months that the
patient has had such as episode. As the nurse caring for this patient, you know that the
patient may have a deficiency of what?

A) Interferons
B) C1esterase inhibitor
C) IgG
D) IgA
B

(Feedback:
Hereditary angioneurotic edema results from the deficiency of C1esterase inhibitor,
which opposes the release of inflammatory mediators. The clinical picture of this
autosomal dominant disorder includes recurrent attacks of edema. A patient with this
diagnosis does not lack interferons, IgG, or IgA.)
A patient with Wiskott-Aldrich syndrome is admitted to the medical unit. The nurse
caring for the patient should prioritize which of the following?

A) Protective isolation
B) Fresh-frozen plasma administration
C) Chest physiotherapy
D) Nutritional supplementation
A

(Feedback:
Patients with Wiskott-Aldrich syndrome (WAS) are at a grave risk for infection; infection
prevention is a priority aspect of nursing care. Nutritional supplementation may be
necessary, but infection prevention is paramount. Chest physiotherapy and FFP
administration are not indicated.)
The nurse is admitting a patient to the unit with a diagnosis of ataxia-telangiectasia. The
nurse's assessment should reflect the patient's increased risk for what complication?

A) Peripheral edema
B) Cancer
C) Anaphylaxis
D) Gastrointestinal bleeds
B

(Feedback:
Frequent causes of death in patients with ataxia-telangiectasiaare chronic pulmonary
disease and malignancy. Peripheral edema, anaphylaxis, and GI bleeding are not noted
to be common among patients with ataxia-telangiectasia.)
The nurse is working with the interdisciplinary team to care for a patient who has
recently been diagnosed with severe combined immunodeficiency disease (SCID).
What treatment is likely of most benefit to this patient?

A) Combined radiotherapy and chemotherapy


B) Antibiotic therapy
C) Hematopoietic stem cell transplantation (HSCT)
D) Treatment with colony-stimulating factors (CSFs)
C

(Feedback:
Treatment options for SCID include stem cell and bone marrow transplantation, but
HSCT is the definitive therapy for the disease and supersedes the importance of
antibiotics. CSFs, radiation therapy, and chemotherapy are not indicated.)
A patient has been admitted with a phagocytic cell disorder and the nurse is reviewing
the most common health problems that accompany these disorders. The nurse should
identify which of the following? Select all that apply.

A) Inflammatory bowel disease


B) Chronic otitis media
C) Cutaneous abscesses
D) Pneumonia
E) Cognitive deficits
B,C,D

(Feedback:
Patients with phagocytic cell disorders experience recurrent cutaneous abscesses,
chronic eczema, bronchitis, pneumonia, chronic otitis media, and sinusitis. Irritable
bowel syndrome and cognitive deficits are atypical.)
A nurse is caring for a patient with a phagocytic cell disorder. The patient states, "My
specialist says that I will likely be cured after I get my treatment tomorrow." To what
treatment is the patient most likely referring?

A) Treatment with granulocyte-macrophage colony-stimulating factor (GM-CSF)


B) Hematopoietic stem cell transplantation
C) Treatment with granulocyte colony-stimulating factor (G-CSF)
D) Brachytherapy
B

(Feedback:
Hematopoietic stem cell transplantation (HSCT), another form of cell therapy, has
proven to be a successful curative modality. Treatment with GM-CSF or G-CSF is not
curative. Brachytherapy is not a treatment for immunodeficiency.)
A patient's primary immunodeficiency disease is characterized by the inability of white
blood cells to initiate an inflammatory response to infectious organisms. What is this
patient's most likely diagnosis?

A) Chronic granulomatous disease


B) Wiskott-Aldrich syndrome
C) Hyperimmunoglobulinemia E syndrome
D) Common variable immunodeficiency
C

(Feedback:
In one rare type of phagocytic disorder, hyperimmunoglobulinemia E syndrome
(formerly known as Job syndrome), white blood cells cannot initiate an inflammatory
response to infectious organisms. The other listed health problems do not have this
pathology.)
A nurse educator is explaining that patients with primary immunodeficiencies are living
longer than in past decades because of advances in medical treatment. This increased
longevity is associated with an increased risk of what?

A) Chronic obstructive pulmonary disease


B) Dementia
C) Pulmonary fibrosis
D) Cancer
D

(Feedback:
Advances in medical treatment have meant that patients with primary
immunodeficiencies live longer, thus increasing their overall risk of developing cancer. It
does not mean that they are at increased risk of COPD, dementia, or pulmonary
fibrosis.)
The nurse educator is differentiating primary immunodeficiency diseases from
secondary immunodeficiencies. What is the defining characteristic of primary
immunodeficiency diseases?

A) They require IVIG as treatment.


B) They are the result of intrauterine infection.
C) They have a genetic origin.
D) They are communicable.
C

(Feedback:
Primary immunodeficiency diseases are genetic in origin and result from intrinsic
defects in the cells of the immune system. Primary immunodeficiency diseases do not
always need IVIG as treatment, and they are not communicable. Primary
immunodeficiencies do not result from intrauterine infection.)
A nurse has created a plan of care for an immunodeficient patient, specifying that care
providers take the patient's pulse and respiratory rate for a full minute. What is the
rationale for this aspect of care?

A) Respirations affect heart rate in immunodeficient patients.


B) These patients' blunted inflammatory responses can cause subtle changes in status.
C) Hemodynamic instability is one of the main complications of immunodeficiency.
D) Immunodeficient patients are prone to ventricular tachycardia and atrial fibrillation.
B

(Feedback:
Pulse rate and respiratory rate should be counted for a full minute, because subtle
changes can signal deterioration in the patient's clinical status. The rationale for this
action is not because of the relationship between heart rate and respirations. These
patients do not have a greatly increased risk of hemodynamic instability or
dysrhythmias.)
A nurse is providing health education regarding self-care to a patient with an
immunodeficiency. What teaching point should the nurse emphasize?

A) The importance of aggressive treatment of acne


B) The importance of avoiding alcohol-based cleansers
C) The need to keep fingernails and toenails closely trimmed
D) The need for thorough oral hygiene
D

(Feedback:
Many patients develop oral manifestations and need education about promoting good
dental hygiene to diminish the oral discomfort and complications that frequently result in
inadequate nutritional intake. Alcohol cleansers do not necessarily need to be avoided
and nail care is not a central concern. Acne care is not a main focus of education, since
it is not relevant to many patients.)
The nurse is applying standard precautions in the care of a patient who has an
immunodeficiency. What are key elements of standard precautions? Select all that
apply.

A) Using appropriate personal protective equipment


B) Placing patients in negative-pressure isolation rooms
C) Placing patients in positive-pressure isolation rooms
D) Using safe injection practices
E) Performing hand hygiene
A,D,E

(Feedback:
Some of the key elements of standard precautions include performing hand hygiene;
using appropriate personal protective equipment, depending on the expected type of
exposure; and using safe injection practices. Isolation is an infection control strategy but
is not a component of standard precautions.)
The nurse is caring for a patient with an immunodeficiency who has experienced
sudden malaise. The nurse's colleague states, "I'm pretty sure that it's not an infection,
because the most recent blood work looks fine." What principle should guide the nurse's
response to the colleague?

A) Immunodeficient patients will usually exhibit subtle and atypical signs of infection.
B) Infections in immunodeficient patients have a slower onset but a more severe
course.
C) Laboratory blood work is often inaccurate in immunodeficient patients.
D) Immunodeficient patients do not develop symptoms of infection.
A

(Feedback:
Immunodeficient patients often lack the typical objective and subjective signs and
symptoms of infection. However, this does not mean that they wholly lack symptoms.
Infections do not normally have a slower onset. Blood work may not be a reliable
diagnostic tool, but that does not mean that the results are inaccurate.)
A nurse is caring for a patient who has an immunodeficiency. What assessment finding
should prompt the nurse to consider the possibility that the patient is developing an
infection?

A) Uncharacteristic aggression
B) Persistent diarrhea
C) Pruritis (itching)
D) Constipation
B

(Feedback:
Persistent diarrhea is among the varied signs and symptoms that may suggest infection
in an immunocompromised patient. Aggression, pruritis, and constipation are less
suggestive of an infectious etiology.)
A patient with a diagnosis of primary immunodeficiency informs the nurse that he has
been experiencing a new onset of a dry cough and occasional shortness of breath. After
determining that the patient's vital signs are within reference ranges, what action should
the nurse take?

A) Administer a nebulized bronchodilator.


B) Perform oral suctioning.
C) Assess the patient for signs and symptoms of infection.
D) Teach the patient deep breathing and coughing exercises.
C

(Feedback:
Dyspnea and cough are among the many signs and symptoms that may suggest
infection in an immunocompromised patient. There is no indication for suctioning or the
use of nebulizers. Deep breathing and coughing exercises do not address the patient's
complaints or the likely etiology.)
A home health nurse is reinforcing health education with a patient who is
immunosuppressed and his family. What statement best suggests that the patient has
understood the nurse's teaching?

A) "My family needs to understand when I can go get the seasonal flu shot."
B) "I need to know how to treat my infections in a home setting."
C) "I need to understand how to give my platelet transfusions."
D) "My family needs to understand that I'll probably need lifelong treatment."
D

(Feedback:
The patient must be made aware that all health-related instructions are lifelong.
Immunizations may be contraindicated and infection usually requires inpatient
treatment. Platelet transfusions are not indicated for most patients who have
immunodeficiencies.)
A nurse is preparing to administer a scheduled dose of IVIG to a patient who has a
diagnosis of severe combined immunodeficiency disease (SCID). What medication
should the nurse administer prior to initiating the infusion?

A) Diphenhydramine
B) Ibuprofen
C) Hydromorphone
D) Fentanyl
A

(Feedback:
Diphenhydramine and acetaminophen are administered 30 minutes prior to an IVIG
infusion.)
An immunocompromised patient is being treated in the hospital. The nurse's
assessment reveals that the patient's submandibular lymph nodes are swollen, a finding
that represents a change from the previous day. What is the nurse's most appropriate
action?

A) Administer a PRN dose of acetaminophen as ordered.


B) Monitor the patient's vital signs q2h for the next 24 hours.
C) Inform the patient's primary care provider of this finding.
D) Implement standard precautions in the patient's care.
C

(Feedback:
Swollen lymph nodes are suggestive of infection and warrant prompt medical
assessment and treatment. Acetaminophen is an ineffective response. The nurse
should monitor the patient's vital signs closely, but the physician should also be
informed. Standard precautions should be in place regardless of the patient's status.)
A nurse caring for a patient who has an immunosuppressive disorder knows that
continual monitoring of the patient is critical. What is the primary rationale behind the
need for continual monitoring?

A) So that the patient's functional needs can be met immediately


B) So that medications can be given as ordered and signs of adverse reactions noted
C) So that early signs of impending infection can be detected and treated
D) So that the nurse's documentation can be thorough and accurate
C

(Feedback:
Continual monitoring of the patient's condition is critical, so that early signs of impending
infection may be detected and treated before they seriously compromise the patient's
status. Continual monitoring is not primarily motivated by the patient's functional needs
or medication schedule. The nurse's documentation is important, but less than infection
control.)
A nurse is planning the care of a patient who requires immunosuppression to ensure
engraftment of depleted bone marrow during a transplantation procedure. What is the
most important component of infection control in the care of this patient?

A) Administration of IVIG
B) Antibiotic administration
C) Appropriate use of gloves and goggles
D) Thorough and consistent hand hygiene
D

(Feedback:
Hand hygiene is usually considered the most important aspect of infection control. IVIG
and antibiotics are not considered infection control measures, though they enhance
resistance to infection and treat infection. Gloves and goggles are sometimes indicated
but are less effective than hand hygiene.)
A home health nurse is caring for a patient who has an immunodeficiency. What is the
nurse's priority action to help ensure successful outcomes and a favorable prognosis?

A) Encourage the patient and family to be active partners in the management of the
immunodeficiency.
B) Encourage the patient and family to manage the patient's activity level and activities
of daily living effectively.
C) Make sure that the patient and family understand the importance of monitoring fluid
balance.
D) Make sure that the patient and family know how to adjust dosages of the medications
used in treatment.
A

(Feedback:
Encouraging the patient and family to be active partners in the management of the
immunodeficiency is the key to successful outcomes and a favorable prognosis. This
transcends the patient's activity and functional status. Medications should not be
adjusted without consultation from the primary care provider. Fluid balance is not
normally a central concern.)
A nurse is preparing to discharge a patient with an immunodeficiency. When preparing
the patient for self-infusion of IVIG in the home setting, what education should the nurse
prioritize?

A) Sterile technique for establishing a new IV site


B) Signs and symptoms of adverse reactions
C) Formulas for calculating daily doses
D) Technique for adding medications to the IVIG
B

(Feedback:
The patient who is to receive IVIG at home will need information about adverse
reactions and their management. A patient would not start a new IV site independently
and the patient does not calculate changes in dose independently. Medications are not
added to IVIG.)
A home health nurse will soon begin administering IVIG to a new patient on a regular
basis. What teaching should the nurse provide to the patient?

A) The need for a sterile home environment


B) Complementary alternatives to IVIG
C) Expected benefits and outcomes of the treatment
D) Technique for managing and monitoring daily fluid intake
C

(Feedback:
The patient who is to receive IVIG at home will need information about the expected
benefits and outcomes of the treatment as well as expected adverse reactions and their
management. The home environment cannot be sterile and complementary alternatives
to IVIG have not been identified. Fluid management is not a central concern.)
The home health nurse is assessing a patient who is immunosuppressed following a
liver transplant. What is the most essential teaching for this patient and the family?

A) How to promote immune function through nutrition


B) The importance of maintaining the patient's vaccination status
C) How to choose antibiotics based on the patient's symptoms
D) The need to report any slight changes in the patient's health status
D

(Feedback:
They must be informed of the need for continuous monitoring for subtle changes in the
patient's physical health status and of the importance of seeking immediate health care
if changes are detected. Nutrition is important, but infection control is the priority.
Patients and families do not choose antibiotics independently. Vaccinations are often
contraindicated in immunocompromised patients.)
Family members of an immunocompromised patient have asked the nurse why
antibiotics are not being given to the patient in order to prevent infection. How should
the nurse best respond?

A) "Using antibiotics to prevent infections can cause the growth of drug-resistant


bacteria."
B) "If an antibiotic is given to prevent a bacterial infection, the patient is at risk of a viral
infection."
C) "Antibiotics can never prevent an infection; they can only cure an infection that is fully
developed."
D) "Antibiotics cannot resolve infections in people who are immunocompromised."
A

(Feedback:
Although prophylactic drug treatment effectively prevents some bacterial and fungal
infections, it must be used with caution because it has been implicated in the
emergence of resistant organisms. Use of antibiotics does not directly increase the risk
of viral infections.)
A 6-month-old infant has been diagnosed with X-linked agammaglobulinemia and the
parents do not understand why their baby did not develop an infection during the first
months of life. The nurse should describe what phenomenon?

A) Cell-mediated immunity in infants


B) Passive acquired immunity
C) Phagocytosis
D) Opsonization
B
(Feedback:
Infants with X-linked agammaglobulinemia usually become symptomatic after the
natural loss of maternally transmitted immunoglobulins (passive acquired immunity),
which occurs at about 5 to 6 months of age. Opsonization is the coating of antigen-
antibody molecules with a sticky substance to facilitate phagocytosis. Cell-mediated
immunity and phagocytosis do not directly affect the timeline of the infant's symptoms.)

What is immunodeficiency?
the inability to produce a normal complement of antibodies or immunologically
sensitized T cells, especially in response to specific antigens

clinical hallmark is unusual or recurrent severe infections


What is the primary immune response?
occurs when an antigen is initially introduced into the system

- involves mast cell degranulation & activation of plasma proteins


What is the secondary or specific antibody response?
activation of the B cells and memory cells (IgG, IgM, IgA, & IgE), T cells, cytotoxic cells,
lymphokine-producting cells, helper cells, and suppressor cells to a specific antigen
What are the immunodeficient (Type I) immunological disorders?
1. AIDS
2. DigGeorge's syndrome - defect on chromosome 22
What are the autoimmune (Type II) immunological disorders?
1. myasthenia gravis
2. ankylosing spondylitis fibromyalgia
3. polymyositis & dermatomyositis
4. RA (also connective tissue disorder)
5. SLE (also vascular & connective tissue disorder)
6. Sjorgren's syndrome
7. vasculitis
8. Reiter's syndrome
9. polymyalgia rheumatica & cranial arteritis
10. Grave's disease
11. hyperthyroidism w/goiter and ophthalmopathy
12. progressive systemic slerosis (scleroderma)
What are the mixed connective tissue disease immunological disorders?
1. lyme disease
2. secondary arthritis
What is the immunoproliferative (Type III) immunological disorder?
leukemia
What are the hypersensitive (Type IV) immunological disorders?
1. allergies
2. asthma
3. contact dermatitis

pretty much any allergic reactions


What is human immunodeficiency virus (HIV)?
a retrovirus that enters the T lymphocyte & alters the RNA of the normal T lymphocyte,
compromising the immune system

can progress to AIDS


What are the risk factors for HIV?
1. unsafe sexual practices
2. rape
3. prostitution
4. multiple sex partners
5. exposure to contaminated blood/needles
6. occupational exposure
7. perinatal exposure
How is HIV transmitted?
1. blood
2. semen
3. vaginal or cervical secretions
4. breast milk
What is HIV NOT transmitted through?
1. tears
2. saliva
3. urine
4. emesis
5. sputum
6. feces
7. sweat
How is AIDS diagnosed?
Have at least one of the following:

1. CD4+ below 200cells/uL


2. development of one or more fungal, viral, protozoal, or bacterial opportunistic
infections
3. development of opportunistic cancer (i.e. Kaposi's sarcoma, immunoblastic
lymphoma)
4. wasting syndrome (loss of 10% or more of body mass)
5. development of dementia
What is Kaposi's sarcoma?
occurs in epithelial cells that line small blood vessels, the skin, mucous membranes, &
GI tissues
What are the characteristic signs of Kaposi's sarcoma?
raised red-violet plaques, or tumors, particularly on the head, in oral tissues, & on the
neck & trunk of the body

progresses rapidly in persons with AIDS


What can effect the progression of HIV to AIDS?
1. alcoholism
2. drug dependence
3. liver, kidney, or other organ diseases
4. hx of STDs
5. psychiatric illnesses
What are the early clinical manifestations of HIV?
1. chills
2. fever
3. myalgia
4. malaise
5. sore throat
6. nausea
7. photophobia
8. lymphadenopathy
9. maculopapular rash
10. H/A
What are the late clinical manifestations of HIV?
1. chills
2. fever
3. night sweats
4. dry productive cough
5. dyspnea
6. lethargy
7. confusion
8. stiff neck
9. seizures
10. H/A
11. malaise
12. fatigue
13. oral lesions
14. skin rash
15. abdominal discomfort
16. diarrhea
17. weight loss
What are the emotional responses to HIV?
1. anger
2. fear
3. guilt
4. denial
5. depression
6. suicidal tendencies
What are the diagnostic tests for HIV?
1. high viral load & dramatic drop in CD4+ cell count during initial infection
2. enzyme-linked immunosorbent assay (ELISA)
3. western blot test
4. at home test kits, salivary tests, & urine tests

emphasis on CD4+ cell counts


HIV Diagnostic Tests Overview
1. CD4 Count - hallmark of HIV infection - decreased amount
2. CD4/CD8+ ratio
3. HIV-RNA concentration - amount of HIV in the blood (viral load)
4. ELISA - detects HIV antibodies
5. Western blot test - detects antibodies to specific viral proteins (used after positive
ELISA)
6. HIV antigen tests - used for diagnosis in early stage of acute infection
7. DNA-PCR amplification - detects proviral DNA molecules in infected nuclei of
lymphocytes
What are some nursing diagnoses for HIV?
1. acute pain r/t HIV
2. fear in response to diagnosis of HIV
3. risk for infection r/t HIV
4. ineffective coping r/t anxiety, lower activity level, & inability to perform normal ADLs r/t
HIV
5. knowledge deficit r/t self-care & risk prevention for HIV
What is the treatment for HIV?
Highly active antiretroviral therapy (HAART)
1. Nucleoside Reverse Transcriptase Inhibitors (NRTIs) - zidovudine (Retrovir),
didanosine (Videx), stavudine (Zerit), lamivudine (Epivir), abacavir (Ziagen)
2. Nucleotide Reverse Transcriptase Inhibitor - tenofovir DF (Viread)
3. Nonnucleoide Reverse Transcriptase Inhibitors (NNRTIs) - nevirapine (Viramune),
delavirdine (Rescriptor), efavirenz (Sustival)
4. Protease Inhibitors (PIs) - saquinavir (Fortovase), indinavir (Crixivan), ritonavir
(Norvir), nelfinavir (Viracept), amprenavir (Agenerase), kaletra (combo of lopinavir &
ritonvir)
5. Fusion inhibitor (entry inhibitor) - enfuvirtide (Fuzeon)

goal is to maintain or increase CD4+ levels to >200cell/uL & delay development of


symptoms & opportunistic infections
What are the side effects of the drug therapy for HIV?
1. N/V
2. anemia
3. leucopenia
4. myopathy
5. fatigue
6. H/A
7. dizziness
8. nasal congestion
9. dose-related peripheral neuropathy
10. hypersensitivity reaction
11. sore throat
12. rash
13. pruritus
14. cough
15. SOB
16. hepatitis
17. bone demineralization
18. hyperlipidemia
19. hyperglycemia
20. GI disturbances
21. skin rashes
22. lactic acidosis
23. pancreatitis
24. anemia
25. neutropenia
What are the long-term effects of HAART?
1. HTN
2. diabetes
3. osteopenia
4. hyperlipidemia
What are the barriers to adherence for HAART?
1. side effects
2. interactions w/other drugs
3. cost
4. substance abuse
5. homelessness
6. mental health issues
7. sense of decreased self-efficacy
What are immunodeficiency disorders?
dysfunction or impairment of one or more of the immune response mechanisms
What is graft-versus-host (GVH) disease?
result of a transfusion of blood or a tissue transplant w/cells that are incompatible with
the host

acute rejection response occurs 7-30 days after transplant


What are the clinical manifestations of GVH disease?
1. maculopapular rash
2. pruritus
3. pain
4. mild jaundice
5. elevated liver enzymes
6. hepatic coma
7. GI bleeding
8. malabsorption
9. bacterial & fungal infections
10. interstitial pneumonitis
What is chronic GVH disease?
occurs approximately 100 days after the transplant

symptoms include
1. scleroderma-like appearance
2. dry lacrimal ducts
3. dry oral mucosa
What is allograft or homograft transplant?
cells and tissue obtained from the same species (i.e. close relative or unrelated donor
with a similar type of cell compatibility)
What is histocompatibility testing?
done to determine the similarities and differences in the complex set of proteins on the
surface membranes of all human nucleated cells, tissues, & blood cells (except RBCs)
What is the treatment for the variety of hematological, malignant, & nonmalignant
disorders?
bone marrow transplant
What is a syngeneic transplant?
use of bone marrow cells donated by an identical twin w/no detectable tissue
incompatibility
What is autologous transplant?
removal of bone marrow cells from the individual - treated and stored - then returned
after the individual receives intensive chemotherapy or radiation
What can the blood from the umbilical cord and placenta used for?
rapid therapy for leukemia, lymphoma, & many other life-threatening diseases

close match is not needed


What is immunosuppresive therapy?
use of several immunosuppressive drugs to suppress the immune response but retain
enough immunity to prevent opportunistic infections

*traditional therapy includes calcineurin inhibitor (cyclosporine), corticosteroid


(prednisone), and mycophenolate mofetil (CellCept)
What are monoclonal antibodies?
genetically engineered immunosuppressive agents - used w/other agents to prevent
graft rejection
What is apheresis?
procedure of withdrawing blood from a donor, removing one or more components from
the blood, and transfusing the disorder components into patients w/low platelet or WBC
counts; the remaining blood is transfused back into the donor

sometimes used to treat autoimmune diseases


What is plasmapheresis?
plasma exchange

process of removing plasma that is causing the disease and replacing it with normal
saline, lactated Ringer's, frozen plasma, or albumen

a needle is inserted into the arm, the blood is circulated through a cell separator that
separates the blood and plasma; remainder of contents are returned via needle into the
opposite arm

platelets, plasma proteins, WBCs, and RBCs can be selected separately


What are the side effects of plasmapheresis?
1. hypotension
2. citrate toxicity - can cause hypocalcemia (H/A, dizziness, paresthesias)
What are hypersensitivity disorders?
heightened immune response

allergies & allergic reaction


What are autoimmune disorders?
the immune system fails to recognize the body's normal tissue as self and begins to
break down normal cells
What are the most common autoimmune disorders?
1. osteoarthritis
2. rheumatoid arthritis (RA)
3. gout
3. SLE
5. progressive systemic sclerosis (scleroderma)
6. ankylosing spondylitis

common clinical manifestation of pain & impaired mobility


What are the risk factors for autoimmune disorders?
1. genetic predisposition
2. infections
3. interaction of T & B cells
4. introduction of foreign tissue (blood transfusion, transplant tissue)
5. viral infections have been linked to MS and Type I DM
6. rheumatic fever has been linked to rheumatic heart disease
7. hormones (women affected more than men)
8. allergies
9. age
What is rheumatoid arthritis (RA)?
causes chronic inflammation of the synovial membrane and leads to swelling, pain, and
eventual erosion of the joint cartilage and bone and buildup of pannus (granulation
tissue)

can be controlled but there is no cure


What are the symptoms of RA?
1. joint deformities
2. fever
3. fatigue
4. joint stiffness, especially in the morning
5. diffuse musculoskeletal pain
6. anorexia
7. weight loss
8. synovitis
9. muscle spasm & weakness
10. sleep disturbances
What are the signs of juvenile rheumatoid arthritis?
1. enlarged knuckles that are warm to the touch
2. complaints of pain in ankles, feet, or knees
3. decreased ROM

initial symptoms often overlooked as growing pains


What are the diagnostic tests for RA?
1. serum RF - positive in 80% of all patients
2. antimitochondrial antibody
3. antinuclear antibodies
4. antistreptolysin O
5. complement C3
6. C-reactive protein
7. lupus erythematosus cell preperation (LE prep)
8. thyroid antibody
9. erythrocyte sedementation rate
10. X-rays
11. bone scans
12. samples of synovial fluid (will be straw-colored, slightly cloudy, & have many flecks
of fibrin in early disease)
13. WBC count 3500 to 25,000/mm3
What are the criteria for the diagnosis of RA?
1. morning stiffness lasting longer than one hour
2. arthritis of three or more joint areas
3. arthritis of hand joints
4. symmetrical arthritis
5. rheumatoid nodules over extensor surfaces or bony prominences
6. serum rheumatoid factors
7. radiographic changes
What are some nursing diagnoses for RA?
1. acute pain
2. chronic pain
3. limited mobility
4. activity intolerance
5. self-care deficit
What is the focus of treatment for RA?
managing pain and reducing inflammation, promoting remission, increasing ADL
function abilities, & helping individuals cope w/disabilities
What is the recommended nutrition for RA?
1. use of monounsaturated fats found in olive oil & avocados
2. omega-3 fatty acids such as walnuts, fish, eggs, grass-fed beef
3. Mediterranean diet
What are the medications used to treat RA?
1. selective and nonselective NSAIDs (ibuprofen, feldene, celecoxib)
2. synthetic & biological disease-modifying antirheumatic drugs (DMARDs)
(methotrexate, infliximab, etanercept, adalimumab)
3. corticosteroids
What is the main surgery used to treat RA?
arthroplasty - replacement of the damaged joints

Other surgeries used


1. synovectomies (removal of synovial membrane)
2. joint fusions (total immobility of the affected joint)
3. tendon transfers
What is Sjogrens syndrome?
chronic inflammatory disorder involving the eyes that can be a primary problem or
secondary to RA

dry eyes & dry mouth are hallmark signs


What is systemic lupus erythematosus (SLE)?
severe form of an autoimmune disorder & is classified as a multisystem disease
What is the discoid type of SLE?
a milder form that affects the skin, primarily the face, neck, and upper chest
What are the causes of the reversible form of SLE?
medications that bind with a person's DNA

1. oral contraceptives
2. isoniazid
3. procainamide
4. methyldopa
What are the risk factors for SLE?
1. gender (female)
2. age (15-40)
3. race (African-American, Hispanic, Asian, Native American)
What are the possible effects of SLE on pregnancy?
1. hx of miscarriages or preterm births
2. increased risk for fetal or newborn deaths when there is kidney damage or HTN
3. pre-eclampsia
4. blood clots in the placenta
5. hyperglycemia
What are the clinical manifestations of SLE?
1. polyarthritis (involving many joints)
2. polyarthralgia (pain in many joints)
3. lethargy
4. malaise
5. fever
6. weight loss
7. butterfly rash on face
8. pleural effusion
9. basilar pneumonia
10. generalized lymphadenopathy
11. pericarditis
12. hepatosplenomegaly

Severe cases
1. delirium
2. convulsions
3. psychosis
4. coma
What is progressive systemic sclerosis (scleroderma)?
rare connective tissue disease that involves excess collagen deposition & changes in
both the humoral and cellular immunity
What is a morphea type skin lesion in sleroderma?
hard, oval, and white w/a purple ring surrounding them
What does a skin lesion in linear scleroderma look like?
thick line of skin on the arms, legs, or forehead that can become fixated to the
subdermal structures including the fascia that covers tendon sheaths & muscles
What body systems are involved in the general form of scleroderma?
skin and many internal organs
What body systems are involved in the subcutaneous scleroderma form of
scleroderma?
involves the skin, the synovium, digital arteries, parenchyma, and small arteries in
internal organs
What body systems are involved in the diffuse subcutaneous form (CREST) of
scleroderma?
skin, muscles, joints, lungs, esophagus, heart, digestive system, and kidneys
What are the clinical manifestations of scleroderma (CREST)?
Calcinosis - development of small white calcium deposits under the skin; chalky fluid
drains when lumps break open

Raynaud's syndrome - spasms of arteries & arterioles occur spontaneously

Esophageal movement is decreased due to deposits of collagen & muscle atrophy

Telangiectasia - a permanent dilation of the capillaries, arterioles, and venules


What are the clinical manifestations of the localized form of scleroderma?

may not occur for 10-20 years


1. pain
2. edema
3. calcification
4. muscular atrophy

heart, kidney, or lung involvement is severe & tends to occur early in the disease;
has high mortality rate
What is the treatment for scleroderma?
supportive & tailored to clinical manifestations; primary goal to promote remission

1. steroids
2. immunosuppressants
During the early stages of a chronic disease, clients tend to focus on:

1. Medication schedule

2. Interpretation of symptoms

3. Impact of lifestyle changes

4. Understanding the disease process


3.

Rationale: Clients tend to focus on lifestyle changes in the early stages of a chronic
disease.
The goal of HAART is to:

1. Minimize side affects of the drugs

2. Encourage client compliance to the medication schedule

3. Lower the CD4 cell levels

4. Avoid viral resistance for each drug


4.

Rationale: The goal of HAART is to avoid viral resistance for each drug.
Early clinical manifestations of GVHD include:

1. Respiratory problems, for example interstitial pneumonitis

2. Skin rash and pruritus


3. Taut, firm, leather-like skin

4. Dry lacrimal ducts and oral mucosa


2.

Rationale: The early clinical manifestations of GVHD are skin rash and pruritus.
Monoclonal antibodies are:

1. B-cell antibodies developed against a foreign antigen

2. Defective T-cell antibodies that do not recognize self tissue

3. Genetically engineered immunosuppressant drugs

4. Used in vaccines to assist in preventing infections


3.

Rationale: Monoclonal antibodies are genetically engineered immunosuppressant


drugs.
Hypersensitivity disorders are due to a (an):

1. Immune deficiency disorder, such as HIV

2. Autoimmune disorder, such as RA

3. Heightened immune response to an antigen

4. Desensitization of humoral immune components


3.

Rationale: Hypersensitivity disorders are because of a heightened immune response to


an antigen.
The reversible form of SLE is due to reaction to drugs that are known to bind with the
individual's DNA, such as:

1. Beta blockers, such as Inderal

2. Oral contraceptives, such as Levora

3. Antibiotics, such as tetracycline

4. Antimalarials, such as Plaquenil


2.

Rationale: The reversible form of SLE is because of a reaction to drugs, such as oral
contraceptives (e.g., Levora).
Cell-mediated immunity is initiated by:

1. Specific antigen recognition by T cells

2. Nonspecific antigen recognition by B cells

3. Release of complement cells into the blood stream

4. Release of cytokines from white blood cells


1.

Rationale: Cell-mediated immunity is initiated by specific antigen recognition by T cells.


Passive immunity involves:

1. Transfer of antibodies through the heart of the mother to fetus

2. Inoculation with vaccine containing live or killed infectious organisms

3. Development of sensitized lymphocytes within the host body

4. Response of memory cells to entry of an infectious organism


2.

Rationale: Passive immunity involves inoculation with vaccine containing live or killed
infectious organisms.
Sero-conversion is the presence of antibodies of HIV in the blood and are detected by
diagnostic studies within:

1. One to two weeks after exposure to HIV

2. One to three months or more after exposure to HIV

3. One month following the start of antiretroviral drug therapy

4. Six months, when immune-antibody complexes are formed


2.

Rationale: Sero-conversion antibodies to HIV are detected by diagnostic studies within


one to three months or more after exposure to HIV.
Criteria used to diagnose AIDS in an individual with HIV include at least one of the
following conditions:

1. CD4+ cell count above 200 cells/μL

2. Anemia due to diminished RBC count


3. Dysfunction of one or more organs

4. Development of an opportunistic cancer


4.

Rationale: Criteria used to diagnose AIDS in an individual with HIV includes


development of an opportunistic cancer.

When the nurse administers intravenous gamma-globulin infusion, she recognizes that
which symptom, if reported by the client, may indicate an adverse effect of the infusion?
Tightness in the chest pg. 991
T-cell deficiency occurs when which gland fails to develop normally during
embryogenesis?
Thymus pg. 992
Nursing students are reviewing information about the various types of primary
immunodeficiencies. The students demonstrate understanding of the material when
they identify which of the following as an example of a primary immunodeficiency
involving B-lymphocyte dysfunction?
IgA deficiency pg. 987
The nurse is aware that the most prevalent cause of immunodeficiency worldwide is
Malnutrition pg. 994
A client is given a dose of ketorolac, a nonsteroidal anti-inflammatory drug for
complaints of abdominal pain. Ten minutes after receiving the medication, the client's
eyes, lips, and face begin to swell, and the nurse hears stridor. What priority measure
should the nurse prepare to do?
Administer epinephrine. pg. 1037
A client received 2 units of packed red blood cells while in the hospital with rectal
bleeding. Three days after discharge, the client experienced an allergic response and
began to itch and break out with hives. What type of reaction does the nurse understand
could be occurring?
Delayed hypersensitivity response pg. 1038
A client calls the clinic and asks the nurse if using oxymetazoline nasal spray would be
alright to relieve the nasal congestion he is experiencing due to seasonal allergies.
What instructions should the nurse provide to the client to avoid complications?
Do not overuse the medication as rebound congestion can occur. pg. 1040
A nurse is assessing a client with a primary immunodeficiency. Afterward the nurse
documents that the client displayed ataxia. The nurse makes this documentation
because the client has
uncoordinated muscle movements. pg. 992
A client with chronic mucocutaneous candidiasis, an autosomal recessive disorder, asks
the nurse, "Will my children have this disease?" Which response by the nurse is
appropriate?
"All of your children will be carriers of the recessive gene but may not develop the
disease." pg. 990
Reproductive health education for women who are HIV-positive includes recommending
which of the following contraceptives?
The female condom pg. 1000
What treatment option does the nurse anticipate for the patient with severe combined
immunodeficiency disease (SCID)?
Bone marrow transplantation pg. 992
A patient is suspected to have an immunodeficiency disorder. The physician orders a
nitroblue tetrazolium reductase (NTR) test to diagnose this patient. What does the nurse
suspect that this disorder is related to?
Phagocytic cells pg. 988
The nurse is working with a parent whose child has just been diagnosed with selective
immunoglobulin A deficiency. The parent asks the nurse, "Does this mean that my child
is going to die?" How should the nurse respond?
"Your child has a mild genetic immune deficiency caused by a lack of immunoglobulin
A, a type of antibody that protects against infections of the lining the mouth and
digestive tract." pg. 987
Which adverse effect(s) should the nurse closely monitor in a client who has secondary
immunodeficiencies due to immunosuppressive therapy?
Respiratory or urinary system infections pg. 994
A client presents to the clinic with complaints that he began to itch and break out in
hives after taking an aspirin this morning. What medication does the nurse anticipate
administering that blocks histamine receptors?
Diphenhydramine pg. 1047
A client has been having joint pain and swelling in the left foot and is diagnosed with
rheumatoid arthritis. The symptoms began suddenly without any identifiable cause, and
the client has significant joint destruction. What type of disease is this considered?
Autoimmune pg. 1054
A client presents at the clinic with an allergic disorder. The client asks the nurse what an
"allergic disorder" means. What would be the nurse's best response?
"It is a hyperimmune response to something in the environment that is usually
harmless." pg. 1030
A client is prescribed antihistamines, and asks the nurse about administration and
adverse effects. The nurse should advise the client to avoid:
Avoid alcohol pg. 1041
A majority of clients with CVID develop which type of anemia?
Pernicious pg. 990
Ataxia refers to
uncoordinated muscle movement. pg. 992
The nurse is caring for a young client who has agammaglobulinemia. The nurse is
teaching the family how to avoid infection at home. Which statement by the family
indicates that additional teaching is needed?
"I can take my child to the beach, as long as we play in the sand rather than swim in the
water." pg. 995
The majority of patient with primary immunodeficiency are in which age group?
Younger than 20 pg. 987
When administering intravenous gamma globulin infusion, the nurse recognizes that
which of the following complaints, if reported by the client, may indicate an adverse
effect of the infusion?
Flank pain pg. 991
Telangiectasia is the term that refers to
Vascular lesions caused by dilated blood vessels pg. 992
A nurse knows that more than 50% of clients with CVID develop the following disorder.
pernicious anemia pg. 990
A client is diagnosed with common variable immunodeficiency (CVID). When assessing
the client for possible infection, which of the following would the nurse identify as a least
likely cause?
Pneumocystis jiroveci pneumonia pg. 990
A nurse educator is preparing to discuss immunodeficiency disorders with a group of
fellow nurses. What would the nurse identify as the most common secondary
immunodeficiency disorder?
AIDS pg. 994
More than 50% of individuals with this disease develop pernicious anemia:
Common variable immunodeficiency (CVID) pg. 990
A treatment option for severe combined immune deficiency (SCID) includes
a stem cell transplant. pg. 992
A nurse is providing discharge teaching to a client who is immunosuppressed. Which
statement by the client indicates the need for additional teaching?
"I can eat whatever I want as long as it's low in fat." pg. 995
A client with common variable immunodeficiency (CVID) comes to the ED reporting
tingling and numbness in the hands and feet, muscle weakness, fatigue, and chronic
diarrhea. An assessment reveals abdominal tenderness, weight loss, and loss of
reflexes. A gastric biopsy shows lymphoid hyperplasia of the small intestine and spleen
as well as gastric atrophy. Based on these findings, what common secondary problem
has this client developed?
Pernicious anemia pg. 988
A client with lupus has had antineoplastic drugs prescribed. Why would the physician
prescribe antineoplastic drugs for an autoimmune disorder?
For their immunosuppressant effects pg. 982
A client has begun sensitivity testing to determine the allergen which caused an
anaphylactic reaction 3 weeks ago. In scratch testing, which part of the body is more
sensitive to allergens?
back pg. 1035
The nurse is admitting a client to the unit with a diagnosis of ataxia-telangiectasia. A
clinical manifestation is telangiectasia. The nurse would recognize that the client is
exhibiting telangiectasia when assessing the presence of what?
Vascular lesions caused by dilated blood vessels pg. 992
A nurse is preparing to give a client an infusion of gamma globulin. The nurse knows to
stop the infusion if the client experiences which of the following symptoms? Choose all
that apply.
• Shaking chills
• Flank pain
• Tightness in the chest pg. 991
A client with severe combined immunodeficiency is to receive a hematopoietic stem cell
transplant. Which of the following would the nurse expect to be started?
Immunosuppressive agents pg. 992
Which assessment finding would the nurse expect to document for a client with ataxia-
telangiectasis?
Vascular lesions pg. 992
A child has just been diagnosed with a primary immune deficiency. The parents state,
"Oh, no. Our child has AIDS." Which response by the nurse would be most appropriate?
"Although AIDS is an immune deficiency, your child's condition is different from AIDS."
pg. 994
A mother brings her young child to the clinic for an evaluation of an infection. The
mother states, "He's been taking antibiotics now for more than 2 months and still doesn't
seem any better. It's like he's always sick." During the history and physical examination,
which of the following would alert the nurse to suspect a primary immunodeficiency?
Ten ear infections in the past year pg. 989
A nurse is preparing an in-service presentation about primary immunodeficiencies.
When describing these conditions, which of the following would the nurse need to
integrate into the presentation?
Most cases are typically diagnosed in infancy pg. 987
There are major differences between primary and secondary immunodeficiencies.
Select the most accurate statement the nurse would use to explain the cause of a
secondary immunodeficiency.
"Your immune system was most likely affected by an underlying disease process." pg.
994
The nurse is teaching the client who has an immunodeficiency disorder how to avoid
infection at home. Which statement indicates that additional teaching is needed?
"I will be sure to eat lots of fresh fruits and vegetables every day." pg. 995
A nurse is reviewing treatment options with parents of an infant born with severe
combined immunodeficiency disease (SCID). The nurse recognizes that the parents
understand the teaching based on which statement?
"We could have our 10-year-old daughter tested, as the ideal stem cell donor is a
human leukocyte antigen-identical sibling." pg. 992
A client is scheduled to receive an intravenous immunoglobulin (IVIG) infusion. The
client asks the nurse about the infusion's administration and its adverse effects. Which
condition should the nurse instruct this client to report immediately?
Tickle in the throat pg. 991
A client with rheumatoid arthritis informs the nurse that since he has been in remission
and not having any symptoms, he doesn't need to take his medication any longer. What
is the best response by the nurse?
"It is important that you continue to take your medication to avoid an acute
exacerbation." pg. 1056
Which allergic reaction is potentially life threatening?
angioedema pg. 1048
A client is presenting an anaphylactic response to unknowingly ingesting nuts at a
family celebration. What type of hypersensitivity did this client exhibit?
Type I pg. 1032
When a nurse infuses gamma globulin intravenously, the rate should not exceed
3 mL/min. pg. 991
A client with ataxia-telangiectasia is admitted to the unit. The nurse caring for the client
would expect to see what included in the treatment regimen?
IV gamma globulin administration pg. 992
Ataxia-telangiectasia is a disorder characterized by the following. Choose all that apply.
Neurologic symptoms that usually occur before 5 years of age
IgA deficiency in 40% of individuals pg. 992

nurse is reinforcing teaching with an assistive personnel about standard precautions


when caring for a client who has vancomycin resistant enterococcus of the urine. Which
of the following images of personal protective equipment should the nurse recommend
the AP to use when caring for this client?
Gloves
A nurse is reinforcing teaching with a client who has a recent diagnosis of Raynaud's
disease about preventing the onset of manifestations. Which of the following statements
by the client indicates an understanding of the teaching?
I should not smoke
A nurse in a providers office is reinforcing teaching with a client who has a new
diagnosis of rheumatoid arthritis and a new prescription for naproxen tablets. Which of
the following statements by the client requires further teaching?
I can take this medication with aspirin.

You should avoid taking naproxen with any other NSAIDs, such as aspirin because this
can increase the risk for bleeding and gastrointestinal ulceration
A nurse is reinforcing discharge teaching with a partner of a client who has acquired
immune deficiency syndrome(AIDS). Which of the following statements by the client
partner indicates the need for further teaching?
I'll clean up blood spills immediately with hot water.

Blood should be cleaned up with a bleach solution while wearing gloves.


A nurse is caring for a client who test positive for the human immunodeficiency virus.
The client asks the nurse, should I tell my partner I am HIV positive? Which of the
following is an appropriate nursing response?
It sounds like you are unsure what to say to your partner
A nurse is collecting data from a client who is experiencing an acute exacerbation of
rheumatoid arthritis. The nurse should anticipate that the clients effective joints will
require which of the following treatments?
Heat paraffin therapy applied to the clients hands
A nurse is reinforcing teaching with a client who has Raynaud's disease. Which of the
following information should the nurse include in the teaching?
Protect against the cold by wearing layers of clothing
A nurse is reinforcing teaching with a client who has aids about preventing infection
while at home. Which of the following instruction should the nurse include in the
teaching?
Wash genitalia using an anti-microbial soap
A nurse is collecting data from a client who has Kaposi's sarcoma. Which of the
following findings should the nurse expect with this condition?
Reddish purple skin lesions
A nurse is reviewing the laboratory results for a client who reports bilateral pain and
swelling in her finger joints, with stiffness in the morning. The nurse should recognize
that an increase in which of the following laboratory values can indicate arthritis?
Rheumatoid factor
A nurse is assisting with the care of a client who has systemic lupus
erythematosus(SLE). The client asks the nurse, what should I do to care for my dry
skin? Which of the following responses should the nurse make?
Apply lotion twice per day to dry skin
A nurse is reinforcing teaching with a client who has human immunodeficiency virus
about how the virus is transmitted. Which of the following statements should the nurse
include in the teaching?
HIV can be transmitted to anyone who has had contact with infected blood
A nurse is reinforcing teaching with a female client who has a new diagnosis of systemic
Lupus erythematosus(SLE) about factors that can trigger an exacerbation of SLE. The
nurse should determine that the client requires further teaching when she identifies
which of the following as a factor that can exacerbate SLE?
Exercise
A nurse is assisting with the care of a client who is two days postoperative period which
of the following findings should alert the nurse that the client is developing an infection?
Erythema at the incision site
A nurse is collecting data from a client who is concerned about the possibility of
contracting Lyme disease after being bitten by a tick. For which of the following early
manifestations of Lyme disease should the nurse monitor the client?
A progressive, circular rash
A nurse is reinforcing teaching with a client who has tuberculosis about a new
prescription for rifampin. Which of the following statements by the client indicates an
understanding of the teaching?
I can expect this medication to turn my skin orange
A nurse is reinforcing teaching with a client who has tested positive for an allergy to dust
about how to reduce her exposure to the allergen. The nurse should determine that the
client understands how to reduce her exposure to this Allergen when she states which
of the following?
I will install an electrostatic filter in my furnace
A nurse is reinforcing teaching with an assistive personnel who is caring for a client who
has active pulmonary tuberculosis, was placed on airborne precautions, and is
scheduled for a chest x-ray. Which of the following instruction should the nurse include
in the teaching?
Have the client wear a surgical mask
A nurse is collecting data from a client who has systemic scleroderma. Which of the
following findings should the nurse expect?
Skin tightening
A nurse is reinforcing teaching with a client who has HIV positive about the early
manifestations of acquired immune deficiency syndrome(AIDS). Which of the following
statements should the nurse include in the teaching?
You can expect a persistent fever and swollen glands
A nurse is reinforcing teaching with a client who is being treated for General to warts.
Which of the following statements indicates that the client understands how to prevent
transmission of his STI?
I will bring my sexual partner in for treatment
A nurse is reinforcing teaching with a client who has systemic lupus erythematosus.
Which of the following instruction should the nurse plan to include in the teaching?
Monitor your body temperature and report any elevations promptly
A nurse is reinforcing teaching with a client who has a diagnosis of hepatitis A. Which of
the following statements by the client indicates an understanding of the teaching?
I should stop eating raw clams
A nurse is preparing to administer a Mantoux skin test to a client. The nurse should
inform the client that the purpose of a MANTOUX skin test using purified protein
derivative(PPD) is to do which of the following actions?
Identify if a client has been infected with Mycobacterium tuberculosis
A nurse is reinforcing teaching With a client about the manifestations of an allergic
reaction. The nurse should explain that histamine release causes which of the following
reactions?
Increased mucus secretion
A nurse is collecting data from a client who had radioallergosorbent (RAST) testing
completed due to seasonal allergies. The nurse should anticipate an elevation in which
of the following laboratory values?
IgE (immunoglobulin E)
A nurse is assisting with the care of a client who has human immunodeficiency virus.
Which of the following types of isolation so the nurse implement to prevent transmission
of HIV?
Standard precautions
A nurse is collecting data from a client who has an exacerbation of herpes zoster.
Which of the following manifestations of the clients skin should the nurse expect to see?
Unilateral, localized, nodular skin lesions
A nurse is reinforcing teaching with a client who has genital herpes about self
management. Which of the following instructions should the nurse include in the
teaching?
Poor running water over the lesions when urinating
A nurse is reinforcing teaching with a client who has aids about the transmission of
pneumocystis jiroveci pneumonia (PCP).
PCP results from an impaired immune system

37. The 26-year-old female client is complaining of a low-grade fever, arthralgias,


fatigue, and a facial rash. Which laboratory tests should the nurse expect the HCP
to order if SLE is suspected?
1. Complete metabolic panel and liver function tests.
2. Complete blood count and antinuclear antibody tests.
3. Cholesterol and lipid profile tests.
4. Blood urea nitrogen and glomerular filtration tests.
2. No single laboratory test diagnoses
SLE, but the client usually presents
with moderate to severe anemia,
thrombocytopenia, leukopenia, and a
positive antinuclear antibody.
38. The client diagnosed with SLE is being discharged from the medical unit. Which
discharge instructions are most important for the nurse to include? Select all that
apply.
1. Use a sunscreen of SPF 30 or greater when in the sunlight.
2. Notify the HCP immediately when developing a low-grade fever.
3. Some dyspnea is expected and does not need immediate attention.
4. The hands and feet may change color if exposed to cold or heat.
5. Explain the client can be cured with continued therapy.
38. 1. Sunlight or UV light exposure has
been shown to initiate an exacerba
tion of SLE, so the client should
be taught to protect the skin when
in the sun.

2. A fever may be the first indication of


an exacerbation of SLE.

4. Raynaud's phenomenon is a condition in


which the digits of the hands and feet
turn red, blue, or white in response to
heat or cold and stress. It occurs with
some immune inflammatory processes.
39. The nurse is developing a care plan for a client diagnosed with SLE. Which goal is
priority for this client?
1. The client will maintain reproductive ability.
2. The client will verbalize feelings of body-image changes.
3. The client will have no deterioration of organ function.
4. The client's skin will remain intact and have no irritation.
39.
3. SLE can invade and destroy any body
system or organ. Maintaining organ
function is the primary goal of SLE
treatment.
40. The nurse is admitting a client diagnosed with R/O SLE. Which assessment data
observed by the nurse support the diagnosis of SLE?
1. Pericardial friction rub and crackles in the lungs.
2. Muscle spasticity and bradykinesia.
3. Hirsutism and clubbing of the fingers.
4. Somnolence and weight gain.
40. 1. SLE can affect any organ. It can cause
pericarditis and myocardial ischemia as
well as pneumonia or pleural effusions.
41. The client diagnosed with an acute exacerbation of SLE is prescribed high-dose
steroids. Which statement best explains the scientific rationale for using high-dose
steroids in treating SLE?
1. The steroids will increase the body's ability to fight the infection.
2. The steroids will decrease the chance of the SLE spreading to other organs.
3. The steroids will suppress tissue inflammation, which reduces damage to organs.
4. The steroids will prevent scarring of skin tissues associated with SLE.
3. The main function of steroid medica
tions is to suppress the inflammatory
response of the body.
42. The nurse enters the room of a female client diagnosed with SLE and finds the
client
crying. Which statement is the most therapeutic response?
1. "I know you are upset, but stress makes the SLE worse."
2. "Please explain to me why you are crying."
3. "I recommend going to an SLE support group."
4. "I see you are crying. We can talk if you would like."
4. The nurse stated a fact, "You are
crying," and then offered self by
saying "Would you like to talk?" This
addresses the nonverbal cue, crying,
and is a therapeutic response.
43. The nurse is assessing a client with cutaneous lupus erythematosus. Which
intervention should be implemented?
1. Use astringent lotion on the face and skin.
2. Inspect the skin weekly for open areas or rashes.
3. Dry the skin thoroughly by patting.
4. Apply anti-itch medication between the toes.
3. The skin should be washed with
mild soap, rinsed, and patted dry.
Rubbing can cause abrasions and skin
breakdown.
44. The nurse is caring for clients on a medical floor. Which client should be assessed
first?
1. The client diagnosed with SLE who is complaining of chest pain.
2. The client diagnosed with MS who is complaining of pain at a "10."
3. The client diagnosed with myasthenia gravis who has dysphagia.
4. The client diagnosed with GB syndrome who can barely move his toes.
44. 1. Chest pain should be considered a
priority regardless of the admitting
diagnosis. Clients diagnosed with SLE
can develop cardiac complications.
45. The nurse and a female unlicensed assistive personnel (UAP) are caring for a group
of clients on a medical floor. Which action by the UAP warrants immediate
intervention by the nurse?
1. The UAP washes her hands before and after performing vital signs on a client.
2. The UAP dons sterile gloves prior to removing an indwelling catheter from a
client.
3. The UAP raises the head of the bed to a high Fowler's position for a client about
to eat.
4. The UAP uses a fresh plastic bag to get ice for a client's water pitcher.
2. The UAP can remove an indwelling
catheter with nonsterile gloves. This is
a waste of expensive equipment. The
nurse is responsible for teaching UAPs appropriate use of equipment and
supplies and cost containment.
46. The client recently diagnosed with SLE asks the nurse, "What is SLE and how did I
get it?" Which statement best explains the scientific rationale for the nurse's
response?
1. SLE occurs because the kidneys do not filter antibodies from the blood.
2. SLE occurs after a viral illness as a result of damage to the endocrine system.
3. There is no known identifiable reason for a client to develop SLE.
4. This is an autoimmune disease that may have a genetic or hormonal component.
4. There is evidence for familial and hor
monal components to the development
SLE. SLE is an autoimmune disease
process in which there is an exagger
ated production of autoantibodies.
47. The client diagnosed with an acute exacerbation of SLE is being discharged with a
prescription for an oral steroid which will be discontinued gradually. Which
statement is the scientific rationale for this type of medication dosing?
1. Tapering the medication prevents the client from having withdrawal symptoms.
2. So thyroid gland starts working, because this medication stops it from working.
3. Tapering the dose allows the adrenal glands to begin to produce cortisol again.
4. This is the health-care provider's personal choice in prescribing the medication.
3. Tapering steroids is important because
the adrenal gland stops producing cor
tisol, a glucocorticosteroid, when the
exogenous administration of steroids
exceeds what normally is produced.
The functions of cortisol in the body
are to regulate glucose metabolism and
maintain blood pressure.
48. The nurse is discussing autoimmune diseases with a class of nursing students.
Which
signs and symptoms are shared by rheumatoid arthritis (RA) and systemic lupus
erythematosus (SLE)?
1. Nodules in the subcutaneous layer and bone deformity.
2. Renal involvement and pleural effusions.
3. Joint stiffness and pain.
4. Raynaud's phenomenon and skin rash.
3. Joint stiffness and pain are symptoms
occuring in both diseases.
49. The school nurse is preparing to teach a health class to ninth graders regarding
sexually transmitted diseases. Which information regarding acquired
immunodeficiency syndrome (AIDS) should be included?
1. Females taking birth control pills are protected from becoming infected
with HIV.
2. Protected sex is no longer an issue because there is a vaccine for the HIV virus.
3. Adolescents with a normal immune system are not at risk for developing AIDS.
4. Abstinence is the only guarantee of not becoming infected with sexually
transmitted HIV.
4. Abstinence is the only guarantee the
client will not contract a sexually trans
mitted disease, including AIDS. An
individual who is HIV negative in a
monogamous relationship with another
individual who is HIV negative and
committed to a monogamous relation
ship is the safest sexual relationship.
50. The nurse is admitting a client diagnosed with protein-calorie malnutrition
secondary to AIDS. Which intervention should be the nurse's first intervention?
1. Assess the client's body weight and ask what the client has been able to eat.
2. Place in contact isolation and don a mask and gown before entering the room.
3. Check the HCP's orders and determine what laboratory tests will be done.
4. Teach the client about total parenteral nutrition and monitor the subclavian
IV site.
50. 1. The client has a malnutrition syndrome.
The nurse assesses the body and what
the client has been able to eat.
51. The client diagnosed with AIDS is complaining of a sore mouth and tongue. When
the nurse assesses the buccal mucosa, the nurse notes white, patchy lesions covering
the hard and soft palates and the right inner cheek. Which interventions should the
nurse implement?
1. Teach the client to brush the teeth and patchy area with a soft-bristle toothbrush.
2. Notify the HCP for an order for an antifungal swish-and-swallow medication.
3. Have the client gargle with an antiseptic-based mouthwash several times a day.
4. Determine what types of food the client has been eating for the last 24 hours.
2. This most likely is a fungal infection
known as oral candidiasis, commonly
called thrush. An antifungal medication
is needed to treat this condition.
52. Which type of isolation technique is designed to decrease the risk of transmission of
recognized and unrecognized sources of infections?
1. Contact Precautions.
2. Airborne Precautions
3. Droplet Precautions.
4. Standard Precautions.
4. Standard Precautions are used for all
contact with blood and body secretions.
53. The nurse is describing the HIV virus infection to a client who has been told he is
HIV positive. Which information regarding the virus is important to teach?
1. The HIV virus is a retrovirus, which means it never dies as long as it has a host to
live in.
2. The HIV virus can be eradicated from the host body with the correct medical
regimen.
3. It is difficult for the HIV virus to replicate in humans because it is a
monkey virus.
4. The HIV virus uses the client's own red blood cells to reproduce the virus
in the body.
53. 1. Retroviruses never die; the virus
may become dormant, only to be
reactivated at a later time.
54. The client who has engaged in needle-sharing activities has developed a flu-like
infection. An HIV antibody test is negative. Which statement best describes the
scientific rationale for this finding?
1. The client is fortunate not to have contracted HIV from an infected needle.
2. The client must be repeatedly exposed to HIV before becoming infected.
3. The client may be in the primary infection phase of an HIV infection.
4. The antibody test is negative because the client has a different flu virus.
3. The primary phase of infection ranges
from being asymptomatic to severe
flu-like symptoms, but during this
time, the test may be negative although
the individual is infected with HIV.
55. The nurse caring for a client who is HIV positive is stuck with the stylet used to start
an IV. Which intervention should the nurse implement first?
1. Flush the skin with water and try to get the area to bleed.
2. Notify the charge nurse and complete an incident report.
3. Report to the employee health nurse for prophylactic medication.
4. Follow up with the infection control nurse to have lab work done.
55. 1. The nurse should attempt to flush the
skin and get the area to bleed. It is
hoped this will remove contaminated
blood from the body prior to infecting
the nurse.
56. The client on a medical floor is diagnosed with HIV encephalopathy. Which client
problem is priority?
1. Altered nutrition, less than body requirements.
2. Anticipatory grieving.
3. Knowledge deficit, procedures and prognosis.
4. Risk for injury.
4. Safety is always an issue with a client
with diminished mental capacity.
57. The client diagnosed with Pneumocystis pneumonia (PCP) is being admitted to the
intensive care unit. Which HCP's order should the nurse implement first?
1. Draw a serum for CD4 and complete blood count STAT.
2. Administer oxygen to the client via nasal cannula.
3. Administer trimethoprim-sulfamethoxazole, a sulfa antibiotic, IVPB.
4. Obtain a sputum specimen for culture and sensitivity.
2. Oxygen is a priority, especially with a
client diagnosed with a respiratory
illness.
58. Which intervention is an important psychosocial consideration for the client
diagnosed with AIDS?
1. Perform a thorough head-to-toe assessment.
2. Maintain the client's ideal body weight.
3. Complete an advance directive.
4. Increase the client's activity tolerance.
3. Clients diagnosed with AIDS should be
encouraged to discuss their end-of-life
issue with the significant others and to
put those wishes in writing. This is
important for all clients, not just those
diagnosed with AIDS.
59. The nurse on a medical floor is caring for clients diagnosed with AIDS. Which client
should be seen first?
1. The client who has flushed, warm skin with tented turgor.
2. The client who states the staff ignores the call light.
3. The client whose vital signs are T 99.9˚F, P 101, R 26, and BP 110/68.
4. The client who is unable to provide a sputum specimen.
59. 1. Flushed warm skin with tented turgor
indicates dehydration. The HCP
should be notified immediately for
fluid orders or other orders to correct
the reason for the dehydration.
60. The client diagnosed with AIDS is angry and yells at everyone entering the room,
and none of the staff members wants to care for the client. Which intervention is
most appropriate for the nurse manager to use in resolving this situation?
1. Assign a different nurse every shift to the client.
2. Ask the HCP to tell the client not to yell at the staff.
3. Call a team meeting and discuss options with the staff.
4. Tell one (1) staff member to care for the client a week at a time.
3. The health-care team should meet to
discuss ways to best help the client
deal with the anger being expressed,
and the staff should be consistent in
working with the client.
61. The charge nurse observes the primary nurse interacting with a client. Which action
by the primary nurse warrants immediate intervention by the charge nurse?
1. The nurse explains the IVP diuretic will make the client urinate.
2. The nurse dons nonsterile gloves to remove the client's dressing.
3. The nurse administers a medication without checking for allergies.
4. The nurse asks the UAP for help moving a client up in bed.
3. Checking for allergies is one (1) of the
five (5) rights of medication. Is it the
right drug? Even if the drug is the one
the HCP ordered, it is not the right
drug if the client is allergic to it. The
nurse should always assess a client's
allergies prior to administering any
medication.
62. The nurse in the emergency department is allergic to latex. Which intervention
should the nurse implement regarding the use of nonsterile gloves?
1. Use only sterile, nonlatex gloves for any procedure requiring gloves.
2. Do not use gloves when starting an IV or performing a procedure.
3. Keep a pair of nonsterile, nonlatex gloves in the pocket of the uniform.
4. Wear white cotton gloves at all times to protect the hands.
3. The nurse should be prepared to care
for a client at all times and should not
place himself or herself at risk because
the facility does not keep nonlatex
gloves available in the rooms. The
nurse should carry the needed equip
ment (nonlatex gloves) with him or her.
63. The client diagnosed with a bee sting allergy is being discharged from the
emergency
department. Which priority discharge instruction should be taught to the client?
1. Demonstrate how to use an EpiPen, an adrenergic agonist.
2. Teach the client to never go outdoors in the spring and summer.
3. Have the client buy diphenhydramine over the counter to use when stung.
4. Discuss wearing a Medic Alert bracelet when going outside.
63. 1. Clients who are allergic to bee sting
venom should be taught to keep an
EpiPen with them at all times and
how to use the device. This could
save their lives.
64. The client comes to the emergency department complaining of dyspnea and
wheezing
after eating at a seafood restaurant. The client cannot speak and has a bluish color
around the mouth. Which intervention should the nurse implement first?
1. Initiate an IV with normal saline.
2. Prepare to intubate the client.
3. Administer oxygen at 100%.
4. Ask the client about an iodine allergy.
3. The client is cyanotic with dyspnea and
wheezing. The nurse should adminis
ter oxygen first.
65. The client in the HCP's office is complaining of allergic rhinitis. Which assessment
question is important for the nurse to ask the client?
1. "What time of year do the symptoms occur?"
2. "Which over-the-counter medications have you tried?"
3. "Do other members of your family have allergies to animals?"
4. "Why do you think you have allergies?"
2. There are many over-the-counter
remedies available. Therefore, the
nurse should assess which medications
the client has tried and what medica
tions the client is currently taking.
66. The client asks the nurse, "Which time of the year is allergic rhinitis least likely to
occur?" Which statement is the nurse's best response?
1. "It is least likely to occur during the springtime."
2. "Allergic rhinitis is not likely to occur during the summer."
3. "It is least likely to occur in the early fall."
4. "Allergic rhinitis is least likely to occur in early winter."
4. Early winter is the beginning of
deciduous plants becoming dormant.
Therefore, allergic rhinitis is least
prevalent during this time of year.
67. The client is highly allergic to insect venom and is prescribed venom
immunotherapy. Which statement is the scientific rationale for this treatment?
1. Immunotherapy is effective in preventing anaphylaxis following a future sting.
2. Immunotherapy will prevent all future insect stings from harming the client.
3. This therapy will cure the client from having any allergic reactions in the future.
4. This therapy is experimental and should not be undertaken by the client.
67. 1. Immunotherapy does not cure the
problem. However, if immunotherapy
is done following a reaction, it provides
passive immunity to the insect venom
(similar to the way RhoGAM prevents a mother who is Rh negative from
building antibodies to the blood of a
baby who is Rh positive). This is the
purpose for immunotherapy in clients
who are allergic.
68. The client in the HCP's office has a red, raised rash covering the forearms, neck,
and
face and is experiencing extreme itching which is diagnosed as an allergic reaction to
poison ivy. Which discharge instructions should the nurse teach?
1. Tell the client never to scratch the rash.
2. Instruct the client in administering IM Benadryl.
3. Explain how to take a steroid dose pack.
4. Have the client wear shirts with long sleeves and high necks.
3. Clients with poison ivy are frequently
prescribed a steroid dose pack. The
dose pack has the steroid provided in
descending doses to help prevent
adrenal insufficiency.
69. The nurse is developing a care plan for a client diagnosed with allergic rhinitis.
Which independent problem has priority?
1. Ineffective breathing pattern.
2. Knowledge deficit.
3. Anaphylaxis.
4. Ineffective coping.
69. 1. This can be an independent or collabo
rative nursing problem. It is an airway
problem and has priority.
70. The nurse on a medical unit has received the morning shift report. Which client
should the nurse assess first?
1. The client who has a 0730 sliding-scale insulin order.
2. The client who received an initial dose of IV antibiotic at 0645.
3. The client who is having back pain at a "4" on a 1-to-10 scale.
4. The client who has dysphagia and needs to be fed.
2. This client has received an initial dose
of antibiotic IV and should be assessed
for tolerance to the medication within
30 minutes.
71. The nurse in the holding area of the operating room is assessing the client prior to
surgery. Which information warrants immediate intervention by the nurse?
1. The client is able to mark the correct site for the surgery.
2. The client can only tell the nurse about the surgery in lay terms.
3. The client is allergic to iodine and does not have an allergy bracelet.
4. The client has signed a consent form for surgery and anesthesia.
3. Iodine is the basic ingredient in
Betadine (povidone-iodine), which is a
common skin prep used for surgeries.
Therefore, the nurse should notify the
surgeon if the client has an allergy to
iodine.
72. The client in the emergency department begins to experience a severe anaphylactic
reaction after an initial dose of IV penicillin, an antibiotic. Which interventions
should the nurse implement? Select all that apply.
1. Prepare to administer Solu-Medrol, a glucocorticoid, IV.
2. Request and obtain a STAT chest x-ray.
3. Initiate the Rapid Response Team.
4. Administer epinephrine, an adrenergic blocker, SQ then IV continuous.
5. Assess for the client's pulse and respirations.
72. 1. Steroid medications decrease inflam
mation and therefore are one of the
treatments for anaphylaxis.
2. A STAT chest x-ray is not indicated
at this time.
3. The Rapid Response Team should be
called because this client will be in res
piratory and cardiac arrest very shortly.
4. Because of its ability to activate a
combination of alpha and beta recep
tors, epinephrine is the treatment of
choice for anaphylactic shock.
5. The first step in initiating cardiopul
monary resuscitation is to assess for a
pulse and respirations.
73. The client diagnosed with RA is being seen in the outpatient clinic. Which
preventive care should the nurse include in the regularly scheduled clinic visits?
1. Perform joint x-rays to determine progression of the disease.
2. Send blood to the lab for an erythrocyte sedimentation rate.
3. Recommend the flu and pneumonia vaccines.
4. Assess the client for increasing joint involvement.
3. RA is a disease with many immunologi
cal abnormalities. The clients have
increased susceptibility to infectious
disease, such as the flu or pneumonia,
and therefore vaccines, which are
preventive, should be recommended.
74. The client with RA has nontender, movable nodules in the subcutaneous tissue over
the elbows and shoulders. Which statement is the scientific rationale for the nodules?
1. The nodules indicate a rapidly progressive destruction of the affected tissue.
2. The nodules are small amounts of synovial fluid that have become crystallized.
3. The nodules are lymph nodes which have proliferated to try to fight the disease.
4. The nodules present a favorable prognosis and mean the client is better.
74. 1. The nodules may appear over bony
prominences and resolve simultane
ously. They appear in clients with the
rheumatoid factor and are associated
with rapidly progressive and destruc
tive disease.
75. The nurse is assessing a client diagnosed with RA. Which assessment findings
warrant immediate intervention?
1. The client complains of joint stiffness and the knees feel warm to the touch.
2. The client has experienced one (1)-kg weight loss and is very tired.
3. The client requires a heating pad applied to the hips and back to sleep.
4. The client is crying, has a flat facial affect, and refuses to speak to the nurse.
4. The client has the signs and symptoms
of depression. The nurse should
attempt to intervene with therapeutic
conversation and discuss these findings
with the HCP.
76. The client diagnosed with RA who has been prescribed etanercept, a tumor
necrosis
factor alpha inhibitor, shows marked improvement. Which instruction regarding the
use of this medication should the nurse teach?
1. Explain the medication loses its efficacy after a few months.
2. Continue to have checkups and lab work while taking the medication.
3. Have yearly magnetic resonance imaging to follow the progress.
4. Discuss the drug is taken for three (3) weeks and then stopped for a week.
2. The drug requires close monitoring to
prevent organ damage.
77. The client diagnosed with RA has developed swan-neck fingers. Which referral is
most appropriate for the client?
1. Physical therapy.
2. Occupational therapy.
3. Psychiatric counselor.
4. Home health nurse.
2. The occupational therapist assists the
client in the use of the upper half of
the body, fine motor skills, and activi
ties of daily living. This is needed for
the client with abnormal fingers.
78. The nurse is planning the care for a client diagnosed with RA. Which intervention
should be implemented?
1. Plan a strenuous exercise program.
2. Order a mechanical soft diet.
3. Maintain a keep-open IV.
4. Obtain an order for a sedative.
4. Sleep deprivation resulting from pain is
common in clients diagnosed with RA.
A mild sedative can increase the client's
ability to sleep, promote rest, and
increase the client's tolerance of pain.
79. The 20-year-old female client diagnosed with advanced unremitting RA is being
admitted to receive a regimen of immunosuppressive medications. Which question
should the nurse ask during the admission process regarding the medications?
1. "Are you sexually active, and, if so, are you using birth control?"
2. "Have you discussed taking these drugs with your parents?"
3. "Which arm do you prefer to have an IV in for four (4) days?"
4. "Have you signed an informed consent for investigational drugs?"
79. 1. Immunosuppressive medications are
considered class C drugs and should
not be taken while pregnant. These
drugs are teratogenic and carcinogenic,
and the client is only 20 years old.
80. Which client problem is priority for a client diagnosed with RA?
1. Activity intolerance.
2. Fluid and electrolyte imbalance.
3. Alteration in comfort.
4. Excessive nutritional intake.
3. The client diagnosed with RA has
chronic pain; therefore, alteration in
comfort is a priority problem.
81. The nurse is caring for clients on a medical floor. Which client should the nurse
assess first?
1. The client diagnosed with RA complaining of pain at a "3" on a 1-to-10 scale.
2. The client diagnosed with SLE who has a rash across the bridge of the nose.
3. The client diagnosed with advanced RA who is receiving antineoplastic drugs IV.
4. The client diagnosed with scleroderma who has hard, waxlike skin near the eyes.
3. Antineoplastic drugs can be caustic to
tissues; therefore, the client's IV site
should be assessed. The client should
be assessed for any untoward reactions
to the medications first.
82. The nurse and a licensed practical nurse are caring for clients in a rheumatologist's
office. Which task can the nurse assign to the licensed practical nurse?
1. Administer methotrexate, an antineoplastic medication, IV.
2. Assess the lung sounds of a client with RA who is coughing.
3. Demonstrate how to use clothing equipped with Velcro fasteners.
4. Discuss methods of birth control compatible with treatment medications.
3. The licensed practical nurse (LPN)
can demonstrate how to use adaptive
clothing.
83. The client with early-stage RA is being discharged from the outpatient clinic. Which
discharge instruction should the nurse teach regarding the use of nonsteroidal anti
inflammatory drugs (NSAIDs)?
1. Take with an over-the-counter medication for the stomach.
2. Drink a full glass of water with each pill.
3. If a dose is missed, double the medication at the next dosing time.
4. Avoid taking the NSAID on an empty stomach.
4. NSAID medications decrease
prostaglandin production in the stomach,
resulting in less mucus production, which
creates a risk for the development of
ulcers. The client should take the
NSAID with food.
84. The nurse is preparing to administer morning medications. Which medication
should the nurse administer first?
1. The pain medication to a client diagnosed with RA.
2. The diuretic medication to a client diagnosed with SLE.
3. The steroid to a client diagnosed with polymyositis.
4. The appetite stimulant to a client diagnosed with OA.
84. 1. Pain medication is important and
should be given before the client's
pain becomes worse.

A nurse is teaching an assistive personnel about standard precautions when caring for
a client who has vancomycin resistant Enterococcus of the urine. Which of the following
images of personal protection equipment should the nurse recommend the AP to use
when caring for this client?
The nurse should don clean gloves when caring for a client who has vancomycin-
resistant enterococcus of the urine. This protects the nurse form coming in contact with
bodily fluids contaminated with the bacteria of the client.
A nurse is teaching a client who has genital herpes about self-management. Which of
the following instructions should the nurse include in the teaching?
"Apply a warm compress to the lesions."
A nurse is planning discharge teaching for a client who has systemic lupus
erythematosus (SLE). Which of the following instructions should the nurse plan to
include?
"Monitor your body temperature and report any elevations promptly."
A nurse is providing discharge teaching to a client who has AIDS about preventing
infection while at home. Which of the following instructions should the nurse include in
the teaching.
"Wash your genitalia using an antimicrobial soap"
A nurse is caring for a client who had radioallergosorbent (RAST) testing completed due
to seasonal allergies. The nurse should anticipate and elevation in which of the
following laboratory tests?
IgE (immunoglobulin E)
A nurse is preparing to administer a Mantoux skin test to a client. The nurse should
inform the client that the purpose of a Mantoux skin test using purified protein derivative
(PPD) is to do which of the following actions?
Identify if a client has been infected with mycobacterium tuberculosis
A nurse in a provider's office is providing teaching to a client who has a recent diagnosis
of rheumatoid arthritis and has a new prescription for naproxen tablets. Which of the
following statements by the client requires further teaching?
"I can take this medication with aspirin."
A nurse is caring for a client who has human immunodeficiency virus (HIV). The client
asks the nurse, "should I tell my partner that I am HIV positive?" Which of the following
statements should the nurse give?
"It sounds like you are unsure what to say to your partner."
A nurse is teaching a client who has Raynaud's disease. Which of the following
information should the nurse include in the teaching?
Protect against the cold by wearing layers of clothing.
A nurse is reviewing the laboratory results for a client who reports bilateral pain and
swelling in her finger joints, with stiffness in the morning. The nurse should recognize
that an increase in which of the following laboratory tests can indicate arthritis?
Rheumatoid factor
A nurse is caring for a client who has systemic lupus erythematosus (SLE) and is
concerned abouth the skin lesions on her face and neck. The client asks the nurse,
"What should I do about these spots?" Which of following responses should the nurse
give?
"Apply moisturizer after bathing the lesions with warm water."
A nurse is teaching a female client who has a new diagnosis of systemic lupus
erythematosus (SLE) about factors that can trigger an exacerbation of SLE. The nurse
should determine that the client needs more teaching when she identifies which of the
following as a factor that can exacerbate SLE?
Exercise
A nurse is caring for a client who is 2 days postoperative. Which of the following findings
should alert the nurse that the client is developing an infection?
Erythema at the incision site
A nurse is monitoring a newly licensed nurse who is caring for a client. The client has
active pulmonary tuberculosis, was placed on airborne precautions, and is scheduled
for a chest x-ray. The nurse should instruct the newly licensed nurse to take which of
the following actions?
Have the client wear a surgical mask
A nurse is providing discharge instructions to a client who is being treated for genital
warts. Which of the following statements indicates that the client understands how to
prevent transmission of the sexually transmitted infection (STI)?
"I will bring my sexual partner in for treatment."
A nurse is teaching a client who has AIDS about the transmission of Pneumocystis
jiroveci pneumonia (PCP). Which of the following information should the nurse include
in the teaching?
"PCP results from an impaired immune system."
A nurse is teaching a client about manifestations of an allergic reaction. The nurse
should explain that histamine release causes which of the following reactions?
Increased mucus secretion
A nurse is teaching a client who has tuberculosis about a new prescription for rifampin.
Which of the following statements by the client indicates an understanding of the
teaching?
"I can expect this medication to turn my skin orange."
A nurse is caring for a client who is concerned about the possibility of contracting Lyme
disease after receiving a tick bite. For which of the following early manifestations of
Lyme disease should the nurse assess the client?
A progressive, circular rash
A nurse is assessing a client who has an exacerbation of herpes zoster. Which of the
following manifestations of the client's skin should the nurse expect?
Unilateral, localized, nodular skin lesions
A nurse is assessing a client who has Kaposi's sarcoma. Which of the following findings
should the nurse expect?
Reddish-purple skin lesions
A nurse is assessing a client who has systemic scleroderma. Which of the following
findings should the nurse expect?
Finger contractures
A nurse is teaching a client who has tested positive for an allergy to dust. The nurse
should determine that the client understands how to reduce her exposure to this
allergen when she states which of the following?
"I will apply a mattress cover to my bed."
A nurse is teaching a client who has human immunodeficiency virus about how the virus
is transmitted. Which of the following statements should the nurse include the teaching?
"HIV can be transmitted to anyone who has had contact with the infected blood."
A nurse is teaching a client who has human immunodeficiency virus about the early
manifestations of acquired immune deficiency syndrome. Which of the following
statements should the nurse include in the teaching?
"You can expect a persistent fever and swollen glands."
A nurse is providing teaching to a client who has a diagnosis of Hepatitis A. Which of
the following statements by the client indicates an understanding of the teaching?
"I should stop eating raw clams."
A nurse is caring for a client who is experiencing an acute exacerbation of rheumatoid
arthritis. The nurse should anticipate that the client's affected joints will requires which of
the following treatments?
Heat paraffin therapy applied to the client's joints
A nurse is teaching a client who was recently diagnosed with Raynaud's disease about
preventing the onset of manifestations. Which of the following statements by the client
indicates an understanding of the teaching?
I should not smoke.
A nurse is caring for client who has human immunodeficiency virus. Which of the
following types of isolation should the nurse implement to prevent transmission of HIV?
Standard Precautions
A nurse is providing discharge teaching to the partner of a client who has acquired
immune deficiency syndrome. Which of the following statements by the client's partner
indicate the need for further teaching?
"I'll clean up blood spills immediately with hot water."

a nurse is caring for a client who has human immunodeficiency virus (HIV). which of the
following types of isolation should the nurse implement to prevent the transmission of
HIV
a. protective isolation
b. droplet precaution
c. standard precautions
d. airborne precaution
standard precautions
a burse is performing a preoperative assessment of a client about to undergo a
cholecystectomy. the nurse should identify a risk for a latex allergy when the client
reports an allergy to which of the following foods
a. cabbage
b. oatmeal
c. milk
d. banana
banana
a nurse is providing discharge teaching to the partner of a client who has acquired
immune deficiency syndrome (AIDS). which of the following statements by the client's
partner indicates a need for further teaching
i will clean up blood spills immediately with hot water
a nurse is teaching a client who has raynaud's disease. which of the following pieces of
information should the nurse include in the teaching
protect against the cold by wearing layers of clothing
a nurse is planning care for a client who has acute systemic lupus erythematosus (SLE)
and is scheduled to begin treatment for systemic manifestations. which of the following
types of medications should the nurse plan to administer
a. corticosteroids
b. antimalarials
c. antidepressants
d. opioids
corticosteroids
a nurse is caring for a client who underwent radioallergosorbent (RAST) testing due to
seasonal allergies. the nurse should anticipate an elevation in which of the following
immunoglobulin laboratory values?
a. IgM
b. IgA
c. IgG
d. IgE
IgE
A nurse is assessing a 66 year old client during a routine physical examination. this is
the client's first clinic visit, and she does not have her medical records. when the nurse
asks if she has received the pneumococcal immunization, the client replies, i am not
sure, but it's been at least 5 years since I have had any immunization which of he
following responses should the nurse provide?
a. in case you had the immunization before, we can't give you another one
b. you will need a series of 3 injections
c. this immunization is unsafe for people over the age of 65 years old
d. let's go ahead and give you this immunization
let's go ahead and give you this immunization
a nurse is planning care for a client. which of the following interventions should the
nurse include in the plan
a. advance diet to soft
b. perform active and passive range-of-motion (ROM) exercises twice daily
c. apply compression garments 23 hr daily
d. restrict visitors
restrict visitors
a nurse is providing discharge instructions to a male client who is being treated for
genital warts. which of the following statements indicates that the client understands
how to prevent the transmission of this sexually transmitted infection (STI)
I will bring my sexual partner for treatment
a nurse is providing teaching to a client who has a diagnosis of hepatitis A. which of the
following statements by the client indicates an understanding of the teaching.
a. i am unable to donate blood
b. i will need to get a booster shot of serum globulin every year
c. i should stop eating raw clams
d. i can develop this disease by getting a tattoo
i should stop eating raw clams
a nurse is planning an educational program for a group of young adults about reducing
the risk of cervical cancer. which of the following interventions should the nurse include
get the human papillomavirus (HPV) immunization
a nurse is providing discharge teaching about foot care to a client who has diabetic
neuropathy. which of the following statements by the client demonstrates an
understanding of the teaching
i will wash my feet daily and apply lotion expect between my toes
a nurse is caring for a client who has pseudomembranous colitis due to a clostridium of
difficile infection. which of the following interventions is the nurse priority
a. performing hand hygiene before and after contact with the client
b. reducing client anxiety due to isolation procedures .
c. assisting the client in making nutritional choices
d. monitoring the client's intake and output
performing hand hygiene before and after contact with the client
a nurse is evaluating the injection site of a client who had a Mantoux skin test 48 hr ago.
the nurse finds 10mm of induration with slight redness. which of the following
conclusions should the nurse make.
a. the client has active tuberculosis
b. the client had an exposure to tuberculosis
c. the nurse must re-evaluate the result in 24 hr
d. the test is negative for tuberculosis
the client had an exposure to tuberculosis
a nurse is providing discharge teaching to a client who has a new diagnosis of systemic
lupus erythematosus (SLE). which of the following statement by the client indicates an
understanding of the teaching
I will need to take methotrexate even if i'm in remission
a nurse is planning discharge teaching for a client who has systemic lupus
erythematosus (SLE). which of the following instructions should the nurse include
a. avoid the use of NSAIDS
b. stop taking the corticosteroids when your symptoms resolves
c. exposure to ultraviolet light will help control the skin rashes
d. monitor your temperature and report any elevations promptly
monitor your temperature and report any elevations promptly
a nurse in a provider's office is teaching a client with a recent diagnosis of rheumatoid
arthritis who has a new prescription for naproxen tablets. which of the following
statements by the client indicates the need for further teaching
a. after taking tis medication for 4 weeks i will start to notice relief in my joint
b. i can take an antacid with this medication for indigestion
c. i can take this medication with aspirin
d. the naproxen goes down easier when i crush it and put it in applesauce
i can take this medication with aspirin
a nurse is teaching a client who was recently diagnosed with Raynaud's disease about
preventing the onset of manifestations. which of the following statements by the client
indicates an understanding of the teaching
a. i should limit my exposure to sunlight
b. i should avoid drinking alcohol
c. i should not smoke
d. i should limit of intake of foods that are high in purinesc
i should not smoke
a nurse is caring for a client who is concerned about the possibility of contracting lyme
disease after receiving a tick bite. for which of the following early manifestations of lyme
disease should the nurse assess the client
a. diffuse maculopapular rash
b. dyspnea
c. double vision
d. progressive circular rash
progressive circular rash
a nurse is teaching a client with systemic lupus erythematosus who has a new
prescription for prednisone. the nurse should instruct the client to monitor for which of
the following adverse effects of this medication
a. hypoglycemia
b. tendinitis
c. infection
d. weight loss
infection
a nurse is planning care for a client who has AIDS and has developed stomatitis. which
of the following interventions should the nurse include in the plan of care
a. rinse the mouth with chlorhexidine solution every 2 hr
b. limit fluid intake with meal
c. provide oral hygiene with a firm-bristled toothbrush after each meal
d. avoid salty foods
avoid salty foods
a nurse is teaching a female client with a new diagnosis of systemic lupus
erythematosus (SLE) about factors that can trigger an exacerbations of SLE. the nurse
should determine that the client requires further teaching if she identifies which of the
following as an exacerbation factor
a. exercise
b. pregnancy
c. infection
d. sunlight
exercise
a nurse is teaching a client who has genital herpes about self-management. which of
the following instructions should the nurse include in the teaching
a. use an alcohol-based soap to clean lesions
b. wear a condom during sexual activity when leisions are present
c. take sitz bath once per day
d. apply a warm compress to the lesions
apply a warm compress to the lesions
a nurse is caring for a male client who reports a thick urethral discharge. which of the
following actions should the nurse take
a. contact the client sexual partners
b. obtain a urethral specimen for culture
c. prepare to administer penicillin to the client
d. obtain blood for a rapid plasma reagin test
obtain a urethral specimen for culture
a nurse is assessing a client who has an exacerbation of herpes zoster. which of the
following manifestations of the client skin should the nurse expect
a. confluent, honey-colored, crusted lesions
b. a large, tender nodule located on a hair follicle
c. unilateral, localized nodular skin lesions
d. a fluid-filled vesicular rash in the genital region
unilateral localized nodular skin lesions
a nurse health nurse is planning care for a client who is receiving chemotherapy and
has neutropenia. which of the following foods should the nurse include in the client's
plan of care
a. soft-boiled egg
b. brie cheese made with unpasteurized milk
c. cold deli-meat sandwiches
d. baked chicken
baked chicken
a nurse is providing dietary teaching for a client with AIDS who has stomatitis of the
mouth. which of the following instructions should the nurse include in the teaching
a. you can suck on popsicles to numb your mouth
b. season food with spices instead of salt
c. avoid the use of a straw to drink liquids
d. eat foods at hot temperature
you can suck on popsicles to numb your mouth
a nurse is reviewing the laboratory data of a client who reports manifestations
suggesting systemic lupus erythematosus (SLE). the nurse should expect an increase
in which of the following parameters for client who has SLE
a. platelet count
b. RBC count
c. Hct
d. erythrocyte sedimentation rate (ESR)
erythrocytes sedimentation rate (ESR)
a nurse is caring for a client who is suspected to have tuberculosis. which of the
following findings should the nurse expect
a. recent weight gain
b. high fever
c. rhinitis
d. blood-streaked sputum
blood-streaked sputum
a nurse is teaching a client who has human immunodeficiency virus (HIV) about how
the virus is transmitted. which of the following statements should the nurse include in
the teaching
HIV can be transmitted to anyone who has had contact with infected blood
a nurse is teaching a client who has human immunodeficiency virus (HIV) about the
early manifestations of acquired immune deficiency syndrome (AIDS). which of the
following statements should the nurse include in the teaching
a. you can expect a persistent fever and swollen glands
b. you can expect an elevated white blood cell count---decrease
c. you can expect increased blood pressure and edema--hypotension
d. you can expect weight gain--weight loss
you can expect a persistent fever and swollen glands
a nurse is monitoring a newly licensed nurse who is caring for a client. the client has
active pulmonary tuberculosis, was places on airborne precautions, and is scheduled for
a chest x-ray. the nurse should instruct the newly licensed nurse to take which of the
following actions
a. have the client wear a surgical mask
b. wear a gown for protection from the client's infection
c. ask the radiology staff to perform a portable chest x-ray in the client's room
d. place an N-95 respirator on the client--- the nurse should wear this
have the client wear a surgical mask
a nurse is preparing to administer a mantoux skin test to a client. what is the purpose of
a mantoux skin test using purified protein derivative ?
to identify if a client has been infected with mycobacterium tuberculosis
a nurse is providing discharge teaching to a client who has HIV. which of the
instructions about infection prevention should the nurse include
a. avoid large gatherings of people
b. clean toothbrush by running through the dishwasher
c. change pet litter boxes with disposable gloves
d. consume fresh fruit and raw vegetables
e. avoid digging in the garden
avoid large gathering of people
clean toothbrush by running through the dishwasher
avoid digging in the garden
a nurse is caring for a client with clostridium defficile who has contact-isolation
precaution in place. which of the following actions should the nurse perform
use dedicated equipment for the client
a nurse is assessing a client who has kaposi sarcoma. which of the following findings
should the nurse expect
reddish -purple skin lesions
a nurse is teaching a client about the manifestations of allergic reactions. the release of
histamine causes which of the following reaction
a. increased mucus secretion
b. bronchial dilation
c. bradycardia
d. vertigo
increased mucus secretion
a nurse is teaching a client who has AIDS about the transmission of pneumocystis
jiroveci pneumonia (PCP). which of the following pieces of information should the nurse
include in the teaching
PCP results from an impaired immune system
a nurse is reviewing the laboratory result for a client who reports bilateral pain and
swelling in her finger joints, with stiffness in the morning. the nurse should recognize
that an increase in which of the following laboratory values can indicate arthritis
a. reticulocyte count
b. rheumatoid factor
c. direct Coomb's test
d. platelet count
rheumatoid factor
a nurse is providing teaching about foot care to a client who has diabetes mellitus.
which of the following pieces of information should the nurse include in the teaching
wash your feet daily using lukewarm water and saop
a nurse is providing discharge teaching to a client who has AIDS about preventing
infection while at home. which of the following instructions should the nurse include in
the teaching
wash your genitalia using an antimicrobial soap
wash toothbrush in dish washer ones per week
a nurse is working with an assistive personnel (AP) who is assigned to bathe a client
with hepes zoster. the AP asks the nurse if hepes zoster is contagious. which of the
following responses should the nurse make
herpes zoster is not contagious to people who have had chickenpox
a nurse is conducting dietary teaching for a client who has AIDS. which of the following
instructions should the nurse include in the teaching
a. discard leftover after 8 hrs
b. use a separate cutting board for poultry
c. thaw frozen foods at room temperature
d. store cold foods at 10 degree (50 degree) or less
use a separate cutting board for poultry
a nurse is teaching a client about preventing the transmission of hepatitis A. the nurse
should identify that hepatitis A is transmitted by which of the following routea
a. maternal-fatal
b. fecal-oral contamination
c. genital sexual contact
d. blood to blood
fecal-oral contamination
a nurse is teaching a client who has tuberculosis about a new prescription for rifampin.
which of the following statements by the client indicates an understanding of the
teaching
i can expect this medication to turn my skin orange
a nurse is caring for a client who has human immunodeficiency virus (HIV). the client
ask the nurse, should i tell my partner that i am HIV positive? which of the following
statements should the nurse provide
it sounds like you are unsure what to say to your partner
a nurse is caring for a client who is experiencing an acute exacerbation of rheumatoid
arthritis. the nurse should anticipate that the client's affected joints will require which of
the following treatment
a. an assistive device when the client is ambulating
b. heat paraffin therapy applied to the client's joint
c. gentle massage of the client hand
d. active range-of-motion exercises on the client's affected joint
heat paraffin therapy applied to the client's joint
a nurse is caring for a client who has systemic lupus erythematosus (SLE) and is
concerned about skin lesions on her face and neck. the client asks the nurse, what
should i do about these spot? which of the following responses should the nurse give
apply moisturizer after bathing the lesions with warm water
apply topical corticosteroids
pat dry lesions with washcloths to dry after washing
a nurse is admitting a client who has manifestations that suggest tuberculosis. which of
the following actions is the nurse's priority
a. initiate airborne precautions
b. administer antimicrobial therapy
c. tell the client that the infection will be communicable for 2-3 weeks from the start of
medication therapy
d. teach the client about the manifestations of tuberculosis
initiate airborne precautions
a nurse in a clinic is assessing a client who was diagnosed with mononucleosis 2 weeks
ago. which of the following findings should the nurse report to the provider immediately
a. headache and fatigue
b. swollen lymph nodes in the neck
c. abdominal pain in the left upper quadrant
d. fever and sore throat
abdominal pain in the left upper quadrant
a nurse is providing teaching to a client who has a new prescription for alprazolam.
which of the following items is a priority for teaching
this medication can affect your ability to drive or handle mechanical equipment
a nurse is caring for a client who is 2 days postoperative. which of the following findings
indicate the client is developing an infection
a. temperature 37.8 (100)
b. erythema at the incision site
c. WBC count 9000/mm3
d. pain reported a 6 on a scale of 0-10
erythema at the incision site
a nurse is teaching a client who tested positive for an allergy to dust. the nurse should
determine that the client understands how to reduce her exposure to this allergen
through which of the following statments
i will put a mattress cover on my bed
a nurse is assessing a client who has systemic scleroderma. which of the following
findings should the nurse expect
a. excessive salivation
b. finger contracture
c. periorbital edema
d. alopecia
finger contracture

A nurse is collecting data from a client who is concerned about the possibility of the
contracting Lyme disease after being bitten by a tick. For which of the following early
manifestations of Lyme disease should the nurse monitor the client?
A progressive, circular rash
A nurse is reinforcing teaching with a client who has AIDS about preventing infection
while at home. Which of the following instructions should the nurse include in the
teaching?
Wash genitalia using an antimicrobial soap
A nurse is reinforcing discharge teaching with the partner of a client who has acquired
immune deficiency syndrome (AIDS). Which of the following statements by the client's
partner indicates the need for further teaching?
I'll clean up blood spills immediately with hot water.
A nurse is collecting data from a client who has Kaposi's sarcoma. Which of the
following findings should the nurse expect with this condition?
Reddish-purple skin lesions
A nurse is reinforcing teaching with a client who has a recent diagnosis of Raynaud's
disease about preventing the onset of manifestations. Which of the following statements
by the client indicates an understanding of the teaching?
I should not smoke
A nurse in a provider's office is reinforcing teaching with a client who has a new
diagnosis of rheumatoid arthritis and a new prescription for naproxen tablets. Which of
the following statements by the client requires further teaching?
I can take this medication with Aspirin
A nurse is reinforcing teaching with a client who is HIV positive about the early
manifestations of acquired immune deficiency syndrome (AIDS). Which of the following
statements should the nurse include in the teaching?
You can expect a persistent fever and swollen glands
A nurse is caring for a client who test positive for the human immunodeficiency virus.
The client asks the nurse, "Should i tell my partner that i HIV positive?" Which of the
following is an appropriate nursing response?
It sounds like you are unsure what to say to your partner
A nurse is collecting data from a client who is experiencing an acute exacerbation of
rheumatoid arthritis. The nurse should anticipate that the client's affected joints will
require which of the following treatments?
Heat paraffin therapy applied to the client's hands
A nurse is reinforcing teaching with a client who is being treated for genital warts. Which
of the following statements indicates that the client understands how to prevent
transmission of his sexually transmitted infection (STI)?
I will bring my sexual partner in for treatment
A nurse is assisting with the care of a client who is 2 days postoperative. Which of the
following findings should alert the nurse that the client is developing an infection?
Erythema at the incision site
A nurse is assisting with the care of a client who has systemic lupus erythematosus
(SLE). The client asks the nurse, "What should i do to care for my dry skin?" Which of
the following responses should the nurse make?
Apply lotion twice per day to dry skin
A nurse is reinforcing teaching with a client about the manifestations of an allergic
reaction. The nurse should explain that histamine release causes which of the following
reactions?
Increased mucus secretion
A nurse is collecting data from a client who had radioallergosorbent (RAST) testing
completed due to seasonal allergies. The nurse should anticipate an elevation in which
of the following laboratory values?
IgE (immunoglobulin E)
A nurse is reinforcing teaching with a client who has AIDS about the transmission of
Pnuemocystis jiroveci pneumonia (PCP). Which of the following information should the
nurse include in the teaching?
PCP results from an impaired immune system
A nurse is reinforcing teaching with a client who has systemic lupus erythematosus
(SLE). Which of the following instructions should the nurse plan to include in the
teaching?
Monitor your body temperature and report any elevations promptly
A nurse is reinforcing teaching with an assistive personnel about standard precautions
when caring for a client who has vancomycin resistant Enterococcus of the urine. Which
of the following of personal protection equipment should the nurse recommend the AP
to use when caring for this client?
Gloves
A nurse is collecting data from a client who has systemic scleroderma. Which of the
following findings should the nurse expect?
Skin tightening
A nurse is reinforcing teaching with a client who has Raynaud's disease. Which of the
following information should the nurse include in the teaching?
Protect against the cold by wearing layers of clothing
A nurse is reviewing the laboratory results for a client who reports bilateral pain and
swelling in her finger joints, with stiffness in the morning. The nurse should recognize
that an increase in which of the following laboratory values can indicate arthritis?
Rheumatoid factor
A nurse is reinforcing teaching with a client who has genital herpes about self-
management. Which of the following instructions should the nurse include in the
teaching?
Pour running water over the lesions when urinating
A nurse is reinforcing teaching with a client who has tested positive for an allergy to dust
about how to reduce her exposure to the allergen. The nurse should determine that the
client understands how to reduce her exposure to the allergen when she states which of
the following?
I will install an electrostatic filter in my furnace
A nurse is reinforcing teaching with a client who has tuberculosis about a new
prescription for rifampin. Which of the following statements by the client indicates an
understanding of the teaching?
I can expect this medication to turn my skin orange
A nurse is reinforcing teaching with a client who has a diagnosis of hepatitis A. Which of
the following statements by the client indicates an understanding of the teaching?
I should stop eating raw clams
A nurse is collecting data from a client who has an exacerbation of herpes zoster.
Which of the following manifestations of the client's skin should the nurse expect to
see?
Unilateral, localized, nodular skin lesions
A nurse is reinforcing teaching with a female client who has a new diagnosis of systemic
lupus erythematosus (SLE) about factors that can trigger an exacerbation of SLE. The
nurse should determine that the client requires further teaching when she identifies
which of the following as a factor that can exacerbate SLE?
Exercise
A nurse is reinforcing teaching with an assistive personnel (AP) who is caring for a client
who has active pulmonary tuberculosis, was placed on airborne precautions, and is
scheduled for a chest x-ray. Which of the following instructions should the nurse include
in the teaching?
Have the client wear surgical mask
A nurse is reinforcing teaching with a client who has human immunodeficiency virus
about how the virus is transmitted. Which of the following statements should the nurse
include in the teaching?
HIV can be transmitted to anyone who has had contact with infected blood
A nurse is assisting with the care of a client who has human immunodeficiency virus.
Which of the following types of isolate should the nurse implement to prevent
transmission of HIV
Standard precautions
A nurse is preparing to administer a Mantoux skin test to a client. The nurse should
inform the client that the purpose of a Mantoux skin test using purified protein derivative
(PPD) is to do which of the following actions?
Identify if a client has been infected with mycobacterium tuberculosis.

A nurse is caring for a client who has human immunodeficiency virus (HIV). The client
asks the nurse, "Should I tell my partner that I am HIV positive?" Which of the following
statements should the nurse give?
a. "This is your decision alone."
b. "I would if I were you."
c. "It sounds like you are unsure what to say to your partner."
d. 'Your provider is required by law to notify your partner."
c. "It sounds like you are unsure what to say to your partner."
Rationale - This response uses the therapeutic communication tools of clarifying and
restatement. It identifies that the client is unsure about if or how to approach the issue of
being HIV positive with his partner, a common concern of clients due to fear of rejection.
This response shows that the nurse is open to further communication with the client and
encourages his expression of feelings.
A nurse is assessing a client who has Kaposi's sarcoma. Which of the following findings
should the nurse expect?
a. A nonproductive cough, fever, and shortness of breath
b. lesions of the retina that produce blurred vision.
c. Onset of progressive dementia.
d. Reddish-purple skin lesions.
d. Reddish-purple skin lesions
Rationale - Kaposi's sarcoma is commonly associated with AIDS and manifests as
hyper-pigmentated multi centric lesions that can be firm, flat, raised, or nodular.
Following biopsy, the lesions are treated with radiation and/or chemotherapy.
A nurse is caring for a client who had radioallergosorbent (RAST) testing completed due
to seasonal allergies. The nurse should anticipate an elevation in which of the following
laboratory tests?
a. IgM
b. IgA
c. IgG
d. IgE
d. IgE
Rationale - RAST testing involves measuring the quantity of IgE present in serum after
exposure to specific antigens selected on a basis of the client's symptom history. An
elevated IgE indicates a positive response and is common among clients who have a
history of allergic manifestations, anaphylaxis, and asthma.
A nurse is caring for a client who is 2 days postoperative. Which of the following findings
should alert the nurse that the client is developing an infection?
a. Temperature 37.8 C (100 F)
b. Erythema at the incision site
c. WBC count 9,000/mm^3
d. Pain reported as 6 on a scale of 0 to 10.
b. Erythema at the incision site.
Rationale - Redness, or erythema, at the incision site is an initial sign of a wound
infection and requires intervention by the nurse.
A nurse is teaching a client who was recently diagnosed with Raynaud's disease about
preventing the onset of manifestations. Which of the following statements by the client
indicates an understanding of the teaching?
a. "I should limit my exposure to sunlight."
b. "I should avoid drinking alcohol."
c. "I should not smoke."
d. "I should limit my intake of foods high in purine."
c. "I should not smoke."
Rationale - Raynaud's disease is a disorder of the blood vessels that supply blood to the
skin and cause the distal extremities to feel numb and cool in response to cold
temperatures or stress. During a Raynaud's attack, these arteries narrow, limiting blood
circulation to affected areas. Strong emotion or exposure to the cold causes these areas
to become white, due to a lack of blood flow in the area. They then turn blue, as tiny
blood vessels dilate to allow more blood to remain in the tissues. When the flow of blood
returns, the area becomes red and then later returns to normal color. This can cause
tingling, swelling, and painful throbbing. The attacks can last from minutes to hours. If
the condition progresses, blood flow to the area could become permanently decreased,
causing the fingers to become thin and tapered, with smooth, shiny skin and slow-
growing nails. If an artery becomes blocked completely, gangrene or ulceration of the
skin can occur. Smoking cessation, not just reduction, is an action the client should take
to prevent the onset of the manifestations of Raynaud's disease.
A nurse is assessing a client who has systemic scleroderma. Which of the following
findings should the nurse expect?
a. Excessive salivation
b. Finger contractures
c. Periorbital edema
d. Alopecia
b. Finger contractures
Rationale - Scleroderma is a chronic disease that can cause thickening, hardening, or
tightening of the skin, blood vessels, and internal organs. There are two types of
scleroderma: localized scleroderma, which mainly affects the skin, and systemic
scleroderma, which can affect internal organs. The manifestations include skin changes,
Raynaud's phenomenon, arthritis, muscle weakness, and dry mucous membranes. With
scleroderma, the body produces and deposits too much collagen, causing thickening
and hardening. In addition to the client's skin and subcutaneous tissues becoming
increasingly hard and rigid, the extremities stiffen and lose mobility. Contractors develop
with advanced systemic scleroderma unless clients follow a regimen of range-of-motion
and muscle-strengthening exercises.
A nurse is caring for a client who has systemic lupus erythematosus (SLE) and is
concerned about the skin lesions on her face and neck. The client asks the nurse,
"What should I do about these spots?" Which of the following responses should the
nurse give?
a. "Keep the lesions covered with a light sterile dressing when going outside."
b. "Rub lesions with a washcloth to dry after washing."
c. "Apply moisturizer after bathing the lesions with warm water."
d. "Apply antibiotic cream twice per day until scabs form on the lesions."
c. "Apply moisturizer after bathing the lesions with warm water."
Rationale - The nurse should instruct the client to clean, dry, and moisturize the skin
using warm (not hot) water, along with an unscented lotion.
A nurse teaching a client who has human immunodeficiency virus about how the virus is
transmitted. Which of the following statements should the nurse include in the teaching?
a. "HIV can be transmitted as soon as a person develops manifestations."
b. "HIV can be transmitted to anyone who has had contact with the infected blood."
c. "HIV is transmitted through the respiratory route through droplets."
d. "HIV is transmitted only during the active phase of the virus."
b. "HIV can be transmitted to anyone who has had contact with the infected blood."
Rationale - The concentration of the virus is highest in blood but also has been isolated
in other body fluids, including sputum, saliva, cerebrospinal fluid, urine and semen.
Clients who have HIV are cautioned to practice safe sex, avoid donating blood, and
abstain from sharing needles with others.
A nurse is caring for a client who has human immunodeficiency virus. Which of the
following types of isolation should the nurse implement to prevent transmission of HIV?
a. Protective isolation
b. Droplet isolation
c. Standard isolation
d. Airborne isolation
c. Standard isolation
Rationale - Standard precautions should be implemented with every client, to prevent
the spread of infection transmitted by direct or indirect contact with infectious blood or
body fluids. Because HIV is spread through blood and bodily fluid, standard precautions
are appropriate.
A nurse is teaching a client who has tested positive for an allergy to dust. The nurse
should determine that the client understands how to reduce her exposure to this
allergen when she states which of the following?
a. "I will begin vacuuming once a week."
b. Carpeting the entire hose will be very expensive, but it will be worth it."
c. I will apply a mattress cover to my bed."
d. "Installing curtains on my windows will help control the dust in the house."
c. "I will apply a mattress cover to my bed."
Rationale - The nurse should instruct the client to apply a hypoallergenic mattress cover
that can be zipped over her bed to control the amount of dust. The client should remove
the mattress cover periodically and machine wash to clean.
A nurse is caring for a client who is experiencing an acute exacerbation of rheumatoid
arthritis. The nurse should anticipate that the client's affected joints will require which of
the following treatments?
a. An assistive device to use when the client is ambulating.
b. Heat paraffin therapy applied to client's joints.
c. Gentle massage of the client's hands.
d. Active range-of-motion exercises on the client's affected joints.
b. Heat paraffin therapy applied to client's joints.
Rationale - The nurse should anticipate the use of heat paraffin to be prescribed as a
non-pharmacologic intervention. The elevated ESR indicates an acute inflammatory
process due to client's rheumatoid arthritis. The use of the warm paraffin relieves the
stiffness of the client's joints and provides comfort.
A nurse is providing discharge teaching to a client who has AIDS about preventing
infection while at home. Which of the following instructions should the nurse include in
the teaching?
a. "Wash your genitalia using an antimicrobial soap."
b. "Rinse your dishes with cold water."
c. "Clean your toothbrush once per month."
d. "Incorporate raw fruits and vegetables into your diet."
a. "Wash your genitalia using an antimicrobial soap."
Rationale - The nurse should instruct the client to bathe daily using an antimicrobial
soap to prevent the spread of infection. If bathing is not possible, washing the genitalia
using an antimicrobial soap is recommended.
A nurse is teaching a client who has tuberculosis about a new prescription for rifampin.
Which of the following statements by the client indicates an underling of the teaching?
a. "I should take this medication with food."
b. "I need to take B-complex vitamin while taking this medication."
c. "I can expect this medication to turn my skin orange."
d. I can expect this medication to make my vision blurry."
c. "I can expect this medication to turn my skin orange."
Rationale - The nurse should instruct the client to expect his skin and urine to turn a
reddish-orange tinge which taking rifampin.
A nurse is providing discharge teaching to the partner of a client who has acquired
immune deficiency syndrome. Which of the following statements by the client's partner
indicates the need for further testing?
a. "I will dispose soiled tissues in separate plastic bags."
b. "I'll clean up blood spills immediately with hot water."
c. "I know that hand washing is an important preventive measure."
d. "I will wash soiled clothes in hot water
b. "I'll clean up blood spills immediately with hot water."
Rationale - The client's partner should clean blood or potentially contaminated body
substances with a bleach solution and wear gloves when coming into contact with blood
products.
A nurse is caring for a client who is concerned about the possibility of contracting lyme
disease after receiving a tick bite. For which of the following early manifestations of
Lyme disease should the nurse asses the client?
a. A diffuse maculopapular rash
b. Dyspnea
c. Double vision
d. A progressive, circular rash
d. A progressive, circular rash
Rationale - Early Lyme Disease is characterized by fever, influenza-like manifestations,
and erythema migraines, a distinct progressive circular or bulls-eye rash that often
develops at the bite site, but can also develop at other sites, such as the thighs and
knees.
A nurse is providing teaching to a client who has a diagnosis of Hepatitis A. Which of
the following statements by the client indicates an understanding of the teaching?
a. "I am unable to donate blood."
b. "I will need to get a booster shot of immune serum globulin every year."
c. "I should stop eating raw clams."
d. "I can get this disease by getting a tattoo."
c. "I should stop eating raw clams."
Rationale - Hepatitis A is transmitted via the fecal-oral route through consumption of
contaminated fruits, vegetables, water, milk, or uncooked shellfish. Individuals who eat
raw or steamed shellfish are at increased risk for acquiring hepatitis A.
A nurse is assessing a client who has an exacerbation of herpes zoster. Which of the
following manifestations of the client's skin should the nurse expect?
a. Confluent, honey-colored, crusted lesions
b. Large, tender nodules located on a hair follicle
c. Unilateral, localized, nodular skin lesions
d. A fluid-filled vesicular rash in the genital region
c. Unilateral, localized, nodular skin lesions.
Rationale - Herpes zoster, or shingles, results from the reactivation of a dormant
varicella virus, It is the acute, unilateral inflammation of the dorsal root ganglion. The
infection typically develops in adults and produces localized vesicular lesions confined
to a dermatome. It produces unilateral, localized, nodular skin lesions.
A nurse is teaching a client who has AIDS about the transmission of Pneumocystis
jiroveci pneumonia (PCP). Which of the following information should the nurse include
in the teaching?
a. "PCP is sexually transmitted from person to person."
b. "You were most likely exposed to a contaminated surface, such as a drinking glass."
c. "PCP results from an impaired immune system."
d. "You may have contracted PCP from a family pet."
c. "PCP results from an impaired immune system."
Rationale - The nurse should explain that the organism that causes PCP exists as part
of the normal flora of the lungs and develops into a fungus. It becomes an aggressive
pathogen when the immune system is compromised and the infection results from an
impaired immune system.
A nurse is teaching a client about manifestations of an allergic reaction. The nurse
should explain that histamine release causes which of the following reactions?
a. Increased mucus secretion
b. Bronchial dilation
c. Bradycardia
d. Vertigo
a. Increased mucus secretion
Rationale - The nurse should instruct the client that increased mucus secretion is a
manifestation of histamine release. Histamine is the neurotransmitter the body produces
during an allergic reaction.
A nurse is teaching a client who has HIV about the early manifestations of AIDS. Which
of the following statements should the nurse include in the teaching?
a. "You can expect a persistent fever and swollen glands."
b. "You can expect an elevated white blood cell count."
c. "You can expect an increase in blood pressure and edema."
d. "You can expect weight gain."
a. "You can expect a persistent fever and swollen glands."
Rationale - Clients who have AIDS can have persistent fever, swollen glands, diarrhea,
weight loss, and fatigue. These manifestations indicate the onset of AIDS.
A nurse is teaching a client who has genital herpes about self-management. Which of
the following instructions should the nurse include in the teaching?
a. Use an alcohol-based to clean lesions.
b. Wear a condom during sexual activity when lesions are present.
c. Take a sitz bath once per day.
d. Apply a warm compress to the lesions.
d. Apply a warm compress to the lesions.
Rationale - The nurse should instruct the client to apply a warm compares to the lesions
to relieve discomfort.
A nurse in a provider's office is providing teaching to a client who has a recent diagnosis
of rheumatoid arthritis and has a new prescription for naproxen tablets. Which of the
following statements by the client requires further teaching?
a. "This medication will take 4 weeks for me to notice relief in my joints."
b. "I can take an antacid with this medication for indigestion."
c. "I can take this medication with aspirin."
d. "The naproxen goes down easier when i crush it and put it in applesauce."
c. "I can take this medication with aspirin."
Rationale - The nurse should instruct the client to avoid taking this medication with any
other NSAIDs, such as aspirin, because this can increase the risk for bleeding and
gastrointestinal ulceration.
A nurse is preparing to administer a Mantoux skin test to a client. The nurse should
inform the client that the purpose of a Mantoux skin test using purified protein derivative
(PPD) is to do which of the following actions?
a. Identify if a client lacks immunity to tuberculosis
b. Find out if a client has active tuberculosis
c. Decrease the hypersensitivity of the client's reaction to PPD
d. Identify if a client has been infected with mycobacterium tuberculosis.
d. Identify if a client has been infected with mycobacterium tuberculosis.
Rationale - The nurse should inform the client that the Mantoux skin test is used to
identify individuals who have been infected with mycobacterium tuberculosis.
A nurse is teaching a female client who has a new diagnosis of system lupus
erythematosus (SLE) about factors that can trigger an exacerbation of SLE. The nurse
should determine that the client needs more teaching when she identifies which of the
following as a factor that can exacerbate SLE?
a. Exercise
b. Pregnancy
c. Infection
d. Sunlight
a. Exercise
Rationale - SLE is a chronic autoimmune disease that develops when the immune
system becomes hyperactive and attacks healthy body tissue. This attack results in
generalized inflammation and the manifestations associated with the specific involved
tissues. Most clients who have SLE can follow an exercise program to increase the
aerobic capacity of cells and improve immune function, and the client should develop
such a program with her provider's assistance. This client needs additional teaching
about the importance of exercise to keep her muscles and joints active.
A nurse is providing discharge instructions to a client who is being treated for genital
warts. Which of the following statements indicates that the client understands how to
prevent transmission of the STI?
a. "I will bring my sexual partner in for treatment."
b. "Now that I've had my first dose of medicine, I can resume sexual activity"
c. "Once I have been treated, it is no longer necessary to use condoms."
d. Once treatment is completed and I am free of symptoms, I don't have to return to the
clinic."
a. "I will bring my sexual partner in for treatment."
Rationale - The client should bring his partner in to be screened for genital warts and
treated.
A nurse is planning discharge teaching for a client who has systemic lupus
erythematosus (SLE). Which of the following instructions should the nurse plan to
include.
a. "Avoid the use of NSAIDs
b. "Stop taking the corticosteroids when your symptoms resolve."
c. "Exposure to ultraviolet light with help control the skin rashes.
d. "Monitor your body temperature and report any elevations promptly."
d. "Monitor your body temperature and report any elevations promptly."
Rationale - SLE is a chronic autoimmune disorder the can affect any organ of the body.
With SLE, the body's immune system becomes hyperactive, forming antibodies that
attack tissues and organs, including the skin, joints, kidneys, brain, heart, lungs, and
blood. SLE is characterized by periods of exacerbation and remissions. The nurse
should teach the client to monitor body temperature and report any elevations promptly,
as fever can suggest either an exacerbation or a potentially life-threatening infection.
A nurse is monitoring a newly licensed nurse who is caring for a client. The client has
active pulmonary tuberculosis, was placed on airborne precautions, and is scheduled
for a chest x-ray. The nurse should instruct the newly licensed nurse to take which of
the following actions?
a. Have the client wear a surgical mask
b. Wear gown for protection from the client's infection.
c. Ask the radiology staff to perform a portable chest x-ray in the client's room.
d. Place an N95 respirator on the client.
a. Have the client wear a surgical mask.
Rationale - The nurse should instruct the client to wear a surgical mask. The mask will
protect anyone who comes into contact with the client, including the nurse.
The nurse is reviewing the laboratory results for a client who reports bilateral pain and
swelling in her finger joints, with stiffness in the morning. The nurse should recognize
that an increase in which of the following laboratory tests can indicate arthritis?
a. Reticulocyte count
b. Rheumatoid factor
c. Direct Coomb's test
d. Platelet count
b. Rheumatoid factor
Rationale - An increase in the client's rheumatoid factor can indicate rheumatoid arthritis
or other connective tissue diseases.
A nurse is teaching a client who has Raynaud's disease. Which of the following
information should the nurse include in the teaching?
a. Protect against the cold by wearing layers of clothing
b. Beging an exercise program of 2-mile walks once per week
c. Increase vitamin A in the diet
a. Protect against the cold by wearing layers of clothing.
Rationale - Clients who have Raynaud's disease are prone to attacks during cold
weather. Extreme cold can lead to tissue damage. Therefore, the client needs to be
protected with layers of clothing to promote warmth and increase circulation to the
extremities.

Which statement by a patient with systemic lupus erythematosus (SLE) indicates that
the patient has understood the nurse's teaching about the condition?
a. "I will exercise even if I am tired."
b. "I will use sunscreen when I am outside."
c. "I should take birth control pills to keep from getting pregnant."
d. "I should avoid aspirin or nonsteroidal antiinflammatory drugs."
ANS: B
Severe skin reactions can occur in patients with SLE who are exposed to the sun.
Patients should avoid fatigue by balancing exercise with rest periods as needed. Oral
contraceptives can exacerbate lupus. Aspirin and nonsteroidal antiinflammatory drugs
are used to treat the musculoskeletal manifestations of SLE.
A 25-year-old female patient with systemic lupus erythematosus (SLE) who has a facial
rash and alopecia tells the nurse, "I never leave my house because I hate the way I
look." An appropriate nursing diagnosis for the patient is
a. activity intolerance related to fatigue and inactivity.
b. impaired social interaction related to lack of social skills.
c. impaired skin integrity related to itching and skin sloughing.
d. social isolation related to embarrassment about the effects of SLE.
ANS: D
The patient's statement about not going anywhere because of hating the way he or she
looks supports the diagnosis of social isolation because of embarrassment about the
effects of the SLE. Activity intolerance is a possible problem for patients with SLE, but
the information about this patient does not support this as a diagnosis. The rash with
SLE is nonpruritic. There is no evidence of lack of social skills for this patient.
A new clinic patient with joint swelling and pain is being tested for systemic lupus
erythematosus. Which test will provide the most specific findings for the nurse to
review?
a. Rheumatoid factor (RF)
b. Antinuclear antibody (ANA)
c. Anti-Smith antibody (Anti-Sm)
d. Lupus erythematosus (LE) cell prep
ANS: C
The anti-Sm is antibody found almost exclusively in SLE. The other blood tests are also
used in screening but are not as specific to SLE
The health care provider has prescribed the following collaborative interventions for a
patient who is taking azathioprine (Imuran) for systemic lupus erythematosus. Which
order will the nurse question?
a. Draw anti-DNA blood titer.
b. Administer varicella vaccine.
c. Naproxen (Aleve) 200 mg BID.
d. Famotidine (Pepcid) 20 mg daily.
ANS: B
Live virus vaccines, such as varicella, are contraindicated in a patient taking
immunosuppressive drugs. The other orders are appropriate for the patient
Which result for a 30-year-old patient with systemic lupus erythematosus (SLE) is most
important for the nurse to communicate to the health care provider?
a. Decreased C-reactive protein (CRP)
b. Elevated blood urea nitrogen (BUN)
c. Positive antinuclear antibodies (ANA)
d. Positive lupus erythematosus cell prep
ANS: B
The elevated BUN and creatinine levels indicate possible lupus nephritis and a need for
a change in therapy to avoid further renal damage. The positive lupus erythematosus
(LE) cell prep and ANA would be expected in a patient with SLE. A drop in CRP shows
an improvement in the inflammatory process.
Which nursing action can the registered nurse (RN) delegate to unlicensed assistive
personnel (UAP) who are assisting with the care of a patient with scleroderma?
a. Monitor for difficulty in breathing.
b. Document the patient's oral intake.
c. Check finger strength and movement.
d. Apply capsaicin (Zostrix) cream to hands.
ANS: B
Monitoring and documenting patients' oral intake is included in UAP education and
scope of practice. Assessments for changes in physical status and administration of
medications require more education and scope of practice, and should be done by
licensed nurses
In teaching a patient with SLE about the disorder, the nurse knows that the
pathophysiology of SLE includes
a. circulating immune complexes formed from IgG autoantibodies reacting with IgG
b. an autoimmune T-cell reaction that results in destruction of the deep dermal skin
layer
c. immunologic dysfunction leading to chronic inflammation in the cartilage and muscles
d. the production of a variety of autoantibodies directed against components of the cell
nucleus
Correct answer: d
Rationale: Systemic lupus erythematosus (SLE) is characterized by the production of
many autoantibodies against nucleic acids (e.g., single-and double-stranded DNA),
erythrocytes, coagulation proteins, lymphocytes, platelets, and many other self-proteins.
Autoimmune reactions characteristically are directed against constituents of the cell
nucleus (e.g., antinuclear antibodies [ANAs]), particularly DNA. Circulating immune
complexes containing antibody against DNA are deposited in the basement membranes
of capillaries in the kidneys, heart, skin, brain, and joints. Complement is activated, and
inflammation occurs. The overaggressive antibody response is also related to activation
of B and T cells. The specific manifestations of SLE depend on which cell types or
organs are involved. SLE is a type III hypersensitivity response.
A female patient's complex symptomatology over the past year has culminated in a
diagnosis of systemic lupus erythematosus (SLE). Which of the patient's following
statements demonstrates the need for further teaching about the disease?

A. "I'll try my best to stay out of the sun this summer."


B. "I know that I probably have a high chance of getting arthritis."
C. "I'm hoping that surgery will be an option for me in the future."
D. "I understand that I'm going to be vulnerable to getting infections."
Correct answer: C. "I'm hoping that surgery will be an option for me in the future."
Rationale: SLE carries an increased risk of infection, sun damage, and arthritis. Surgery
is not a key treatment modality for SLE.
The pathophysiology of systemic lupus erthematosus (SLE) is characterized by
a. destruction of nucleic acids and other self-proteins by autoantibodies
b. overproduction of collagen that disrupts the functioning of internal organs
c. formation of abnormal IgG that attaches to cellular antigens, activating complement
d. increased activity of T-suppressor cells with B-cell hypoactivity, resulting in an
immunodeficiency
A. destruction of nucleic acids and other self-proteins by autoantibodies
A patient with newly diagnosed SLE asks the nurse how the disease will affect her life.
The best response by the nurse is
a. You can plan to have a near-normal life since SLE rarely causes death
b. it is difficult to tell because to disease is so variable in its severity and progression
c. life span is shortened somewhat in people with SLE, but the disease can be
controlled with long-term use of corticosteroids
d. most people with SLE have alternating periods of remissions and exacerbations with
rapid progression to permanent organ damage
B. it is difficult to tell because the disease is so variable in its severity and progression
During an acute exacerbation, a patient with SLE is treated with corticosteroids. The
nurse would expect the steroids to begin to be tapered when serum laboratory results
indicate
a. increased RBCs
b. decreased ESR
c. decreased anti-DNA
d. increased complement
C. decreased anti-DNA
Teaching that the nurse will plan for the patient with SLE includes
a. ways to avoid exposure to sunlight
b. increasing dietary protein and carbohydrate intake
c. the necessity of genetic counseling before planning a family
d. the use of no pharmacologic pain interventions instead of analgesics
A. ways to avoid exposure to sunlight

A client is suspected of having systemic lupus erythematous. The nurse monitors the
client, knowing that which of the following is one of the initial characteristic sign of
systemic lupus erythematous?
1. Weight gain
2. Subnormal temperature
3. Elevated red blood cell count
4. Rash on the face across the bridge of the nose and on the cheeks
Answer: 4
Rationale: Skin lesions or rash on the face across the bridge of the nose and on the
cheeks is an initial characteristic sign of systemic lupus erythematosus (SLE). Fever
and weight loss may also occur. Anemia is most likely to occur later in SLE.
The nurse is assisting in planning care for a client with a diagnosis of immune
deficiency. The nurse would incorporate which of the following as a priority in the plan of
care?
1. Protecting the client from infection
2. Providing emotional support to decrease fear
3. Encouraging discussion about lifestyle changes
4. Identifying factors that decreased the immune function
Answer: 1
Rationale: The client with immune deficiency has inadequate or absent immune bodies
and is at risk for infection. The priority nursing intervention would be to protect the client
from infection. Options 2, 3, and 4 may be components of care but are not the priority.
The nurse is assigned to care for a client with systemic lupus erythematosus (SLE). The
nurse plans care knowing that this disorder is:
1. A local rash that occurs as a result of allergy
2. A disease caused by overexposure to sunlight
3. An inflammatory disease of collagen contained in connective tissue
4. A disease caused by the continuous release of histamine in the body
Answer: 3
Rationale: SLE is an inflammatory disease of collagen contained in connective tissue.
Options 1, 2, and 4 are not associated with this disease.
The nurse is assigned to care for a client admitted to the hospital with a diagnosis of
systemic lupus erythematosus (SLE). The nurse reviews the health care provider's
prescriptions. Which of the following medications would the nurse expect to be
prescribed?
1. Antibiotic
2. Antidiarrheal
3. Corticosteroid
4. Opioid analgesic
Answer: 3
Rationale: Treatment of SLE is based on the systems involved and symptoms.
Treatment normally consists of anti-inflammatory drugs, corticosteroids, and
immunosuppressants. The incorrect options are not standard components of medication
therapy for this disorder.
A nurse is collecting data on a client who complains of fatigue, weakness, malaise,
muscle pain, joint pain at multiple sites, anorexia, and photosensitivity. Systematic lupus
erythematosus (SLE) is suspected. The nurse further checks for which of the following
that is also indicative of the presence of SLE?
1. Emboli
2. Ascites
3. Two hemoglobin S genes
4. Butterfly rash on cheeks and bridge of nose
Answer: 4
Rationale: SLE is a chronic inflammatory disease that affects multiple body systems. A
butterfly rash on the cheeks and on the bridge of the nose is a classic sign of SLE.
Option 3 is found in sickle cell anemia. Options 1 and 2 may be found in many
conditions but are not associated with SLE.
Which client is at the highest risk for systemic lupus erythematous (SLE)?
1. An Asian male
2. A white female
3. An African-American male
4. An African-American female
1. An Asian male
2. A white female
3. An African-American male
4. An African-American female
Answer: 4
Rationale: SLE affects females more commonly than males. It is more common in
African-American females than in white females.
A nurse is reviewing the medical record of a young female client who is suspected of
having systematic lupus erythematosus (SLE). Which of the following would the nurse
expect to note documented in the record that is related to this diagnosis?
1. Presence of two hemoglobin S genes in the blood cell report
2. Ascites noted in the abdomen
3. Recurrent emboli
4. Butterfly rash on cheeks and bridge of the nose
Answer: 4
Rationale: SLE primarily occurs in females 10 to 35 years of age and is a chronic
inflammatory disease that affects multiple body systems. A butterfly rash on the cheeks
and the bridge of the nose is a characteristic sign of SLE. Option 1 is found in sickle cell
anemia. Options 2 and 3 are found in many conditions but are not usually noted in SLE.
The nurse provides home care instructions to a client with systemic lupus
erythematosus and tells the client about methods to manage fatigue. Which statement
by the client indicates a need for further instructions?
1. "I should take hot baths because they are relaxing."
2. "I should sit whenever possible to conserve my energy."
3. "I should avoid long periods of rest because it causes joint stiffness."
4. "I should do some exercises, such as walking, when I am not fatigued."
Answer: 1
Rationale: To help reduce fatigue in the client with systemic lupus erythematosus, the
nurse should instruct the client to sit whenever possible, avoid hot baths (because they
exacerbate fatigue), schedule moderate low-impact exercises when not fatigued, and
maintain a balanced diet. The client is instructed to avoid long periods of rest because it
promotes joint stiffness.
A complete blood cell count is performed on a client with systemic lupus erythematosus
(SLE). The nurse would suspect that which of the following findings will be reported
from this blood test?
1. Increased red blood cell count
2. Decrease of all cell types
3. Increased white blood cell count
4. Increased neutrophils
Answer: 2
Rationale: In the client with SLE, a complete blood count commonly shows
pancytopenia, a decrease of all cell types, probably caused by a direct attack of all
blood cells or bone marrow by immune complexes. The other options are incorrect.
A client is diagnosed with an immune deficiency. The nurse focuses on which of the
following as the highest priority when providing care to this client?
1. Encouraging discussion about emotional impact of the disorder
2. Identifying historical factors that placed the client at risk
3. Providing emotional support to decrease fear
4. Protecting the client from infection
Answer: 4
Rationale: The client with immune deficiency has inadequate immune bodies and is at
risk for infection. The priority nursing intervention would be to protect the client from
infection. The other options are also part of the plan of care but are not the highest
priority.
1. A 26-year-old woman has been diagnosed with early systemic lupus erythematosus
(SLE) involving her joints. In teaching the patient about the disease, the nurse includes
the information that SLE is a(n):
A. Hereditary disorder of women but usually does not show clinical symptoms unless a
woman becomes pregnant.
B. Autoimmune disease of women in which antibodies are formed that destroy all
nucleated cells in the body.
C. Disorder of immune function, but it is extremely variable in its course, and there is no
way to predict its progression.
D. Disease that causes production of antibodies that bind with cellular estrogen
receptors, causing an inflammatory response.
1. Answer: C
Rationale: SLE has an unpredictable course, even with appropriate treatment. Women
are more at risk for SLE, but it is not confined exclusively to women. Clinical symptoms
may worsen during pregnancy but are not confined to pregnancy or the perinatal period.
All nucleated cells are not destroyed by the antinuclear antibodies. The inflammation in
SLE is not caused by antibody binding to cellular estrogen receptors.
2. A patient with an acute exacerbation of systemic lupus erythematosus (SLE) is
hospitalized with incapacitating fatigue, acute hand and wrist pain, and proteinuria. The
health care provider prescribes prednisone (Deltasone) 40 mg twice daily. Which
nursing action should be included in the plan of care?

A. Institute seizure precautions.


B. Reorient to time and place PRN.
C. Monitor intake and output.
D. Place on cardiac monitor.
2. Answer: C
Rationale: Lupus nephritis is a common complication of SLE, and when the patient is
taking corticosteroids, it is especially important to monitor renal function. There is no
indication that the patient is experiencing any nervous system or cardiac problems with
the SLE.
3. A patient with systemic lupus erythematosus (SLE) who has a facial rash and
alopecia tells the nurse, "I hate the way I look! I never go anyplace except here to the
health clinic." An appropriate nursing diagnosis for the patient is:

A. Activity intolerance related to fatigue and inactivity.


B. Impaired skin integrity related to itching and skin sloughing.
C. Social isolation related to embarrassment about the effects of SLE.
D. Impaired social interaction related to lack of social skills.
3. Answer: C
Rationale: The patient's statement about not going anyplace because of hating the way
he or she looks supports the diagnosis of social isolation because of embarrassment
about the effects of the SLE. Activity intolerance is a possible problem for patients with
SLE, but the information about this patient does not support this as a diagnosis. The
rash with SLE is nonpruritic. There is no evidence of lack of social skills for this patient.
4. A patient with polyarthralgia with joint swelling and pain is being evaluated for
systemic lupus erythematosus (SLE). The nurse knows that the serum test result that is
the most specific for SLE is the presence of:

A. Rheumatoid factor.
B. Anti-Smith antibody (Anti-Sm).
C. Antinuclear antibody (ANA).
D. Lupus erythematosus (LE) cell prep.
4. Answer: B
Rationale: The anti-Sm is antibody found almost exclusively in SLE. The other blood
tests are also used in screening but are not as specific to SLE.
5. Following instruction for a patient with newly diagnosed systemic lupus
erythematosus (SLE), the nurse determines that teaching about the disease has been
effective when the patient says:

A. "I should expect to have a low fever all the time with this disease."
B. "I need to restrict my exposure to sunlight to prevent an acute onset of symptoms."
C. "I should try to ignore my symptoms as much as possible and have a positive
outlook."
D. "I can expect a temporary improvement in my symptoms if I become pregnant."
5. Answer: B
Rationale: Sun exposure is associated with SLE exacerbation, and patients should use
sunscreen with an SPF of at least 15 and stay out of the sun between 11:00 AM and
3:00 PM. Low-grade fever may occur with an exacerbation but should not be expected
all the time. A positive attitude may decrease the incidence of SLE exacerbations, but
patients are taught to self-monitor for symptoms that might indicate changes in the
disease process. Symptoms may worsen during pregnancy and especially during the
postpartum period.
6. A 19-year-old patient who is taking azathioprine (Imuran) for systemic lupus
erythematosus has a check-up before leaving home for college. The health care
provider writes all of these orders. Which one should the nurse question?

A. Naproxen (Aleve) 200 mg BID


B. Give measles-mumps-rubella (MMR) immunization
C. Draw anti-DNA titer
D. Famotidine (Pepcid) 20 mg daily
6. Answer: B
Rationale: Live virus vaccines, such as rubella, are contraindicated in a patient taking
immunosuppressive drugs. The other orders are appropriate for the patient.
7. A client is suspected of having systemic lupus erythematous. The nurse monitors the
client, knowing that which of the following is one of the initial characteristic sign of
systemic lupus erythematous?

A. Weight gain
B. Subnormal temperature
C. Elevated red blood cell count
D. Rash on the face across the bridge of the nose
7. Answer: D
Rationale: Skin lesions or rash on the face across the bridge of the nose and on the
cheeks is an initial characteristic sign of systemic lupus erythematosus (SLE). Fever
and weight loss may also occur. Anemia is most likely to occur later in SLE.
8. The nurse is assigned to care for a client with systemic lupus erythematosus (SLE).
The nurse plans care knowing that this disorder is:

A. A local rash that occurs as a result of allergy


B. A disease caused by overexposure to sunlight
C. An inflammatory disease of collagen contained in connective tissue
D. A disease caused by the continuous release of histamine in the body
8. Answer: C
An inflammatory disease of collagen contained in connective tissue
Test-Taking Strategy:
Use the process of elimination. Eliminate option 1 because SLE is a systemic disorder,
not a local one. Next eliminate option 2 because of its similarity to option 1. From the
remaining options, select option 3 because of its systemic characteristic. If you are
unfamiliar with this disorder, review its characteristics.
9. The nurse is assigned to care for a client admitted to the hospital with a diagnosis of
systemic lupus erythematosus (SLE). The nurse reviews the health care provider's
prescriptions. Which of the following medications would the nurse expect to be
prescribed?

A. Antibiotic
B. Antidiarrheal
C. Corticosteroid
D. Opioid analgesic
9. Answer: C
Rationale: Treatment of SLE is based on the systems involved and symptoms.
Treatment normally consists of anti-inflammatory drugs, corticosteroids, and
immunosuppressants. The incorrect options are not standard components of medication
therapy for this disorder.
10. A nurse is collecting data on a client who complains of fatigue, weakness, malaise,
muscle pain, joint pain at multiple sites, anorexia, and photosensitivity. Systematic lupus
erythematosus (SLE) is suspected. The nurse further checks for which of the following
that is also indicative of the presence of SLE?

A. Emboli
B. Ascites
C. Two hemoglobin S genes
D. Butterfly rash on cheeks and bridge of nose
10. Answer: D
Rationale: SLE is a chronic inflammatory disease that affects multiple body systems. A
butterfly rash on the cheeks and on the bridge of the nose is a classic sign of SLE.
Option 3 is found in sickle cell anemia. Options 1 and 2 may be found in many
conditions but are not associated with SLE.
11. Which client is at the highest risk for systemic lupus erythematous (SLE)?

A. An Asian male
B. A white female
C. An African-American male
D. An African-American female
11. Answer: D
An African-American female
12. The nurse monitors a patient to have Systemic Lupus Erythematosus. Which of the
following symptoms is characteristic of this diagnosis?

A. Increased T-cell count


B. Scaley, inflamed rash on shoulders, neck, and face
C. Swelling of the extremities
D. Decreased erythrocyte sedimentation rate (ESR)
12. Answer: B
Scaley, inflamed rash on shoulders, neck, and face
13. In teaching a patient with SLE about the disorder, the nurse knows that the
pathophysiology of SLE includes:

A. Circulating immune complexes formed from IgG autoantibodies reacting with IgG
B. An autoimmune T-cell reaction that results in destruction of the deep dermal skin
layer
C. Immunologic dysfunction leading to chronic inflammation in the cartilage and
muscles
D. The production of a variety of autoantibodies directed against components of the cell
nucleus
13. Answer: D
Rationale: Systemic lupus erythematosus (SLE) is characterized by the production of
many autoantibodies against nucleic acids (e.g., single-and double-stranded DNA),
erythrocytes, coagulation proteins, lymphocytes, platelets, and many other self-proteins.
Autoimmune reactions characteristically are directed against constituents of the cell
nucleus (e.g., antinuclear antibodies [ANAs]), particularly DNA. Circulating immune
complexes containing antibody against DNA are deposited in the basement membranes
of capillaries in the kidneys, heart, skin, brain, and joints. Complement is activated, and
inflammation occurs. The overaggressive antibody response is also related to activation
of B and T cells. The specific manifestations of SLE depend on which cell types or
organs are involved. SLE is a type III hypersensitivity response.
14. A female patient's complex symptomatology over the past year has culminated in a
diagnosis of systemic lupus erythematosus (SLE). Which of the patient's following
statements demonstrates the need for further teaching about the disease?

A. "I'll try my best to stay out of the sun this summer."


B. "I know that I probably have a high chance of getting arthritis."
C. "I'm hoping that surgery will be an option for me in the future."
D. "I understand that I'm going to be vulnerable to getting infections."
14. Answer: C
"I'm hoping that surgery will be an option for me in the future."
Rationale: SLE carries an increased risk of infection, sun damage, and arthritis. Surgery
is not a key treatment modality for SLE.
15. The pathophysiology of systemic lupus erthematosus (SLE) is characterized by:

A. Destruction of nucleic acids and other self-proteins by autoantibodies


B. Overproduction of collagen that disrupts the functioning of internal organs
C. Formation of abnormal IgG that attaches to cellular antigens, activating complement
D. Increased activity of T-suppressor cells with B-cell hypoactivity, resulting in an
immunodeficiency
15. Answer: A
Destruction of nucleic acids and other self-proteins by autoantibodies
16. A patient with newly diagnosed SLE asks the nurse how the disease will affect her
life. The best response by the nurse is:
A. You can plan to have a near-normal life since SLE rarely causes death
B. It is difficult to tell because to disease is so variable in its severity and progression
C. Life span is shortened somewhat in people with SLE, but the disease can be
controlled with long-term use of corticosteroids
D. Most people with SLE have alternating periods of remissions and exacerbations with
rapid progression to permanent organ damage
16. Answer: B
It is difficult to tell because to disease is so variable in its severity and progression
17. During an acute exacerbation, a patient with SLE is treated with corticosteroids. The
nurse would expect the steroids to begin to be tapered when serum laboratory results
indicate:

A. Increased RBCs
B. Decreased ESR
C. Decreased anti-DNA
D. Increased complement
17. Answer: C
Decreased anti-DNA
18. Teaching that the nurse will plan for the patient with SLE includes:

A. Ways to avoid exposure to sunlight


B. Increasing dietary protein and carbohydrate intake
C. The necessity of genetic counseling before planning a family
D. The use of no pharmacologic pain interventions instead of analgesics
18. Answer: A
Ways to avoid exposure to sunlight
25. A 26-year-old woman has been diagnosed with early systemic lupus erythematosus
(SLE) involving her joints. In teaching the patient about the disease, the nurse includes
the information that SLE is a(n)
a. hereditary disorder of women but usually does not show clinical symptoms unless a
woman becomes pregnant.
b. autoimmune disease of women in which antibodies are formed that destroy all
nucleated cells in the body.
c. disorder of immune function, but it is extremely variable in its course, and there is no
way to predict its progression.
d. disease that causes production of antibodies that bind with cellular estrogen
receptors, causing an inflammatory response.
Answer: C
Rationale: SLE has an unpredictable course, even with appropriate treatment. Women
are more at risk for SLE, but it is not confined exclusively to women. Clinical symptoms
may worsen during pregnancy but are not confined to pregnancy or the perinatal period.
All nucleated cells are not destroyed by the antinuclear antibodies. The inflammation in
SLE is not caused by antibody binding to cellular estrogen receptors.
Cognitive Level: Comprehension Text Reference: pp. 1717, 1719
Nursing Process: Implementation NCLEX: Physiological Integrity
26. A patient with an acute exacerbation of systemic lupus erythematosus (SLE) is
hospitalized with incapacitating fatigue, acute hand and wrist pain, and proteinuria. The
health care provider prescribes prednisone (Deltasone) 40 mg twice daily. Which
nursing action should be included in the plan of care?
a. Institute seizure precautions.
b. Reorient to time and place PRN.
c. Monitor intake and output.
d. Place on cardiac monitor.
Answer: C
Rationale: Lupus nephritis is a common complication of SLE, and when the patient is
taking corticosteroids, it is especially important to monitor renal function. There is no
indication that the patient is experiencing any nervous system or cardiac problems with
the SLE.

Cognitive Level: Application Text Reference: pp. 1718, 1720


Nursing Process: Planning NCLEX: Physiological Integrity
28. A patient with polyarthralgia with joint swelling and pain is being evaluated for
systemic lupus erythematosus (SLE). The nurse knows that the serum test result that is
the most specific for SLE is the presence of
a. rheumatoid factor.
b. anti-Smith antibody (Anti-Sm).
c. antinuclear antibody (ANA).
d. lupus erythematosus (LE) cell prep.
Answer: B
Rationale: The anti-Sm is antibody found almost exclusively in SLE. The other blood
tests are also used in screening but are not as specific to SLE.

Cognitive Level: Comprehension Text Reference: pp. 1718-1719


Nursing Process: Assessment NCLEX: Physiological Integrity
29. Following instruction for a patient with newly diagnosed systemic lupus
erythematosus (SLE), the nurse determines that teaching about the disease has been
effective when the patient says,
a. "I should expect to have a low fever all the time with this disease."
b. "I need to restrict my exposure to sunlight to prevent an acute onset of symptoms."
c. "I should try to ignore my symptoms as much as possible and have a positive
outlook."
d. "I can expect a temporary improvement in my symptoms if I become pregnant."
Answer: B
Rationale: Sun exposure is associated with SLE exacerbation, and patients should use
sunscreen with an SPF of at least 15 and stay out of the sun between 11:00 AM and
3:00 PM. Low-grade fever may occur with an exacerbation but should not be expected
all the time. A positive attitude may decrease the incidence of SLE exacerbations, but
patients are taught to self-monitor for symptoms that might indicate changes in the
disease process. Symptoms may worsen during pregnancy and especially during the
postpartum period.

Cognitive Level: Application Text Reference: pp. 1717, 1720


Nursing Process: Evaluation NCLEX: Physiological Integrity
30. A 19-year-old patient who is taking azathioprine (Imuran) for systemic lupus
erythematosus has a check-up before leaving home for college. The health care
provider writes all of these orders. Which one should the nurse question?
a. Naproxen (Aleve) 200 mg BID
b. Give measles-mumps-rubella (MMR) immunization
c. Draw anti-DNA titer
d. Famotidine (Pepcid) 20 mg daily
Answer: B
Rationale: Live virus vaccines, such as rubella, are contraindicated in a patient taking
immunosuppressive drugs. The other orders are appropriate for the patient.

Cognitive Level: Application Text Reference: p. 1718


Nursing Process: Implementation NCLEX: Psychosocial Integrity
41. When the nurse is reviewing laboratory results for a patient with systemic lupus
erythematosus (SLE), which result is most important to communicate to the health care
provider?
a. Elevated blood urea nitrogen (BUN) and creatinine
b. Positive lupus erythematosus cell prep
c. Positive antinuclear antibodies (ANA)
d. Decreased C-reactive protein (CRP)
Answer: A
Rationale: The elevated BUN and creatinine levels indicate possible lupus nephritis and
a need for a change in therapy to avoid further renal damage. The positive LE cell prep
and ANA would be expected in a patient with SLE. A drop in CRP shows an
improvement in the inflammatory process.

Cognitive Level: Application Text Reference: p. 1718


Nursing Process: Assessment NCLEX: Physiological Integrity
Which statement by a patient with systemic lupus erythematosus (SLE) indicates that
the patient has understood the nurse's teaching about the condition?
a. "I will exercise even if I am tired."
b. "I will use sunscreen when I am outside."
c. "I should take birth control pills to keep from getting pregnant."
d. "I should avoid aspirin or nonsteroidal antiinflammatory drugs."
ANS: B
Severe skin reactions can occur in patients with SLE who are exposed to the sun.
Patients should avoid fatigue by balancing exercise with rest periods as needed. Oral
contraceptives can exacerbate lupus. Aspirin and nonsteroidal antiinflammatory drugs
are used to treat the musculoskeletal manifestations of SLE.
A 25-year-old female patient with systemic lupus erythematosus (SLE) who has a facial
rash and alopecia tells the nurse, "I never leave my house because I hate the way I
look." An appropriate nursing diagnosis for the patient is
a. activity intolerance related to fatigue and inactivity.
b. impaired social interaction related to lack of social skills.
c. impaired skin integrity related to itching and skin sloughing.
d. social isolation related to embarrassment about the effects of SLE.
ANS: D
The patient's statement about not going anywhere because of hating the way he or she
looks supports the diagnosis of social isolation because of embarrassment about the
effects of the SLE. Activity intolerance is a possible problem for patients with SLE, but
the information about this patient does not support this as a diagnosis. The rash with
SLE is nonpruritic. There is no evidence of lack of social skills for this patient.
A new clinic patient with joint swelling and pain is being tested for systemic lupus
erythematosus. Which test will provide the most specific findings for the nurse to
review?
a. Rheumatoid factor (RF)
b. Antinuclear antibody (ANA)
c. Anti-Smith antibody (Anti-Sm)
d. Lupus erythematosus (LE) cell prep
ANS: C
The anti-Sm is antibody found almost exclusively in SLE. The other blood tests are also
used in screening but are not as specific to SLE
The health care provider has prescribed the following collaborative interventions for a
patient who is taking azathioprine (Imuran) for systemic lupus erythematosus. Which
order will the nurse question?
a. Draw anti-DNA blood titer.
b. Administer varicella vaccine.
c. Naproxen (Aleve) 200 mg BID.
d. Famotidine (Pepcid) 20 mg daily.
ANS: B
Live virus vaccines, such as varicella, are contraindicated in a patient taking
immunosuppressive drugs. The other orders are appropriate for the patient
Which result for a 30-year-old patient with systemic lupus erythematosus (SLE) is most
important for the nurse to communicate to the health care provider?
a. Decreased C-reactive protein (CRP)
b. Elevated blood urea nitrogen (BUN)
c. Positive antinuclear antibodies (ANA)
d. Positive lupus erythematosus cell prep
ANS: B
The elevated BUN and creatinine levels indicate possible lupus nephritis and a need for
a change in therapy to avoid further renal damage. The positive lupus erythematosus
(LE) cell prep and ANA would be expected in a patient with SLE. A drop in CRP shows
an improvement in the inflammatory process.
In teaching a patient with SLE about the disorder, the nurse knows that the
pathophysiology of SLE includes
a. circulating immune complexes formed from IgG autoantibodies reacting with IgG
b. an autoimmune T-cell reaction that results in destruction of the deep dermal skin
layer
c. immunologic dysfunction leading to chronic inflammation in the cartilage and muscles
d. the production of a variety of autoantibodies directed against components of the cell
nucleus
Correct answer: d
Rationale: Systemic lupus erythematosus (SLE) is characterized by the production of
many autoantibodies against nucleic acids (e.g., single-and double-stranded DNA),
erythrocytes, coagulation proteins, lymphocytes, platelets, and many other self-proteins.
Autoimmune reactions characteristically are directed against constituents of the cell
nucleus (e.g., antinuclear antibodies [ANAs]), particularly DNA. Circulating immune
complexes containing antibody against DNA are deposited in the basement membranes
of capillaries in the kidneys, heart, skin, brain, and joints. Complement is activated, and
inflammation occurs. The overaggressive antibody response is also related to activation
of B and T cells. The specific manifestations of SLE depend on which cell types or
organs are involved. SLE is a type III hypersensitivity response.
A female patient's complex symptomatology over the past year has culminated in a
diagnosis of systemic lupus erythematosus (SLE). Which of the patient's following
statements demonstrates the need for further teaching about the disease?

A. "I'll try my best to stay out of the sun this summer."


B. "I know that I probably have a high chance of getting arthritis."
C. "I'm hoping that surgery will be an option for me in the future."
D. "I understand that I'm going to be vulnerable to getting infections."
Correct answer: C. "I'm hoping that surgery will be an option for me in the future."
Rationale: SLE carries an increased risk of infection, sun damage, and arthritis. Surgery
is not a key treatment modality for SLE.
A patient with OA uses NSAIDs to decrease pain and inflammation. The nurse teaches
the patient that common side effects of these drugs include
a. allergic reactions, fever, and oral lesions
b. fluid retention, hypertension, and bruising
c. skin rashes, gastric irritation, and headache
d. prolonged bleeding time, blood dyscrasias, and hepatic damage
C. skin rashes, gastric irritation, and headacche
The pathophysiology of systemic lupus erthematosus (SLE) is characterized by
a. destruction of nucleic acids and other self-proteins by autoantibodies
b. overproduction of collagen that disrupts the functioning of internal organs
c. formation of abnormal IgG that attaches to cellular antigens, activating complement
d. increased activity of T-suppressor cells with B-cell hypoactivity, resulting in an
immunodeficiency
A. destruction of nucleic acids and other self-proteins by autoantibodies
A patient with newly diagnosed SLE asks the nurse how the disease will affect her life.
The best response by the nurse is
a. You can plan to have a near-normal life since SLE rarely causes death
b. it is difficult to tell because to disease is so variable in its severity and progression
c. life span is shortened somewhat in people with SLE, but the disease can be
controlled with long-term use of corticosteroids
d. most people with SLE have alternating periods of remissions and exacerbations with
rapid progression to permanent organ damage
B. it is difficult to tell because the disease is so variable in its severity and progression
During an acute exacerbation, a patient with SLE is treated with corticosteroids. The
nurse would expect the steroids to begin to be tapered when serum laboratory results
indicate
a. increased RBCs
b. decreased ESR
c. decreased anti-DNA
d. increased complement
C. decreased anti-DNA
Teaching that the nurse will plan for the patient with SLE includes
a. ways to avoid exposure to sunlight
b. increasing dietary protein and carbohydrate intake
c. the necessity of genetic counseling before planning a family
d. the use of no pharmacologic pain interventions instead of analgesics
A. ways to avoid exposure to sunlight

25. A 26-year-old woman has been diagnosed with early systemic lupus erythematosus
(SLE) involving her joints. In teaching the patient about the disease, the nurse includes
the information that SLE is a(n)
a. hereditary disorder of women but usually does not show clinical symptoms unless a
woman becomes pregnant.
b. autoimmune disease of women in which antibodies are formed that destroy all
nucleated cells in the body.
c. disorder of immune function, but it is extremely variable in its course, and there is no
way to predict its progression.
d. disease that causes production of antibodies that bind with cellular estrogen
receptors, causing an inflammatory response.
Answer: C
Rationale: SLE has an unpredictable course, even with appropriate treatment. Women
are more at risk for SLE, but it is not confined exclusively to women. Clinical symptoms
may worsen during pregnancy but are not confined to pregnancy or the perinatal period.
All nucleated cells are not destroyed by the antinuclear antibodies. The inflammation in
SLE is not caused by antibody binding to cellular estrogen receptors.

Cognitive Level: Comprehension Text Reference: pp. 1717, 1719


Nursing Process: Implementation NCLEX: Physiological Integrity
26. A patient with an acute exacerbation of systemic lupus erythematosus (SLE) is
hospitalized with incapacitating fatigue, acute hand and wrist pain, and proteinuria. The
health care provider prescribes prednisone (Deltasone) 40 mg twice daily. Which
nursing action should be included in the plan of care?
a. Institute seizure precautions.
b. Reorient to time and place PRN.
c. Monitor intake and output.
d. Place on cardiac monitor.
Answer: C
Rationale: Lupus nephritis is a common complication of SLE, and when the patient is
taking corticosteroids, it is especially important to monitor renal function. There is no
indication that the patient is experiencing any nervous system or cardiac problems with
the SLE.

Cognitive Level: Application Text Reference: pp. 1718, 1720


Nursing Process: Planning NCLEX: Physiological Integrity
28. A patient with polyarthralgia with joint swelling and pain is being evaluated for
systemic lupus erythematosus (SLE). The nurse knows that the serum test result that is
the most specific for SLE is the presence of
a. rheumatoid factor.
b. anti-Smith antibody (Anti-Sm).
c. antinuclear antibody (ANA).
d. lupus erythematosus (LE) cell prep.
Answer: B
Rationale: The anti-Sm is antibody found almost exclusively in SLE. The other blood
tests are also used in screening but are not as specific to SLE.

Cognitive Level: Comprehension Text Reference: pp. 1718-1719


Nursing Process: Assessment NCLEX: Physiological Integrity
29. Following instruction for a patient with newly diagnosed systemic lupus
erythematosus (SLE), the nurse determines that teaching about the disease has been
effective when the patient says,
a. "I should expect to have a low fever all the time with this disease."
b. "I need to restrict my exposure to sunlight to prevent an acute onset of symptoms."
c. "I should try to ignore my symptoms as much as possible and have a positive
outlook."
d. "I can expect a temporary improvement in my symptoms if I become pregnant."
Answer: B
Rationale: Sun exposure is associated with SLE exacerbation, and patients should use
sunscreen with an SPF of at least 15 and stay out of the sun between 11:00 AM and
3:00 PM. Low-grade fever may occur with an exacerbation but should not be expected
all the time. A positive attitude may decrease the incidence of SLE exacerbations, but
patients are taught to self-monitor for symptoms that might indicate changes in the
disease process. Symptoms may worsen during pregnancy and especially during the
postpartum period.

Cognitive Level: Application Text Reference: pp. 1717, 1720


Nursing Process: Evaluation NCLEX: Physiological Integrity
30. A 19-year-old patient who is taking azathioprine (Imuran) for systemic lupus
erythematosus has a check-up before leaving home for college. The health care
provider writes all of these orders. Which one should the nurse question?
a. Naproxen (Aleve) 200 mg BID
b. Give measles-mumps-rubella (MMR) immunization
c. Draw anti-DNA titer
d. Famotidine (Pepcid) 20 mg daily
Answer: B
Rationale: Live virus vaccines, such as rubella, are contraindicated in a patient taking
immunosuppressive drugs. The other orders are appropriate for the patient.

Cognitive Level: Application Text Reference: p. 1718


Nursing Process: Implementation NCLEX: Psychosocial Integrity
41. When the nurse is reviewing laboratory results for a patient with systemic lupus
erythematosus (SLE), which result is most important to communicate to the health care
provider?
a. Elevated blood urea nitrogen (BUN) and creatinine
b. Positive lupus erythematosus cell prep
c. Positive antinuclear antibodies (ANA)
d. Decreased C-reactive protein (CRP)
Answer: A
Rationale: The elevated BUN and creatinine levels indicate possible lupus nephritis and
a need for a change in therapy to avoid further renal damage. The positive LE cell prep
and ANA would be expected in a patient with SLE. A drop in CRP shows an
improvement in the inflammatory process.

Cognitive Level: Application Text Reference: p. 1718


Nursing Process: Assessment NCLEX: Physiological Integrity

1. A 26-year-old woman has been diagnosed with early systemic lupus erythematosus
(SLE) involving her joints. In teaching the patient about the disease, the nurse includes
the information that SLE is a(n):
A. Hereditary disorder of women but usually does not show clinical symptoms unless a
woman becomes pregnant.
B. Autoimmune disease of women in which antibodies are formed that destroy all
nucleated cells in the body.
C. Disorder of immune function, but it is extremely variable in its course, and there is no
way to predict its progression.
D. Disease that causes production of antibodies that bind with cellular estrogen
receptors, causing an inflammatory response.
1. Answer: C
Rationale: SLE has an unpredictable course, even with appropriate treatment. Women
are more at risk for SLE, but it is not confined exclusively to women. Clinical symptoms
may worsen during pregnancy but are not confined to pregnancy or the perinatal period.
All nucleated cells are not destroyed by the antinuclear antibodies. The inflammation in
SLE is not caused by antibody binding to cellular estrogen receptors.
2. A patient with an acute exacerbation of systemic lupus erythematosus (SLE) is
hospitalized with incapacitating fatigue, acute hand and wrist pain, and proteinuria. The
health care provider prescribes prednisone (Deltasone) 40 mg twice daily. Which
nursing action should be included in the plan of care?
A. Institute seizure precautions.
B. Reorient to time and place PRN.
C. Monitor intake and output.
D. Place on cardiac monitor.
2. Answer: C
Rationale: Lupus nephritis is a common complication of SLE, and when the patient is
taking corticosteroids, it is especially important to monitor renal function. There is no
indication that the patient is experiencing any nervous system or cardiac problems with
the SLE.
3. A patient with systemic lupus erythematosus (SLE) who has a facial rash and
alopecia tells the nurse, "I hate the way I look! I never go anyplace except here to the
health clinic." An appropriate nursing diagnosis for the patient is:

A. Activity intolerance related to fatigue and inactivity.


B. Impaired skin integrity related to itching and skin sloughing.
C. Social isolation related to embarrassment about the effects of SLE.
D. Impaired social interaction related to lack of social skills.
3. Answer: C
Rationale: The patient's statement about not going anyplace because of hating the way
he or she looks supports the diagnosis of social isolation because of embarrassment
about the effects of the SLE. Activity intolerance is a possible problem for patients with
SLE, but the information about this patient does not support this as a diagnosis. The
rash with SLE is nonpruritic. There is no evidence of lack of social skills for this patient.
4. A patient with polyarthralgia with joint swelling and pain is being evaluated for
systemic lupus erythematosus (SLE). The nurse knows that the serum test result that is
the most specific for SLE is the presence of:

A. Rheumatoid factor.
B. Anti-Smith antibody (Anti-Sm).
C. Antinuclear antibody (ANA).
D. Lupus erythematosus (LE) cell prep.
4. Answer: B
Rationale: The anti-Sm is antibody found almost exclusively in SLE. The other blood
tests are also used in screening but are not as specific to SLE.
5. Following instruction for a patient with newly diagnosed systemic lupus
erythematosus (SLE), the nurse determines that teaching about the disease has been
effective when the patient says:

A. "I should expect to have a low fever all the time with this disease."
B. "I need to restrict my exposure to sunlight to prevent an acute onset of symptoms."
C. "I should try to ignore my symptoms as much as possible and have a positive
outlook."
D. "I can expect a temporary improvement in my symptoms if I become pregnant."
5. Answer: B
Rationale: Sun exposure is associated with SLE exacerbation, and patients should use
sunscreen with an SPF of at least 15 and stay out of the sun between 11:00 AM and
3:00 PM. Low-grade fever may occur with an exacerbation but should not be expected
all the time. A positive attitude may decrease the incidence of SLE exacerbations, but
patients are taught to self-monitor for symptoms that might indicate changes in the
disease process. Symptoms may worsen during pregnancy and especially during the
postpartum period.
6. A 19-year-old patient who is taking azathioprine (Imuran) for systemic lupus
erythematosus has a check-up before leaving home for college. The health care
provider writes all of these orders. Which one should the nurse question?

A. Naproxen (Aleve) 200 mg BID


B. Give measles-mumps-rubella (MMR) immunization
C. Draw anti-DNA titer
D. Famotidine (Pepcid) 20 mg daily
6. Answer: B
Rationale: Live virus vaccines, such as rubella, are contraindicated in a patient taking
immunosuppressive drugs. The other orders are appropriate for the patient.
7. A client is suspected of having systemic lupus erythematous. The nurse monitors the
client, knowing that which of the following is one of the initial characteristic sign of
systemic lupus erythematous?

A. Weight gain
B. Subnormal temperature
C. Elevated red blood cell count
D. Rash on the face across the bridge of the nose
7. Answer: D
Rationale: Skin lesions or rash on the face across the bridge of the nose and on the
cheeks is an initial characteristic sign of systemic lupus erythematosus (SLE). Fever
and weight loss may also occur. Anemia is most likely to occur later in SLE.
8. The nurse is assigned to care for a client with systemic lupus erythematosus (SLE).
The nurse plans care knowing that this disorder is:

A. A local rash that occurs as a result of allergy


B. A disease caused by overexposure to sunlight
C. An inflammatory disease of collagen contained in connective tissue
D. A disease caused by the continuous release of histamine in the body
8. Answer: C
An inflammatory disease of collagen contained in connective tissue
Test-Taking Strategy:
Use the process of elimination. Eliminate option 1 because SLE is a systemic disorder,
not a local one. Next eliminate option 2 because of its similarity to option 1. From the
remaining options, select option 3 because of its systemic characteristic. If you are
unfamiliar with this disorder, review its characteristics.
9. The nurse is assigned to care for a client admitted to the hospital with a diagnosis of
systemic lupus erythematosus (SLE). The nurse reviews the health care provider's
prescriptions. Which of the following medications would the nurse expect to be
prescribed?
A. Antibiotic
B. Antidiarrheal
C. Corticosteroid
D. Opioid analgesic
9. Answer: C
Rationale: Treatment of SLE is based on the systems involved and symptoms.
Treatment normally consists of anti-inflammatory drugs, corticosteroids, and
immunosuppressants. The incorrect options are not standard components of medication
therapy for this disorder.
10. A nurse is collecting data on a client who complains of fatigue, weakness, malaise,
muscle pain, joint pain at multiple sites, anorexia, and photosensitivity. Systematic lupus
erythematosus (SLE) is suspected. The nurse further checks for which of the following
that is also indicative of the presence of SLE?

A. Emboli
B. Ascites
C. Two hemoglobin S genes
D. Butterfly rash on cheeks and bridge of nose
10. Answer: D
Rationale: SLE is a chronic inflammatory disease that affects multiple body systems. A
butterfly rash on the cheeks and on the bridge of the nose is a classic sign of SLE.
Option 3 is found in sickle cell anemia. Options 1 and 2 may be found in many
conditions but are not associated with SLE.
11. Which client is at the highest risk for systemic lupus erythematous (SLE)?

A. An Asian male
B. A white female
C. An African-American male
D. An African-American female
11. Answer: D
An African-American female
12. The nurse monitors a patient to have Systemic Lupus Erythematosus. Which of the
following symptoms is characteristic of this diagnosis?

A. Increased T-cell count


B. Scaley, inflamed rash on shoulders, neck, and face
C. Swelling of the extremities
D. Decreased erythrocyte sedimentation rate (ESR)
12. Answer: B
Scaley, inflamed rash on shoulders, neck, and face
13. In teaching a patient with SLE about the disorder, the nurse knows that the
pathophysiology of SLE includes:

A. Circulating immune complexes formed from IgG autoantibodies reacting with IgG
B. An autoimmune T-cell reaction that results in destruction of the deep dermal skin
layer
C. Immunologic dysfunction leading to chronic inflammation in the cartilage and
muscles
D. The production of a variety of autoantibodies directed against components of the cell
nucleus
13. Answer: D
Rationale: Systemic lupus erythematosus (SLE) is characterized by the production of
many autoantibodies against nucleic acids (e.g., single-and double-stranded DNA),
erythrocytes, coagulation proteins, lymphocytes, platelets, and many other self-proteins.
Autoimmune reactions characteristically are directed against constituents of the cell
nucleus (e.g., antinuclear antibodies [ANAs]), particularly DNA. Circulating immune
complexes containing antibody against DNA are deposited in the basement membranes
of capillaries in the kidneys, heart, skin, brain, and joints. Complement is activated, and
inflammation occurs. The overaggressive antibody response is also related to activation
of B and T cells. The specific manifestations of SLE depend on which cell types or
organs are involved. SLE is a type III hypersensitivity response.
14. A female patient's complex symptomatology over the past year has culminated in a
diagnosis of systemic lupus erythematosus (SLE). Which of the patient's following
statements demonstrates the need for further teaching about the disease?

A. "I'll try my best to stay out of the sun this summer."


B. "I know that I probably have a high chance of getting arthritis."
C. "I'm hoping that surgery will be an option for me in the future."
D. "I understand that I'm going to be vulnerable to getting infections."
14. Answer: C
"I'm hoping that surgery will be an option for me in the future."
Rationale: SLE carries an increased risk of infection, sun damage, and arthritis. Surgery
is not a key treatment modality for SLE.
15. The pathophysiology of systemic lupus erthematosus (SLE) is characterized by:

A. Destruction of nucleic acids and other self-proteins by autoantibodies


B. Overproduction of collagen that disrupts the functioning of internal organs
C. Formation of abnormal IgG that attaches to cellular antigens, activating complement
D. Increased activity of T-suppressor cells with B-cell hypoactivity, resulting in an
immunodeficiency
15. Answer: A
Destruction of nucleic acids and other self-proteins by autoantibodies
16. A patient with newly diagnosed SLE asks the nurse how the disease will affect her
life. The best response by the nurse is:

A. You can plan to have a near-normal life since SLE rarely causes death
B. It is difficult to tell because to disease is so variable in its severity and progression
C. Life span is shortened somewhat in people with SLE, but the disease can be
controlled with long-term use of corticosteroids
D. Most people with SLE have alternating periods of remissions and exacerbations with
rapid progression to permanent organ damage
16. Answer: B
It is difficult to tell because to disease is so variable in its severity and progression
17. During an acute exacerbation, a patient with SLE is treated with corticosteroids. The
nurse would expect the steroids to begin to be tapered when serum laboratory results
indicate:

A. Increased RBCs
B. Decreased ESR
C. Decreased anti-DNA
D. Increased complement
17. Answer: C
Decreased anti-DNA
18. Teaching that the nurse will plan for the patient with SLE includes:

A. Ways to avoid exposure to sunlight


B. Increasing dietary protein and carbohydrate intake
C. The necessity of genetic counseling before planning a family
D. The use of no pharmacologic pain interventions instead of analgesics
18. Answer: A
Ways to avoid exposure to sunlight

Systemic Lupus Erythematosus


SLE is a chronic inflammatory immune disorder affecting the skin and other body
organs. Antibodies to DNA and RNA cause an autoimmune inflammatory response,
resulting in swelling and pain. It is most common in young women, and has a strong
genetic factor. The etiology is not known
SLE characterized by periods of
remission and exacerbation
autoantibody production results from
hyperreactivity of B cells(humoral immune response)
SLE autoantibodies react with their corresponding antigen to form immune complexes,
which are then deposited in the:
connective tissues of blood vessels, lymphatic vessels, and other tissues
frequent site of complex deposition and damage occur in the
kidneys
Systemic Lupus
Systemic lupus is the most common form of lupus, and is what most people mean when
they refer to "lupus." Systemic lupus can be mild or severe. Some of the more serious
complications involving major organ systems are:

inflammation of the kidneys (lupus nephritis), which can affect the body's ability to filter
waste from the blood and can be so damaging that dialysis or kidney transplant may be
needed
an increase in blood pressure in the lungs (pulmonary hypertension)
inflammation of the nervous system and brain, which can cause memory problems,
confusion, headaches, and strokes
SLE Triggers
While a person's genes may increase the chance that he or she will develop lupus, it
takes some kind of environmental trigger to set off the illness or to bring on a flare.
Examples include:
• ultraviolet rays from the sun
• ultraviolet rays from fluorescent light bulbs
• sulfa drugs, which make a person more sensitive to the sun, such as: Bactrim® and
Septra® (trimethoprim-sulfamethoxazole); sulfisoxazole (Gantrisin®); tolbutamide
(Orinase®); sulfasalazine (Azulfidine®); diuretics
• sun-sensitizing tetracycline drugs such as minocycline (Minocin®)
• penicillin or other antibiotic drugs such as: amoxicillin (Amoxil®); ampicillin (Ampicillin
Sodium ADD-Vantage®); cloxacillin (Cloxapen®)
• an infection
• a cold or a viral illness
• exhaustion
• an injury
• emotional stress, such as a divorce, illness, death in the family, or other life
complications
• anything that causes stress to the body, such as surgery, physical harm, pregnancy, or
giving birth
Although many seemingly unrelated factors can trigger the onset of lupus in a
susceptible person, scientists have noted some common features among many people
who have lupus, including:
• exposure to the sun
• an infection
• being pregnant
• giving birth
• a drug taken to treat an illness
Clients with active disease have an increased risk for
infections
Early manifestations include
Because lupus can affect so many different organs, a wide range of symptoms can
occur. These symptoms may come and go, and different symptoms may appear at
different times during the course of the disease.
The most common symptoms of lupus, which are the same for females and males, are:
• extreme fatigue (tiredness)
• headaches
• painful or swollen joints
• fever
• anemia (low numbers of red blood cells or hemoglobin, or low total blood volume)
• swelling (edema) in feet, legs, hands, and/or around eyes
• pain in chest on deep breathing (pleurisy)
• butterfly-shaped rash across cheeks and nose
• sun- or light-sensitivity (photosensitivity)
• hair loss
• abnormal blood clotting
• fingers turning white and/or blue when cold (Raynaud's phenomenon)
• mouth or nose ulcers
Approximately 50% of people with SLE experience renal manifestations
Proteinuria, cellular casts, nephritic syndromes
Up to 10% renal failure
Hematologic abnormalities
Anemia,
leukopenia,
thrombocytopenia
Cardiovascular
Pericarditis,
vascultitis
Many clients with SLE develop transient nervous system involvement
Decline in intellect, memory loss, disorientation
Psychosis, seizures, depression, stroke
Ocular
Conjunctivitis,
photophobia,
transient blindness due to retinal vasculitis
GI
o Anorexia
o Nausea
o Abdominal pain
o Diarrhea
o hepatomegaly
Lung
o Pleurisy
o Pleural effusion
o pneumonitis
o interstitial fibrosis
Brain
o Neuropathies
o Seizures
o Depression
o Psychosis
o Stroke
Skin
o Butterfly rash on face
o Photosensitivity
o Maculopapular rash on expose body surfaces
o Discoid lesions
o Erythematous fingertip lesions
o Splinter hemorrhages
o Alopecia
o Ulcers (lips, mouth, nose
Sensory System
o Conjunctivitis
o Photophobia
o Retinal vasculitis with transient blindness
o Cotton-wool spots on retina
Diagnostic test
• Anti-DNA antibody testing
• Erythrocyte sedimentation rate::Typically elevated
• Serum complement levels::Usually decreased
• CBC::Anemia, leukopenia, lymphocytopenia, thrmocytopenia
• Urinalysis::Renal function: BUN, Serum Creatinine
• Kidney biopsy
Medications
hydroxychloroguine
Aspirin
Prednisone
Immunosuppressive agents such as cyclophosphamide or azathiprine
Low levels of WBC's and platelets?
Prednisone
Risk for infection R/T compromised immune system
• Prophylactic antibiotics
• Yearly influence vaccine
• Hand hygine
The white blood cell (WBC) count of a client with systemic lupus erythematosus (SLE)
shows a shift to the left. What is the highest priority nursing diagnosis for the nurse
caring for this client?

Impaired Skin Integrity


Ineffective Individual Coping
Ineffective Health Maintenance
Ineffective Protection
Ineffective Protection
Rationale:
All of these diagnoses are appropriate for the client with SLE. However, the results of
this client's WBC indicate that the client is at risk for infection due to the disease
process or its treatments. Ineffective protection, then, is the highest priority of care for
the nurse at this time.
The nurse should instruct a client that which of the following factors might increase the
risk of developing an exacerbation of systemic lupus erythematosus (SLE)?

GI upset
Fever
Hypotension
Pregnancy
Pregnancy
Rationale:
Pregnancy can be associated with exacerbation because of increased estrogen levels.
Hypotension, fever, and GI upset do not cause exacerbation of SLE.
The nurse would assess which of the following in a client who is being evaluated for
systemic lupus erythematosus (SLE)?

Fever
Rash on the face across the nose
Elevated red blood cell count
Fatigue
Rash on the face across the nose
Rationale:
Rash on the face across the nose is the classic sign of SLE. Fever and fatigue are
symptoms that occur during exacerbations. The client with SLE is apt to be anemic.
The nurse, instructing a client with systemic lupus erythematosus (SLE) about the
management of fatigue during an exacerbation, would determine that learning outcomes
need reinforcement if the client states a plan to:

Sit when possible.


Take short rest periods.
Engage in low-impact exercise when not fatigued.
Take a hot bath before bed.
Take a hot bath before bed.
Rationale:
The nurse should instruct the client to avoid hot baths to help reduce fatigue; heat can
cause an exacerbation. The client should be instructed that long periods of rest can
promote joint stiffness. The nurse should encourage the client to sit when possible, and
to engage in low-impact exercise when not fatigued.
The nurse, providing nutritional instruction for a client diagnosed with systemic lupus
erythematosus (SLE), should teach the client to avoid which of the following foods?
(Select all that apply.)

Turkey
Steak
Broccoli
Bacon
Cantaloupe
Bacon

Steak

Rationale:
The client with SLE is at risk for cardiovascular disorders. The client should be
instructed on nutritional lifestyle changes to prevent these disorders. The client should
be instructed to reduce fat (steak, bacon), salt (bacon), and to quit smoking. The other
food choices are part of a healthy diet.
The nurse, caring for a teen diagnosed with systemic lupus erythematosus (SLE),
should instruct the family about which of the following challenges?

Planning meals
Planning family vacations
Violence from the client
Adherence to treatment
Adherence to treatment
Rationale:
Consistent with this developmental stage, the teen client is apt to rebel more strongly
and deny the disease. Adherence to a plan of treatment, medications, and the fact that
this will differentiate them from the peer group can cause nonadherence. Anger at
having the disease could turn to violence, but this is not typical. Planning meals and
vacations should not pose problems for the family.
A client with systemic lupus erythematosus (SLE) asks the nurse if getting a flu vaccine
in the fall is important. The nurse responds with which of the following?

"No, you could get the flu from the shot."


"Yes, you should get the nasal flu vaccine."
"Yes, flu vaccine is a priority every year."
"No, you are not at risk for the flu."
"Yes, flu vaccine is a priority every year."
Rationale:
The client with SLE has a compromised immune system and should be protected from
the flu with a shot every year. The flu shot is not a live virus. The nasal vaccine is a live
virus and should not be used by a client with a compromised immune system.
Immunization with a killed virus will not cause the disease.
A client with systemic lupus erythematosus (SLE) asks the nurse what medications are
used to cure autoimmune diseases. The nurse should respond with which of the
following?

"The NSAIDS provide a cure for autoimmune diseases."


"Antibiotics are used to treat autoimmune diseases."
"Autoimmune diseases are not curable."
"Autoimmune diseases are temporary and do not need medications."
"Autoimmune diseases are not curable."
Rationale:
The nurse should respond that autoimmune diseases are not curable by medications or
by any other means. The client should be instructed on health promotion and
management of symptoms with pharmacological and nonpharmacological methods.
Antibiotics are used when the client with an autoimmune disease acquires an infection.
A 26-year-old woman has been diagnosed with early systemic lupus erythematosus
(SLE) involving her joints. In teaching the patient about the disease, the nurse includes
the information that SLE is a(n)
a. hereditary disorder of women but usually does not show clinical symptoms unless a
woman becomes pregnant.
b. autoimmune disease of women in which antibodies are formed that destroy all
nucleated cells in the body.
c. disorder of immune function, but it is extremely variable in its course, and there is no
way to predict its progression.
d. disease that causes production of antibodies that bind with cellular estrogen
receptors, causing an inflammatory response.
Answer: C
Rationale: SLE has an unpredictable course, even with appropriate treatment. Women
are more at risk for SLE, but it is not confined exclusively to women. Clinical symptoms
may worsen during pregnancy but are not confined to pregnancy or the perinatal period.
All nucleated cells are not destroyed by the antinuclear antibodies. The inflammation in
SLE is not caused by antibody binding to cellular estrogen receptors.
A patient with an acute exacerbation of systemic lupus erythematosus (SLE) is
hospitalized with incapacitating fatigue, acute hand and wrist pain, and proteinuria. The
health care provider prescribes prednisone (Deltasone) 40 mg twice daily. Which
nursing action should be included in the plan of care?
a. Institute seizure precautions.
b. Reorient to time and place PRN.
c. Monitor intake and output.
d. Place on cardiac monitor.
Answer: C
Rationale: Lupus nephritis is a common complication of SLE, and when the patient is
taking corticosteroids, it is especially important to monitor renal function. There is no
indication that the patient is experiencing any nervous system or cardiac problems with
the SLE.
A patient with systemic lupus erythematosus (SLE) who has a facial rash and alopecia
tells the nurse, "I hate the way I look! I never go anyplace except here to the health
clinic." An appropriate nursing diagnosis for the patient is
a. activity intolerance related to fatigue and inactivity.
b. impaired skin integrity related to itching and skin sloughing.
c. social isolation related to embarrassment about the effects of SLE.
d. impaired social interaction related to lack of social skills.
Answer: C
Rationale: The patient's statement about not going anyplace because of hating the way
he or she looks supports the diagnosis of social isolation because of embarrassment
about the effects of the SLE. Activity intolerance is a possible problem for patients with
SLE, but the information about this patient does not support this as a diagnosis. The
rash with SLE is nonpruritic. There is no evidence of lack of social skills for this patient.
A patient with polyarthralgia with joint swelling and pain is being evaluated for systemic
lupus erythematosus (SLE). The nurse knows that the serum test result that is the most
specific for SLE is the presence of
a. rheumatoid factor.
b. anti-Smith antibody (Anti-Sm).
c. antinuclear antibody (ANA).
d. lupus erythematosus (LE) cell prep.
Answer: B
Rationale: The anti-Sm is antibody found almost exclusively in SLE. The other blood
tests are also used in screening but are not as specific to SLE.
Following instruction for a patient with newly diagnosed systemic lupus erythematosus
(SLE), the nurse determines that teaching about the disease has been effective when
the patient says,
a. "I should expect to have a low fever all the time with this disease."
b. "I need to restrict my exposure to sunlight to prevent an acute onset of symptoms."
c. "I should try to ignore my symptoms as much as possible and have a positive
outlook."
d. "I can expect a temporary improvement in my symptoms if I become pregnant.
Answer: B
Rationale: Sun exposure is associated with SLE exacerbation, and patients should use
sunscreen with an SPF of at least 15 and stay out of the sun between 11:00 AM and
3:00 PM. Low-grade fever may occur with an exacerbation but should not be expected
all the time. A positive attitude may decrease the incidence of SLE exacerbations, but
patients are taught to self-monitor for symptoms that might indicate changes in the
disease process. Symptoms may worsen during pregnancy and especially during the
postpartum period.
A 19-year-old patient who is taking azathioprine (Imuran) for systemic lupus
erythematosus has a check-up before leaving home for college. The health care
provider writes all of these orders. Which one should the nurse question?
a. Naproxen (Aleve) 200 mg BID
b. Give measles-mumps-rubella (MMR) immunization
c. Draw anti-DNA titer
d. Famotidine (Pepcid) 20 mg daily
Answer: B
Rationale: Live virus vaccines, such as rubella, are contraindicated in a patient taking
immunosuppressive drugs. The other orders are appropriate for the patient.
A client is suspected of having systemic lupus erythematosus. The nurse monitors the
client, knowing that which of the following is one of the initial characteristic signs of
systemic lupus erythematosus?

a) weight gain
b) subnormal temperature
c) elevated red blood cell count
d) rash on the face across the bridge of the nose and on the cheeks
D
- Skin lesions or rash on the face across the bridge of the nose and on the cheeks is an
initial characteristic sign of systemic lupus erythematosus (SLE). Fever and weight loss
may also occur. Anemia is most likely to occur later in SLE.
The nurse provides home care instructions to a client with systemic lupus
erythematosus and tells the client about methods to manage fatigue. Which statement
by the client indicates a need for further instructions?

a) I should take hot baths because they are relaxing


b) I should sit whenever possible to conserve my energy
c) I should avoid long periods of rest because it causes joint stiffness
d) I should do some exercises, such as walking, when I am not fatigued
A
- To help reduce fatigue in the client with systemic lupus erythematosus, the nurse
should instruct the client to sit whenever possible, avoid hot baths (because they
exacerbate fatigue), schedule moderate low-impact exercises when not fatigued, and
maintain a balanced diet. The client is instructed to avoid long periods of rest because it
promotes joint stiffness.
A 32-year-old female with systemic lupus erythematosus (SLE) complains that her
hands become pale, blue, and painful when exposed to the cold. What disorder should
the nurse cite as an explanation to these sign and symptoms?
a) Buerger's disease occur as a result to cold exposure.
b) Guillain Barre Syndrome results from very cold climate.
c) Raynaud's disease results from reduced blood flow to the lower extremities when
exposed to cold or stress.
d) Multiple Sclerosis disease occur as a result to cold exposure.
C
The nurse is assigned to care for a client with systemic lupus erythematosus. The nurse
plans care, knowing that this disorder is a(n):

a) local rash that occurs as a result of allergy


b) disease caused by overexposure to sunlight
c) inflammatory disease of collagen contained in connective tissue
d) disease caused by the continuous release of histamine in the body
C
- Systemic lupus erythematosus is an inflammatory disease of collagen in connective
tissue. Options A, B, and D are not associated with this disease.
The nurse is assigned to care for a client admitted to the hospital with a diagnosis of
systemic lupus erythematosus. The nurse reviews the physician's orders, expecting to
note that which type of medication is prescribed?

a) antibiotic
b) antidiarrheal
c) corticosteroid
d) opioid analgesic
C
- Treatment of systemic lupus erythematosus is based on the systems involved and
symptoms. Treatment normally consists of anti-inflammatory drugs, corticosteroids, and
immunosuppressants. Options A, B, and D are not standard components of medication
therapy.
The nurse is teaching a client with Systemic lupus erythematosus (SLE) about self care.
Which of the following teaching points is most important for the nurse to review with the
client?
Encourage the client to apply suntan lotion SPF 8 and higher to skin before exposure.
Encourage the client to verbalize feelings of anxiety related to the diagnosis.
Encourage the client to perform good hand washing after using the bathroom and
before eating.
Encourage the client to utilize clean technique in caring for any wound care.
Encourage the client to perform good hand washing after using the bathroom and
before eating.
Rationale: # 1 is incorrect because suntan lotion of an SPF 15, 30 or higher should be
used since sun exposure can bring on a flare. # 2 is incorrect because psychological
needs do not precede physical needs. # 3 is correct. Hand washing is the most
important client needs because hand washing reduces the risk of infection with
endogenous organisms. # 4 is incorrect because clean technique is not indicated. The
use of strict aseptic technique in caring for intravenous lines and indwelling urinary
catheters or performing any wound care is indicated. Aseptic technique offers protection
against external and resident host microorganisms.
The nurse is caring for a client with systemic lupus erythematosis (SLE). The major
complication associated with systemic lupus erythematosis is:

a. Nephritis

b. Cardiomegaly

c. Desquamation

d. Meningitis
Answer A is correct.

The major complication of SLE is lupus nephritis, which results in end-stage renal
disease. SLE affects the musculoskeletal, integumentary, renal, nervous, and
cardiovascular systems, but the major complication is renal involvement; therefore,
answers B and D are incorrect. Answer C is incorrect because the SLE produces a
"butterfly" rash, not desquamation.
A patient is diagnosed with a systematic lupus erythematous (SLE). SLE primarily
attacks which tissues?
a. Heart
b. Lung
c. Nerve
d. Connective
D
A classic sign of SLE is:
a. Rashes over the cheeks and nose
b. Weight loss
c. Vomiting
d. Difficulty urinating
A
A sign of neurologic involvement in SLE is manifested by:
a. CVA
b. Infection
c. Psychosis
d. Facial tic
B
A laboratory test result that supports the diagnosis of SLE is:
a. Leukocytosis, elevated BUN and CREA
b. Pancytopenia, elevated antinuclear antibody (ANA) titer
c. Thrombocytosis, elevated ESR
d. None of these
B
Which of the following statements when made by the client with systemic lupus
erythematosus (SLE) indicates the need for further teaching?

a) I will wear long-sleeved clothings when I go walking in the morning


b) I will walk in shaded areas only
c) I will go sunbathing in summer
d) I will wear wide-breamed hat when I go to the beach
C
The client had been diagnosed to have systemic lupus erythematosus (SLE). Which of
the following assessment findings should the nurse watch out for?

a) pericardial friction rub


b) elevated blood pressure
c) tachycardia
d) hemoptysis
A
the 26 year old female client is c/o low-grade fever, arthralgias, fatigue, and facial rash.
which lab test should nurse expect HCP to order?
1. complete metabolic panel and LFTs
2. CBC and antinuclear antibody test
3. cholesterol and lipid profile
4. BUN and glomerular filtration test
2. CBC and antinuclear antibody test
the client diagnosed with SLE is being discharged from the medical unit. which
discharge instructions are most important. select all:
1. use sunscreen of SPF 30 or greater
2. notify HCP of fever
3. some dyspnea is expected
4. the hands and feet may change color
5. clt can be cured
1-2-4
the nurse is developing a care plan for a client diagnosed with SLE. which goal is
priority?
1. maintain reproductive ability
2. verbalize feelings of body image
3. no deterioration of organ function
4. client's skin will remain intact
3. no deterioration of organ function
Prednisone prescribed because
the steroids will suppress tissue inflammation, which reduces damages to organs

A client is suspected of having systemic lupus erythematous. The nurse monitors the
client, knowing that which of the following is one of the initial characteristic sign of
systemic lupus erythematous?
Weight gain

Subnormal temperature

Elevated red blood cell count

Rash on the face across the bridge of the nose and on the cheeks
Rash on the face across the bridge of the nose and on the cheeks
Rationale:
Skin lesions or rash on the face across the bridge of the nose and on the cheeks is an
initial characteristic sign of systemic lupus erythematosus (SLE). Fever and weight loss
may also occur. Anemia is most likely to occur later in SLE.
...
Test-Taking Strategy:
Use the process of elimination and note the strategic words "characteristic sign."
Recalling the characteristic butterfly rash associated with SLE will direct you to the
correct option. If you are unfamiliar with this disorder, review this content.
...
A client with pemphigus is being seen in the clinic regularly. The nurse plans care based
on which of the following descriptions of this condition?

The presence of tiny red vesicles

An autoimmune disease that causes blistering in the epidermis

The presence of skin vesicles found along the nerve caused by a virus

The presence of red, raised papules and large plaques covered by silvery scales
An autoimmune disease that causes blistering in the epidermis
Pemphigus
emphigus is an autoimmune disease that causes blistering in the epidermis. The client
has large flaccid blisters (bullae). Because the blisters are in the epidermis, they have a
thin covering of skin and break easily, leaving large denuded areas of skin. On initial
examination, clients may have crusting areas instead of intact blisters. Option 1
describes eczema, option 3 describes herpes zoster, and option 4 describes psoriasis.
The nurse is assigned to care for a client with systemic lupus erythematosus (SLE). The
nurse plans care knowing that this disorder is:
A local rash that occurs as a result of allergy

A disease caused by overexposure to sunlight

An inflammatory disease of collagen contained in connective tissue

A disease caused by the continuous release of histamine in the body


An inflammatory disease of collagen contained in connective tissue
Test-Taking Strategy:
Use the process of elimination. Eliminate option 1 because SLE is a systemic disorder,
not a local one. Next eliminate option 2 because of its similarity to option 1. From the
remaining options, select option 3 because of its systemic characteristic. If you are
unfamiliar with this disorder, review its characteristics.
...
The nurse is assigned to care for a client admitted to the hospital with a diagnosis of
systemic lupus erythematosus (SLE). The nurse reviews the health care provider's
prescriptions. Which of the following medications would the nurse expect to be
prescribed?
Antibiotic

Antidiarrheal

Corticosteroid

Opioid analgesic
Corticosteroid
Rationale:
Treatment of SLE is based on the systems involved and symptoms. Treatment normally
consists of anti-inflammatory drugs, corticosteroids, and immunosuppressants. The
incorrect options are not standard components of medication therapy for this disorder.
...
The community health nurse is conducting a research study and is identifying clients in
the community who are at risk for latex allergy. Which client population is at most risk
for developing this type of allergy?
Hairdressers

The homeless

Children in day care centers

Individuals living in a group home


Hairdressers
Rationale:
Individuals at risk for developing a latex allergy include health care workers; individuals
who work in the rubber industry; individuals having multiple surgeries; individuals with
spina bifida; individuals who wear gloves frequently such as food handlers,
hairdressers, and auto mechanics; and individuals allergic to kiwis, bananas,
pineapples, tropical fruits, grapes, avocados, potatoes, hazelnuts, and water chestnuts.
...
The home care nurse is collecting data from a client who has been diagnosed with an
allergy to latex. In determining the client's risk factors associated with the allergy, the
nurse questions the client about an allergy to which food item?
Eggs

Milk

Yogurt

Bananas
Bananas
Rationale:
Individuals who are allergic to kiwis, bananas, pineapples, tropical fruits, grapes,
avocados, potatoes, hazelnuts, and water chestnuts are at risk for developing a latex
allergy. This is thought to be due to a possible cross-reaction between the food and the
latex allergen. The incorrect options are unrelated to latex allergy.
...
Test-Taking Strategy:
Use the process of elimination and knowledge regarding the food items related to a
latex allergy. Eliminate the incorrect options because they are comparable or alike and
relate to dairy products. Review the food items that are associated with a risk for latex
allergy if you had difficulty with this question.
...
A nurse is assigned to care for a client who returned home from the emergency
department following treatment for a sprained ankle. The nurse notes that the client was
sent home with crutches that have rubber axillary pads and needs instructions regarding
crutch walking. On data collection, the nurse discovers that the client has an allergy to
latex. Before providing instructions regarding crutch walking, the nurse should:
Contact the health care provider (HCP).

Cover the crutch pads with cloth.

Call the local medical supply store, and ask for a cane to be delivered.

Tell the client that the crutches must be removed immediately from the house.
Cover the crutch pads with cloth.
Rationale:
The rubber pads used on crutches may contain latex. If the client requires the use of
crutches, the nurse can cover the pads with a cloth to prevent cutaneous contact.
Option 4 is inappropriate and may alarm the client. The nurse cannot prescribe a cane
for a client. In addition, this type of assistive device may not be appropriate, considering
this client's injury. No reason exists to contact the HCP at this time.
...
Test-Taking Strategy:
Use the process of elimination and knowledge regarding the alternative resources for a
client with an allergy to latex. No data in the question support the need to contact the
HCP. The nurse should not prescribe assistive devices for the client. Option 4 is not a
therapeutic action. Review care to the client with a latex allergy if you had difficulty with
this question.
...
The home care nurse is ordering dressing supplies for a client who has an allergy to
latex. The nurse asks the medical supply personnel to deliver which of the following?

Elastic bandages

Adhesive bandages

Brown Ace bandages

Cotton pads and silk tape


Cotton pads and silk tape
Test-Taking Strategy:
Use the process of elimination and knowledge regarding the products that contain latex
to answer this question. Eliminate options 1 and 3 first because they are comparable or
alike. Noting the strategic words, "cotton" and "silk," in option 4 will assist you in
answering correctly from the remaining options. Review the list of products that contain
latex if you had difficulty with this question.
...
A nurse is assisting in developing a plan of care for a client with immunodeficiency. The
nurse understands that which problem is a priority for the client?

Infection

Inability to cope

Lack of information about the disease

Feeling uncomfortable about body changes


Infection
Rationale:
The client with immunodeficiency has inadequate or an absence of immune bodies and
is at risk for infection. The priority problem is infection. The question presents no data
indicating that options 2, 3, or 4 are a problem.
...
Test-Taking Strategy:
Use Maslow's Hierarchy of Needs theory to answer the question. Recall that
physiological needs are the priority. This will easily direct you to option 1. Review the
care of a client with immunodeficiency if you had difficulty with this question.
...
A client calls the emergency department and tells the nurse that he received a bee sting
to the arm while weeding a garden. The client states that he has received bee stings in
the past and is not allergic to bees. The client states that the site is painful and asks the
nurse for advice to alleviate the pain. The nurse tells the client to first:

Take two acetaminophen (Tylenol).

Place a heating pad to the site.

Apply ice and elevate the site.

Lie down and elevate the arm.


Apply ice and elevate the site.
Rationale:
When a bee sting occurs and is painful, it is best to treat the site locally rather than
systemically. Pain may be alleviated by the application of an ice pack and elevating the
site. A heating pad will increase discomfort at the site. Acetaminophen may be taken by
the client to assist in alleviating discomfort, but this would not treat the injury at a local
level. Lying down and elevating the arm may have some effect on reducing edema at
the site but will not directly assist in alleviating the pain at the site of injury.
...
est-Taking Strategy:
Use the process of elimination, noting the strategic word, "first." Focus on the subject,
that the site of the bee sting is painful. Eliminate option 1 because this measure
produces a systemic and not a local effect. Eliminate option 4 next because this
measure will not directly assist in alleviating the pain at the site of injury. From the
remaining options, recalling the effects of heat will assist in eliminating option 2 and
direct you to option 3. Review the initial measures that will alleviate pain from a bee
sting if you had difficulty with this question.
...
Study Mode
Question 23 of 64

A client with human immunodeficiency virus (HIV) who has contracted tuberculosis (TB)
asks the nurse how long the medication therapy lasts. The nurse responds that the
duration of therapy would likely be for at least:
6 total months and at least 1 month after cultures convert to negative

6 total months and at least 3 months after cultures convert to negative

9 total months and at least 3 months after cultures convert to negative


9 total months and at least 6 months after cultures convert to negative
9 total months and at least 6 months after cultures convert to negative
Rationale:
The client with tuberculosis who is coinfected with HIV requires that antitubercular
therapy last longer than usual. The prescription is usually for a total of 9 months and at
least 6 months after sputum cultures convert to negative.
...
Test-Taking Strategy:
Use the process of elimination. Knowing that the client with HIV requires longer therapy
helps you eliminate options 1 and 2 first. To select between the remaining options, it is
necessary to recall that sputum cultures must be negative for 6 months before
terminating medication therapy because of the immunosuppressed status of the client.
Review the guidelines related to medication therapy in the client with TB if you had
difficulty with this question.
...
A client who is human immunodeficiency virus (HIV) positive has had a Mantoux skin
test. The results show a 7-mm area of induration. The nurse evaluates that this result is:
Positive
Rationale:
The client with HIV is considered to have positive results on Mantoux skin testing with
an area of 5 mm of induration or greater. The client without HIV is positive with
induration greater than 10 or 15 mm if the client is at low risk. The client with HIV is
immunosuppressed, making a smaller area of induration positive for this type of client. It
is also possible for the client infected with HIV to have false negative readings because
of the immunosuppression factor.
...
Test-Taking Strategy:
Use the process of elimination to answer the question. Begin by eliminating options 2
and 3 because they are comparable or alike. Remembering that the client with HIV is
immunosuppressed will assist in directing you to option 4, the correct option. Review the
procedures for interpreting the results of this test if you had difficulty with this question.
...
A client with acquired immunodeficiency syndrome (AIDS) is taking zidovudine
(Retrovir) 200 mg orally three times daily. The client reports to the health care clinic for
follow-up blood studies, and the results of the blood studies indicate severe
neutropenia. Which of the following would the nurse anticipate to be prescribed for the
client?
Reduction in the medication dosage

Discontinuation of the medication

The administration of prednisone concurrent with the therapy

Administration of epoetin alfa (Epogen)


Discontinuation of the medication
Rationale:
Hematological monitoring should be done every 2 weeks in the client taking zidovudine.
If severe anemia or severe neutropenia develops, treatment should be discontinued
until there is evidence of bone marrow recovery. If anemia or neutropenia is mild, a
reduction in dosage may be sufficient. The administration of prednisone may further
alter the immune function. Epoetin alfa is given to clients experiencing anemia.
...
Test-Taking Strategy:
Knowledge regarding the adverse effects related to the administration of zidovudine is
required to answer this question. Focus on the strategic words, "severe neutropenia," to
assist in directing you to the correct option. Review the adverse effects of this
medication if you had difficulty with this question.
...
A client with acquired immunodeficiency syndrome (AIDS) is taking didanosine (Videx).
The client calls the nurse at the health care provider's office and reports nausea,
vomiting, and abdominal pain. Which of the following instructions would the nurse
provide to the client?

This is an expected side effect of the medication.

Come to the office to be seen by the health care provider.

Take crackers and milk with the administration of the medication.

Decrease the dose of the medication until the next health care provider's visit.
Come to the office to be seen by the health care provider.
Rationale:
Pancreatitis, which can be fatal, is the major dose-limiting toxicity associated with the
administration of didanosine. Clients should be monitored for indications of developing
pancreatitis, which include increased serum amylase in association with increased
serum triglycerides, decreased serum calcium, and nausea, vomiting, or abdominal
pain. If evolving pancreatitis is diagnosed, the medication should be discontinued. The
client should be seen by the health care provider.
...
Focus on the data in the question. Recalling that nausea, vomiting, and abdominal pain
are signs of pancreatitis and that pancreatitis is associated with the use of this
medication should direct you to the correct option. Review the adverse effects of this
medication if you had difficulty with this question.
...
A nurse is assisting in developing a plan of care for a client with acquired
immunodeficiency syndrome (AIDS) who is experiencing night fever and night sweats.
Which nursing intervention should the nurse suggest including in the plan of care to
manage this symptom?
Keep the call bell within reach for the client.

Administer a sedative at bedtime.


Administer an antipyretic at bedtime.

Provide a back rub and comfort measures before bedtime.


Administer an antipyretic at bedtime.
Rationale:
For clients with AIDS who experience night fever and night sweats, it is useful to offer
an antipyretic at bedtime. It is also helpful to keep a change of bed linens and night
clothes nearby for use. The pillow should have a plastic cover, and a towel may be
placed over the pillowcase if there is profuse diaphoresis. The client should have liquids
at the bedside to drink. Options 1, 2, and 4 are important interventions but they are
unrelated to the subject of fever and night sweats.
...
est-Taking Strategy:
Focus on the subject of the question, which is night fever and night sweats. Options 1,
2, and 4 are helpful and important interventions but do not address the subject of the
question. Because night fever and sweats occur serially, it is helpful to give the
antipyretic before sleep as a prophylactic measure. Review nursing measures for the
client with AIDS if you had difficulty with this question.
...
A nurse is assisting in preparing a plan of care for a client with acquired
immunodeficiency syndrome (AIDS) who has nausea. Which dietary measure should
the nurse include in the plan?
Dairy products with each snack and meal

Red meat daily

Adding spices to food to make the taste more palatable

Foods that are at room temperature


Foods that are at room temperature
Rationale:
The client with AIDS experiencing nausea should avoid fatty products, such as dairy
products and red meat. Meals should be small and frequent to lessen the chance of
vomiting. Spices and odorous foods should be avoided because they aggravate
nausea. Foods are best tolerated either cold or at room temperature.
...
Test-Taking Strategy:
Use knowledge related to the effects of AIDS on the gastrointestinal system to assist in
answering the question. Additionally, general principles related to nutrition in a client
with an immunosuppressive disorder will assist in directing you to option 4. Review
dietary measures for the client with AIDS if you had difficulty with this question.
...
A nurse is assisting in developing a plan of care for a pregnant client with acquired
immunodeficiency syndrome (AIDS). The nurse determines that which of the following is
the priority concern for this client?
Inability to care for self at home

Development of an infection

Lack of available support services

Isolation
Development of an infection
Rationale:
Acquired immunodeficiency syndrome decreases the body's immune response, making
the infected person susceptible to infections. AIDS affects helper T lymphocytes, which
are vital to the body's defense system. Opportunistic infections are a primary cause of
death in people affected with AIDS. Therefore preventing infection is a priority of nursing
care. Although the concerns in options 1, 3, and 4 may need to be addressed at some
point in the care of the client, these are not the priority.
...
Test-Taking Strategy:
Note the strategic word, "priority." Use Maslow's Hierarchy of Needs theory to eliminate
options 1, 3, and 4. Also, recalling that AIDS affects the body's immune system will
direct you to option 2. Review the priority concerns related to the client with AIDS if you
had difficulty with this question.
...
Study Mode
Question 31 of 64

The nurse is assessing a client who has small groups of vesicles over his chest and
upper abdominal area. They are located only on the right side of his body. The client
states his pain level is 8/10, and describes the pain as burning in nature. Which
question is most appropriate to include in the data collection?
Did you have chicken pox as a child?"

"How many sexual partners have you had?"

"Did you use an electric blanket on your side?"

"Why don't you try docosanol cream (Abreva) on your lesions


Did you have chicken pox as a child?"
ationale:
The client has the symptoms of herpes zoster, or shingles, which is caused by the same
organism as chicken pox. Asking about sexual partners is inappropriate for this
disorder. An electric blanket use does not cause this type of lesions. Abreva is used on
herpes simplex I (cold sores).
...
Test-Taking Strategy:
Recall the classic symptoms of herpes zoster: clear vesicles in groups, pain, and
lesions that occur on one side of the body. This will assist in determining that the client
has shingles. From this point, recall that this disorder is associated with the chicken pox
virus. Review the manifestations of shingles if you had difficulty with this question.
...
The nurse interprets that the client who is prescribed zalcitabine (Hivid) is experiencing
an adverse effect of this medication when which event is reported by the client?
Diarrhea

Tinnitus

Burning with urination

Numbness in the legs


Numbness in the legs
Rationale:
Peripheral neuropathy is an adverse effect associated with the use of zalcitabine, which
manifests initially as numbness and burning sensations in the extremities. They may
progress to sharp shooting pains or severe continuous burning if the medication is not
withdrawn. The other options are not associated with use of this medication.
...
A nurse is collecting data on a client with rheumatoid arthritis. The nurse looks at the
client's hands and notes these characteristic deformities. The nurse identifies this
deformity as: Refer to figure.
Ulnar drift

Rheumatoid nodules

Swan neck deformity

Boutonniere deformity
Ulnar drift
Rationale:
All of the conditions identified in the options can occur in rheumatoid arthritis. Ulnar drift
occurs when synovitis stretches and damages the tendons, and eventually the tendons
become shortened and fixed. This damage causes subluxation (drift) of the joints.
...
A nurse is collecting data on a client who complains of fatigue, weakness, malaise,
muscle pain, joint pain at multiple sites, anorexia, and photosensitivity. Systematic lupus
erythematosus (SLE) is suspected. The nurse further checks for which of the following
that is also indicative of the presence of SLE?

Emboli

Ascites

Two hemoglobin S genes


Butterfly rash on cheeks and bridge of nose
Butterfly rash on cheeks and bridge of nose
Rationale:
SLE is a chronic inflammatory disease that affects multiple body systems. A butterfly
rash on the cheeks and on the bridge of the nose is a classic sign of SLE. Option 3 is
found in sickle cell anemia. Options 1 and 2 may be found in many conditions but are
not associated with SLE.
...
A health care provider aspirates synovial fluid from a knee joint of a client with
rheumatoid arthritis. The nurse reviews the laboratory analysis of the specimen and
would expect the results to indicate which finding?
Cloudy synovial fluid

Presence of organisms

Bloody synovial fluid

Presence of urate crystals


Cloudy synovial fluid
Rationale:
Cloudy synovial fluid is diagnostic of rheumatoid arthritis. Organisms present in the
synovial fluid are characteristic of a septic joint condition. Bloody synovial fluid is seen
with trauma. Urate crystals are found in gout.
...
se the process of elimination. Remember that organisms indicate infection, blood
indicates trauma, and urates indicate gout. Review the characteristics of rheumatoid
arthritis if you had difficulty with this question.
...
Which client is at the highest risk for systemic lupus erythematous (SLE)?
An Asian male

A white female

An African-American male

An African-American female
An African-American female
A client with acquired immunodeficiency syndrome (AIDS) has difficulty swallowing. The
nurse has given the client suggestions to minimize the problem. The nurse determines
that the client has understood the instructions if the client verbalized to increase intake
of foods such as:
Raw fruits and vegetables

Hot soup
Peanut butter

Puddings
Puddings
Rationale:
The client is instructed to avoid spicy, sticky, or excessively hot or cold foods. The client
also is instructed to avoid foods that are rough, such as uncooked fruits or vegetables.
The client is encouraged to take in foods that are mild, nonabrasive, and easy to
swallow. Examples of these include baked fish, noodle dishes, well-cooked eggs, and
desserts such as ice cream or pudding. Dry grain foods such as crackers, bread, or
cookies may be softened in milk or another beverage before eating.
...
Test-Taking Strategy:
Use the process of elimination, and focus on the subject of difficulty swallowing.
Evaluate each of the foods listed in terms of how easily they are swallowed. The rough,
hot, and sticky foods in options 1, 2, and 3, respectively, help you to choose option 4 as
the correct option. Review nutritional concepts for the AIDS client if you had difficulty
with this question.
...
A client with acquired immunodeficiency syndrome (AIDS) is experiencing shortness of
breath related to Pneumocystis jiroveci pneumonia. Which measure should the nurse
suggest to assist the client in performing activities of daily living?

Provide supportive care with hygiene needs.

Provide meals and snacks with high protein, high calorie, and high nutritional value.

Provide small, frequent meals.

Offer low microbial food.


Provide supportive care with hygiene needs.


Rationale:
Providing supportive care with hygiene needs as needed reduces the client's physical
and emotional energy demands and conserves energy resources for other functions
such as breathing. Options 2, 3, and 4 are important interventions for the client with
AIDS but do not address the subject of activities of daily living. Option 2 will assist the
client in maintaining appropriate weight and proper nutrition. Option 3 will assist the
client in tolerating meals better. Option 4 will decrease the client's risk of infection.
...
Test-Taking Strategy:
Focus on the subject, performing activities of daily living. Options 2, 3, and 4 are
important interventions for the client with AIDS but do not address the subject. Option 1
is the only option that addresses the subject of the question. Also note that options 2, 3,
and 4 are comparable or alike and relate to nutrition. Review care to the client with
AIDS if you had difficulty with this question.
...
A client with acquired immunodeficiency syndrome has a respiratory infection from
Pneumocystis jiroveci and a nursing diagnosis of Impaired Gas Exchange written in the
plan of care. Which of the following indicates that the expected outcome of care has not
yet been achieved?
The client limits fluid intake.

The client has clear breath sounds.

The client expectorates secretions easily.

The client is free of complaints of shortness of breath.


The client limits fluid intake.


Study Mode
Question 53 of 64

A client with acquired immunodeficiency syndrome has a respiratory infection from


Pneumocystis jiroveci and a nursing diagnosis of Impaired Gas Exchange written in the
plan of care. Which of the following indicates that the expected outcome of care has not
yet been achieved?
Rationale:
The status of the client with a nursing diagnosis of Impaired Gas Exchange would be
evaluated against the standard outcome criteria for this nursing diagnosis. These would
include that the client breathes easier, coughs up secretions effectively, and has clear
breath sounds. The client should not limit fluid intake because fluids are needed to
decrease the viscosity of secretions for expectoration.
...
Test-Taking Strategy:
Use the process of elimination, and note the strategic words, "expected outcome" and
"has not yet been achieved." These words indicate a negative event query and ask you
to select an option that is an incorrect statement. This will direct you to option 1. Review
care to the client with acquired immunodeficiency syndrome if you had difficulty with this
question.
...
A client reports to the health care clinic to obtain testing regarding human
immunodeficiency virus (HIV) status after being exposed to an individual who is HIV
positive. The test results are reported as negative, and the client tells the nurse that he
feels so much better knowing that he had not contracted HIV. The nurse explains the
test results to the client, telling the client that:
There is no further need for testing.
A negative HIV test is considered accurate.

A negative HIV test is not considered accurate during the first 6 months after exposure.

The test should be repeated in 1 week.


A negative HIV test is not considered accurate during the first 6 months after exposure.
Rationale:
A test done for HIV should be repeated. There might be a lag period after the infection
occurs and before antibodies appear in the blood. Therefore a negative HIV test is not
considered accurate during the first 6 months after exposure.
...
Test-Taking Strategy:
Use the process of elimination. Eliminate options 1 and 2 first because they are
comparable or alike. From the remaining options, recalling that antibodies do not appear
immediately in the blood will assist in directing you to option 3. Review HIV testing
procedures if you had difficulty with this question.
...

You might also like